Vous êtes sur la page 1sur 142

@lgebra Linear

Geovane Krger
geovanek @ gmail.com

Estudos Sobre

Sumrio

1 Matrizes Exerccios . . . . . . . . . . . . . . . . . . . . . . . . . . . . . . . . . . . . . . . . . . . . . 2 Sistemas Lineares 3 Espaos Vetoriais 4 Independncia Linear, Base e Dimenso 5 Transformaes Lineares 6 Ortogonalidade 7 Determinantes A Notao Sigma ou Notao Somatrio

2 14 18 40 55 73 93 107 127

Exerccios . . . . . . . . . . . . . . . . . . . . . . . . . . . . . . . . . . . . . . . . . . . . . 129 B Matrizes Elementares 130

Exerccios . . . . . . . . . . . . . . . . . . . . . . . . . . . . . . . . . . . . . . . . . . . . . 135 C O Processo de Gauss-Jordan D Inversas Generalizadas Bibliogra 136 137 141

1
Matrizes
Denio 1.1: Uma matriz Amn , um arranjo retangular de m, n nmeros reais (ou complexos) denominados entradas, arrumados em m linhas e n colunas, onde m n representa o tamanho da matriz: a11 a12 a1n a21 a22 a2n A= (1.1) . . .. . . . . . . . . am1 am2 amn A i-sima linha de A ai1 ai2 A j -sima coluna de A a1j a2j . . . amj ain (1 i m)

(1

m)

Dizemos que A m por n ( e escrevemos m n). Uma matriz A do tipo m n tambm pode ser indicada por: A = (aij ); i {1, 2, 3, . . . , m} e j {1, 2, 3, . . . , n} ou simplesmente A = (aij )mn , onde aij indica cada elemento da matriz. O ndice i representa a linha e o ndice j a coluna s quais o elemento pertence.

1 Matrizes

Uma matriz Amn pode ser representada por: a11 a21 A= . . . a12 a22 . . . .. . a1n a11 a12 a2n a22 a , A = 21 . . . .. . . . . . . . amn am1 am2 a1n a11 a12 a2n a21 a22 ou A = . . . .. . . . . . . . amn am1 am2 a1n a2n . . . . amn

am1 am2

No que segue, denimos alguns tipos especiais de matrizes. Denio 1.2: Duas matrizes A = (aij ) e B = (bij ), so ditas iguais se aij = bij , para i = 1, 2, . . . , m e j = 1, 2, . . . , n, isto , se os elementos correspondentes forem iguais. Denio 1.3 (Matriz Quadrada): aquela cujo nmero de linhas igual ao nmero de colunas, ou seja (m = n). a11 a12 a13 A = a21 a22 a23 (1.2) a31 a32 a33
33

Denio 1.4 (Matriz Nula): aquela em que aij = 0, para todo i e j . A= 0 0 0 0 0 0 (1.3)
23

Denio 1.5 (Matriz-Coluna): aquela que possui uma nica coluna (n = 1). a11 a21 A= . . . am1

(1.4)
m1

Denio 1.6 (Matriz-Linha): aquela que possui uma nica linha (m = 1). A = a11 a12 . . . a1n
1n

(1.5)

Denio 1.7 (Matriz Diagonal): uma matriz quadrada A = (aij ) em que todos os elementos fora da diagonal principal so nulos, isto , aij = 0 para i = j . a11 0 0 A = 0 a22 0 0 0 a33 33

(1.6)

..............................................................................................

1 Matrizes

Denio 1.8 (Matriz Escalar): uma matriz diagonal A = (aij ) em que todos os elementos da diagonal principal so iguais, isto , aij = c (c=constante) para i = j e aij = 0 para i = j . c 0 0 (1.7) A = 0 c 0 0 0 c 33 Denio 1.9 (Matriz Identidade): uma matriz quadrada, representada por In , I = (ij ) em que todos os elementos da diagonal principal so 1, isto , ij = 1 para i = j e ij = 0 para i = j . 1 0 0 A = 0 1 0 (1.8) 0 0 1 33 Denio 1.10 (Matriz Simtrica): uma matriz quadrada A = (aij ) onde aij = aji . a b c d b e f g A= c f h i d g i j
44

(1.9)

Denio 1.11 (Matriz Triangular Superior): uma matriz quadrada A = (aij ) onde todos os elementos abaixo da diagonal so nulos, isto , m = n e aij = 0 para i > j . a11 a12 a13 A = 0 a22 a23 (1.10) 0 0 a33 33 Denio 1.12 (Matriz Triangular Inferior): uma matriz quadrada A = (aij ) onde todos os elementos acima da diagonal so nulos, isto , m = n e aij = 0 para i < j . a11 0 0 A = a21 a22 0 (1.11) a31 a32 a33 33 Vamos denir agora, uma srie de operaes que produziro novas matrizes a partir de matrizes dadas. Essas operaes sero teis nas aplicaes de matrizes. Denio 1.13 (Adio de Matriz): Sejam as matrizes A = (aij )mn e B = (bij )mn , a soma de A e B a matriz Cmn denida por: cij = aij + bij (1 i m, 1 j n)

..............................................................................................

1 Matrizes E XEMPLO 1: Sejam A= ento, A+B = 1 + 0 2 + 2 4 + (4) 1 0 = 2 + 1 1 + 3 3+1 3 2 0 4 1 2 4 2 1 3 eB= 0 2 1 3 4 1

Observao: A soma de duas matrizes A e B est denida apenas quando A e B so do mesmo tamanho. Teorema 1.14 (Propriedades da Soma de Matrizes): Sejam Amn , Bmn , Cmn e Dmn matrizes i) A + B = B + A (Comutatividade) ii) A + (B + C ) = (A + B ) + C (Associatividade) iii) Existe uma nica matriz Omn tal que A + O = A, onde O chamada matriz nula ou elemento neutro para a soma de matrizes. iv) Para cada matriz Amn , existe uma nica matriz Dmn tal que, A + D = O. Denotaremos D por A, de modo que A + D = O pode ser escrito da forma A + (A) = O. A matriz A chamada de matriz inversa aditiva ou negativa de A. Demonstrao. i) Fazendo A + B = X e B + A = Y , temos: xij = aij + bij = bij + aij = yij , ii) Fazendo A + (B + C ) = X e (A + B ) + C = Y , temos: xij = aij + (bij + cij ) = (aij + bij ) + cij = yij , iii) Seja U = (uij ). Ento, A + U = A se, e somente se, aij + uij = aij que vlido se, e somente se uij = 0. Logo Umn a matriz que tem os elementos nulos; U denotado por O.
iv) Seja A = (a ij ). Ento, A + A = O se, e somente se,

i, j.

i, j.

aij + a ij = 0
que vlido se, e somente se, a ij = aij . Logo Amn a matriz no qual cada elemento simtrico do correspondente em A; A denotada por A.

..............................................................................................

1 Matrizes

Denio 1.15 (Multiplicao Por Um Escalar): Se A = (aij )mn uma matriz e r um nmero real, ento o mltiplo escalar de A por r , rA, a matriz B = (bij )mn , onde bij = raij , (1 i m, 1 j n)

em outras palavras, B obtido multiplicando-se cada elemento de A por r .


E XEMPLO 2: Se r = 3 e A = 1 4 ento, 4 3 rA = 3 1 4 4 (3) 1 (3) (4) 3 12 = = 3 (3) 4 (3) (3) 12 9

Teorema 1.16 (Propriedades da Multiplicao Por Escalar): Se r e s so nmeros reais e A e B so matrizes, ento: i) r (sA) = (rs)A ii) (r + s)A = rA + sA iii) r (A + B ) = rA + rB Demonstrao. i) Seja A = (aij ) e r, s R. Ento, r (sA) = r (saij ) = (rs)aij = (rs)A ii) Seja A = (aij ) e r, s R. Logo, (r + s)A = (r + s)aij = raij + saij = rA + sA iii) Seja A = (aij ), B = [bij ] e r R. Ento, r (A + B ) = r (aij + bij ) = raij + rbij = rA + rB Denio 1.17 (Transposta de Uma Matriz): Se A = (aij )mn uma matriz, ento a matriz AT = (aT ij )mn , onde aT (1 i n, 1 j m) ij = aji , chamada de transposta de A. Portanto, a transposta de A obtida trocando-se as linhas de A por suas colunas e vice-versa ..............................................................................................

1 Matrizes E XEMPLO 3: 2 Seja A = 0 1 1 2 0 3 sua transposta ser AT = 1 3 4

1 4

Teorema 1.18 (Propriedades da Transposta): Se r um escalar e A, B so matrizes, ento: i) (AT )T = A ii) (A + B )T = AT + B T iii) (AB )T = B T AT iv) (rA)T = rAT Demonstrao. i) Seja A = (aij ). Logo,
T T (AT )T = (aT ij ) = (aji ) = aij = A

ii) Seja A = (aij ) e B = (bij ). Ento,


T T T (A + B )T = (aij + bij )T = aji + bji = aT ij + bij = A + B

iii) Sejam A = (aij )mp e B = (bij )pn . Ento, o elemento (i, j ) de (AB )T cT ij . Por outro lado, cT ij = cji = linhaj (A) coli (B ) = aj 1 b1i + aj 2 b2i + . . . + ajp bpi
T T T T T T T T T T T = aT 1j bi1 + a2j bi2 + . . . + apj bip = bi1 a1j + bi2 a2j + . . . + bip apj

= linhai (B T ) colj (AT ) que o elemento (i, j ) de B T AT . iv) Seja A = (aij ) e r R. Logo,
T (rA)T = (raij )T = raji = raT ij = rA

Denio 1.19 (Multiplicao de Matrizes): Sejam A = (aij )mp e B = (bij )pn matrizes, ento o produto de A e B , denotado por AB , a matriz C = (cij )mn denida por
p

cij = ai1 b1j + ai2 b2j + . . . + aip bpj =


k =1

aik bkj ,

(1

m, 1

n)

..............................................................................................

1 Matrizes

Observao: Note que o produto de A e B esta denido apenas quando o nmero de linhas de B exatamente igual ao nmero de colunas de A.
E XEMPLO 4: Sejam A = 1 2 3 1 2 5 1 e B = 4 3, ento, 4 2 1 (1) (2) + (2) (4) + (1) (2) (1) (5) + (2) (3) + (1) (1) (3) (2) + (1) (4) + (4) (2) (3) (5) + (1) (3) + (4) (1) AB = 4 6 2 16

AB =

Teorema 1.20 (Propriedades da Multiplicao de Matrizes): i) Se A, B e C tm os tamanhos apropriados, ento, A(BC ) = (AB )C ii) Se A, B e C tm os tamanhos apropriados, ento, A(B + C ) = AB + AC iii) Se A, B e C tm os tamanhos apropriados, ento, (A + B )C = AC + BC iv) AI = IA = A v) OA = AO = O vi) (kA)B = A(kB ) = k(AB ), Demonstrao. i) Esta demonstrao mais delicada. Sejam A = (aij )mp , B = (bjk )pq e C = (ckl )qn matrizes, respectivamente. Vamos utilizar a notao de somatrio nesta demonstrao.
p p q p q

(Associatividade)

(Distributividade a esquerda)

(Distributividade a direita) (existncia da matriz neutra) (existncia da matriz nula) k R

A(BC ) =
j =1 p q

aij [BC ]jl =


j =1 q

aij
k =1 p

bjk ckl

=
j =1 k =1 q p

aij (bjk ckl ) =

=
k =1 k =1

(aij bjk )ckl =


k =1 j =1 p

(aij bjk )ckl =


k =1 j =1

aij bjk ckl =

=
k =1

[AB ]ik ckl = (AB )C

..............................................................................................

1 Matrizes

ii) A(B + C ) =

aij (B + C )jk =
j =1 p p j =1

aij (bjk + cjk ) =


j =1

(aij bjk + aij cjk ) =

=
j =1

aij bjk +
j =1

aij cjk = [AB ]ik + [AC ]ik = AB + AC

iii)
p P p

(A + B )C =
j =1 P

[A + B ]ij cjk =
j =1 p

(aij + bij )cjk =


j =1

(aij cjk + bij cjk ) =

=
j =1

aij cjk +
j =1

bij cjk = [AC ]ik + [BC ]ik = AC + BC

iv) Sendo I = (ij ) e A = (aij ). Temos: AI = ai1 1j + ai2 2j + . . . + aii ii + . . . + ain nj = = ai1 0 + ai2 0 + ai3 0 + . . . + aii 1 + . . . + ain 0 = aii para todos i e j , ento AI = A. A outra igualdade anloga. v) Seja O = [oij ] e A = [aij ]. Temos: AO = ai1 o1j + ai2 o2j + . . . + ain onj = ai1 0 + ai2 0 + . . . +in 0 = 0 para todos i e j , logo AO = 0. A outra igualdade anloga. vi) (kA)B =
j =1 p p p p

(kaij )bjk =
j =1

(aij k)bjk =
j =1 p

aij (kbjk ) = A(kB ),

A(kB ) =
j =1

aij (kbjk = k
j =1 p p

aij bjk = k(AB )

e (kA)B =

(kaij )bjk = k
j =1 j =1

aij bjk = k(AB )

Denio 1.21: Uma matriz A = (aij )nn (quadrada), dita invertvel (ou no-singular), se existe uma matriz B = (bij )nn , tal que AB = BA = In Se A no tm inversa, dizemos que A no-invertvel (ou singular). ..............................................................................................

1 Matrizes

10

Denio 1.22 (Matriz Inversa): Dada uma matriz invertvel A, chama-se inversa de A, a matriz A1 , tal que AA1 = A1 A = In Teorema 1.23 (Unicidade da Inversa): Se uma matriz A = (aij )nn tem inversa, ento a inversa nica. Demonstrao: Suponhamos que as matrizes B = (bij )nn e C = (cij )nn sejam inversa da matriz A, ento AB = BA = In = CA = AC logo B = BIn = B (AC ) = (BA)C = In C = C Portanto, segue a tese. Teorema 1.24 (Propriedades da Inversa): i) Se A uma matriz invertvel, ento A1 invertvel e (A1 )1 = A ii) Se A = (aij )nn e B = (bij )nn so matrizes invertveis, ento AB invertvel e (AB )1 = B 1 A1 iii) Se A uma matriz invertvel, ento (AT )1 = (A1 )T Demonstrao. i) A1 invertvel se existe uma matriz B tal que A1 B = BA1 = In mas, A1 a inversa de A, logo AA1 = A1 A = In assim, B = A, pois A uma inversa de A1 e, como a inversa nica, conclumos que (A1 )1 = A.

..............................................................................................

1 Matrizes

11

ii) Devemos mostrar que a inversa de AB B 1 A1 , isto , (AB )(B 1 A1 ) = In e que (B 1 A1 )(AB ) = In , logo (AB )(B 1 A1 ) = A(BB 1 )A1 = AIn A1 = AA1 = In e (B 1 A1 )(AB ) = B 1 (A1 A)B = B 1 In B = B 1 B = In Portanto AB invertvel e, como a inversa nica segue que (AB )1 = B 1 A1 . iii) Sabemos que AA1 = In e A1 A = In ento, transpondo essas matrizes, temos
T (AA1 )T = In = In

e
T (A1 A)T = In = In

logo, (A1 )T AT = In e AT (A1 )T = In Portanto (AT )1 = (A1 )T . Corolrio 1.25: Se A1 , A2 , . . . , Ar , r N so matrizes n n, invertveis, ento A1 A2 . . . Ar invertvel e 1 1 1 (A1 A2 . . . Ar )1 = A r . . . A2 A1 . Demonstrao. Vamos provar por induo que vlido. Se r = 1, temos a matriz A1 , que , por hiptese, invertvel. Suponhamos vlido para r = k, para algum k N, isto ,
1 1 1 (A1 A2 . . . Ak )1 = A k . . . A2 A1

(Hiptese de Induo)

Provaremos que vale para r = k + 1, isto ,


1 1 1 1 (A1 A2 . . . Ak Ak+1 )1 = A k +1 Ak . . . A2 A1 .

..............................................................................................

1 Matrizes

12

Da hiptese de induo, obtemos a matriz B = A1 A2 . . . Ak , que invertvel, logo, pelo item ii) do teorema anterior, temos
1 1 1 1 1 1 (A1 A2 . . . Ak Ak+1 )1 = (BAk+1 )1 = A = A k +1 B k +1 Ak . . . A2 A1 .

Portanto, pelo Princpio de Induo, segue a tese. Teorema 1.26: Sejam A e B matrizes n n i) Se AB = In , ento BA = In ii) Se BA = In , ento AB = In

A demonstrao segue diretamente da denio de inversa. Assim, para vericar se uma matriz B uma inversa da matriz A basta efetuar um dos produtos AB ou BA e vericar se igual a In . Observao: Nem toda matriz A tem inversa. Matrizes inversas sero bastante utilizadas aqui nos nossos estudos, e no decorrer do texto, mostraremos vrios mtodos para encontrar a inversa de uma matriz, um mtodo prtico e bastante utilizado mostrado no apndice C. Denio 1.27: Seja A = (aij )nn , ento o trao de A, denotado por T r (A), a soma de todos os elementos da diagonal principal de A, isto , T r (A) = a11 + a22 + . . . + ann Teorema 1.28 (Propriedades do Trao de Uma Matriz): Sejam A = (aij )nn e B = (bij )nn matrizes, ento: i) T r (AT ) = T r (A) ii) T r (cA) = cT r (A) (c R)

iii) T r (A B ) = T r (A) T r (B ) iv) T r (AB ) = T r (BA) v) T r (AT A) Demonstrao. i) Temos que,


n n

T r (A) =
i=1

aii = a11 + a22 + . . . + ann =

aT 11

aT 22

+ ... +

aT nn

=
i=1

T aT ii = T r (A )

..............................................................................................

1 Matrizes

13

ii) Como c R, temos


n

T r (cA) =
i=1

caii = ca11 + ca22 + . . . + cann = c(a11 + a22 + . . . + ann ) = c


i=1

aii = cT r (A)

iii) Lembrando da propriedade de soma de matrizes, temos


n

T r (A B ) =

i=1

(aii bii ) = (a11 b11 ) + (a22 b22 ) + . . . + (ann bnn ) =


n n

= (a11 + a22 + . . . + ann ) (b11 + b22 + . . . + bnn ) =

i=1

aii

i=1

bii = T r (A) T r (B )

iv) Para provarmos que T r (AB ) = T r (BA) vamos utilizar as propriedades de somatrio, que podem ser encontradas no apndice A, logo,
n n n

T r (AB ) =
i=1 k =1

aik bki

=
i=1

(ai1 b1i + ai2 b2i + . . . + ain bni ) =

= (a11 b11 + a21 b12 + . . . + an1 b1n ) + (a12 b21 + a22 b22 + . . . + an2 b2n ) + . . . . . . + (a1n bn1 + a2n bn2 + . . . + ann bnn ) = = (a11 b11 + a12 b21 + . . . + a1n bn1 ) + (a21 b12 + a22 b22 + . . . + a2n bn2 ) + . . . . . . + (an1 b1n + an2 b2n + . . . + ann bnn ) = = (b11 a11 + b21 a12 + . . . + bn1 a1n ) + (b12 a21 + b22 a22 + . . . + bn2 a2n ) + . . . . . . + (b1n an1 + b2n an2 + . . . + bnn anb ) =
n n n

=
k =1

(bk1 a1k + bk2 a2k + . . . + bkn ank ) =


k =1 i=1

bki aik

= T r (BA)

v) Temos que A = (aij )nn , ento AT = (aji )nn , assim, a linhai da matriz A ser igual a coli da matriz AT , ento,
n n n

T r (AT A) =
j =1 i=1

aji aij

=
j =1

(aj 1 a1j + aj 2 a2j + . . . + ajn anj ) =

= (a11 a11 + a21 a12 + . . . + an1 a1n ) + (a12 a21 + a22 a22 + . . . + an2 a2n ) + . . . . . . + (a1n an1 + a2n an2 + . . . + ann ann ) =
2 2 2 2 2 2 2 2 = (a2 11 + a21 + . . . + an1 ) + (a12 + a22 + . . . + an2 ) + . . . + (a1n + a2n + . . . + ann ) = n n n

=
j =1

(a2 j1

a2 j2

+ ... +

a2 jn )

=
j =1 i=1

a2 ji

0.

..............................................................................................

1 Matrizes

14

Exerccios
1) Construa as seguintes matrizes: 1, a) A = [aij ]33 tal que aij = 0, 1, b) B = [bij ]33 tal que bij = 0, 1, c) C = [cij ]32 tal que cij = i2 , 2) Seja A= 1 2 3 , 2 1 4

se i = j se i = j se i + j = 4 se i + j = 4 se i = j se i = j 0 1 , 2 3 1 3 C = 4 1 5 , 2 1 3 F = 4 5 2 3

D= Calcule se possvel: a) C + E b) 2C 3E c) 2B + F d) 3D + 2F e) AT

3 2 , 2 4

1 B = 2 3 1 E = 0 3

4 5 1 4 , 2 1

f) (C + E )T g) D DT h) 2AT + B i) (3D 2F )T j) (3B T 2A)T

k) (3AT 5B T )T l) (A)T m) (AT ) n) (C + E + F T )T o) (A + B T (AT B )T )T

3) Mostre que a soma e a diferena de duas matrizes triangulares superiores (inferiores) so uma matriz triangular superior (inferior). 4) Mostre que se A uma matriz triangular superior (inferior) ento AT triangular inferior (superior). 1 1 0 6 e , calcule: 5 7 9 3 1 1 a) 2A b) B c) (A + B ) 3 2 x 1 2 x 6 6) Seja A = e B = y . Se AB = encontre x e y . 3 1 2 8 1 5) Sendo dadas A = ..............................................................................................

1 Matrizes

15 1 4 , 5 2 3 1 C = 3 4 5 , 1 1 2 F = 2 3 4 1

7) Sejam A= 1 2 3 , 4 0 2 2 3 , 1 2

D= Calcule se possvel: a) AB b) CB + D c) AB + DF d) BA + F D 8) Sejam A =

3 B= 2 1 1 E = 2 3

6 3 1 5 , 4 2

e) A(BD) f) (AB )D g) A(C + E ) h) AC + AE

i) (D + F )A j) AE T k) (AB ) (DF ) l) (CAT ) (DB )T

1 2 2 1 eB = . Mostre que AB = BA. 3 2 3 4

9) Encontre um valor para r e uma valor para s de modo que AB T = 0, onde: A= 1 r 1 e B = 2 2 s

10) Indique se a armao dada verdadeira (V) ou falsa (F). Justique sua resposta. i) Se AB e BA so ambas denidas, ento A e B so ambas matrizes quadradas. ii) Se AB + BA est denida, ento A e B so matrizes quadradas de mesmo tamanho. iii) Se B tem uma coluna de zeros e o produto AB est denido, ento AB tambm tem uma coluna de zeros. iv) Se B tem uma coluna de zeros e o produto BA est denido, ento BA tambm tem uma coluna de zeros. v) Se as expresses AT A e T r (AAT ) esto denidas para qualquer matriz A. vi) Se u e v so vetores linha, ento uT v = u v . 11) Prove que se A e B so matrizes comutveis, ento valem as igualdades a seguir: a) (A + B )(A B ) = A2 B 2

b) (A + B )2 = A2 + 2AB + B 2 c) (A B )2 = A2 2AB + B 2

..............................................................................................

1 Matrizes

16 1 x 5 A = 2 7 4 y z 3

12) Determine x, y, z para que a matriz

seja simtrica.

13) Uma matriz A = [aij ] dita anti-simtrica se A = AT . Determine x, y, z de modo que a matriz 0 4 2 A = x y 1 z y 2z 0

seja anti-simtrica. 14) Prove que, se A e B so matrizes simtricas de ordem n, ento A + B tambm simtrica. 15) (Custo de Produo) Um fabricante de mveis faz cadeiras e mesas, cada uma das quais passa por um processo de montagem e outro de acabamento. O tempo necessrio para esse processo dado (em horas) pela matriz
g em nta Mo ab a pin Ca m as Ac me nto

A=

2 3

2 Cadeira 4 Mesa

O fabricante tem uma fbrica em So Paulo e outra em Campinas. As taxas por hora para cada um dos processos em cada fbrica so dadas (em Reais) pela matriz
Pau lo So

A=

9 10

10 Montagem 12 Acabamento

Qual o signicado dos elementos do produto matricial AB ? 16) Suponha que A = 0 e AB = AC onde A, B, C so matrizes tais que a multiplicao esteja denida. a) B = C ? b) Se existir uma matriz Y tal que Y A = I , onde I a matriz identidade, ento B = C ? 17) Dada uma matriz A denimos sua k-sima potncia por Ak = A A A, k N. Pergunta: se k > 1, qual deve ser a ordem da matriz A para que Ak esteja denida? 18) Seja A uma matriz quadrada qualquer. Mostre que A + AT simtrica e A AT anti-simtrica. Escreva A como a soma de duas matrizes, sendo uma simtrica e a outra anti-simtrica. ..............................................................................................

1 Matrizes

17

19) Um construtor tem contratos para construir trs estilos de casa:


ei ra Vi dr Ti jo lo Fe r o Ti nt a ro ad M

5 Mediterrneo 7 Colonial 6
Moderno

20 18 25

16 12 8

7 9 5

17 21 13

a) Se ele vai construir 5,7 e 12 casas do tipo moderno, mediterrneo e colonial, respectivamente, quantas unidades de cada material sero empregadas? b) Suponha agora que os preos por unidade de ferro, madeira, vidro, tinta e tijolo sejam, respectivamente, 15, 8 ,5, 1 e 10. Qual o preo unitrio de cada tipo de casa? c) Qual o custo total do material empregado? 20) Sejam A e B matrizes diagonais n n. AB = BA? Justique sua resposta. 21) Sejam p e q inteiros no-negativos e seja A uma matriz quadrada. Mostre que a) Ap Aq = Ap+q b) (Ap )q = Apq 22) Se AB = BA e p um inteiro no-negativo, mostre que (AB )p = Ap B p 23) Seja A uma matriz invertvel de ordem n e sejam u e v vetores-coluna de Rn , isto , u, v Rn1 . Mostre que A + uv T invertvel e (A + uv T )1 = A1 + A1 uv T A1 1 + v T A1 u (Frmula de Sherman-Morrinson)

24) Se A, B e C so matrizes n n invertveis, ento mostre que (ABC )1 = C 1 A1 B 1 25) Uma matriz A dita idempotente se A2 = A. a) Mostre que, se A idempotente, ento I A tambm . b) Mostre que, se A idempotente, ento 2A I invertvel e tambm sua prpria inversa.

..............................................................................................

2
Sistemas Lineares
Denio 2.1: Chamamos de equao linear, uma equao da forma a1 x1 + a2 x2 + . . . + an xn = b (n N) (2.1) que expressa b R em termos das variveis x1 , x2 , . . . , xn R e das constantes a1 , a2 , . . . , an R que so dadas. As variveis x1 , x2 , . . . , xn tambm so chamadas de incgnitas e, b dito termo independente da equao. O uso da palavra linear se deve ao fato de que uma equao linear representa gracamente uma linha reta.
E XEMPLO 5: 1 Considere a equao linear x y = 1, gracamente temos: 3 y

Denio 2.2: Denominamos de equao linear homognea, quando a equao (2.1) toma a forma a1 x1 + a2 x2 + . . . + an xn = 0, ou seja, o termo independente sempre nulo (b = 0); caso contrrio, chamamos de equao linear no homognea.

2 Sistemas Lineares

19

Denio 2.3: Chamamos de sistema de equaes lineares ou simplesmente de sistema linear, uma coleo de m (m N) equaes lineares, onde cada equao possui n (n N) incgnitas. De modo geral, denotamos um sistema linear da seguinte forma: a11 x1 a21 x1 . . . + + a12 x2 a22 x2 . . . + ... + + ... + .. . a1n xn a2n xn . . . = = b1 b2 . . .

am1 x1 + am2 x2 + . . . + amn xn

= = bm

(2.2)

Podemos reescrever o sistema linear acima, de uma forma composta, como:


n

Sn =
j =1

aij xj = bi

(i = 1, 2, . . . , m).

(2.3)

Os coecientes i e j nos coecientes das incgnitas aij , com i = 1, 2, . . . , m e j = 1, 2, . . . , n, indicam a sua posio no sistema linear. O ndice i, indica a qual equao pertence, enquanto o ndice j , indica qual varivel xj que est sendo multiplicada pela constante. Denio 2.4: Uma n-upla (s1 , s2 , . . . , sn ) R de elementos, denominado conjunto-soluo do sistema linear, quando estes elementos substitudos em (x1 , x2 , . . . , xn ) tornam verdadeiras ambas as equaes. Denio 2.5: Quando b1 = b2 = . . . = bn = 0, o sistema linear em (2.2) chamado sistema linear homogneo, caso contrrio, dito sistema linear no-homogneo. Denio 2.6: Um sistema linear dito possvel e determinado, quando admite uma nica soluo. Quando o sistema linear admite mais de uma soluo, chamado indeterminado e, um sistema linear que no admite soluo dito impossvel. Observao: Todo sistema linear homogneo possui soluo, logo, o mesmo compatvel. Denio 2.7: Quando a soluo de um sistema linear homogneo for x1 = x2 = . . . = xn = 0, chamamos de soluo trivial. Todas as outras solues so ditas solues no-triviais. Um sistema linear com duas equaes e duas incgnitas x e y : a1 x + b1 y = c1 a2 x + b2 y = c2 representam duas retas em R2 e, cada soluo (x, y ) deste sistema corresponde a um ponto da interseo das retas. Dessa forma, temos trs possibilidades: ..............................................................................................

2 Sistemas Lineares

20

i) O sistema tem uma nica soluo; o caso onde as duas retas se interceptam em um nico ponto. ii) O sistema no tem soluo; o caso em que as retas so paralelas, portanto no existe interseo. iii) O sistema tem innitas solues; o caso em que as retas so coincidentes. y y y

x (i) uma nica soluo (ii) nenhuma soluo

x (iii) innitas solues

Quando o sistema linear possui trs equaes com trs incgnitas: a1 x + b1 y + c1 z = d1 a2 x + b2 y = c2 z = d2 a3 x + b3 y + c3 z = d3 cada equao representa um plano em R3 e, a soluo do sistema linear est na interseo dos trs planos. Do mesmo modo, temos alguns casos a considerar:

(i) Nenhuma soluo, trs planos paralelos sem interseo em comum.

(ii) Nenhuma soluo

..............................................................................................

2 Sistemas Lineares

21

(iii) Uma innidade de solues, os trs planos so coincidentes, logo a interseo um plano.

(iv) Nenhuma soluo

(v) Innitas solues. A interseo uma reta.

(vi) Uma nica soluo. A interseo comum um ponto

E para o caso em que o sistema linear possui equaes com mais de trs incgnitas, cada equao linear representa um plano em Rn , tambm chamado de hiperplano e, a soluo do sistema linear, tambm a interseo dos m planos em Rn . Denio 2.8: Um sistema linear que possui o nmero de equaes igual ao nmero de incgnitas, dito sistema linear quadrado ou sistema quadrado. ..............................................................................................

2 Sistemas Lineares

22

Denio 2.9: Um sistema quadrado dito triangular superior, quando os coecientes aij = 0 para i > j , com i, j = 1, 2, . . . , n. a11 x1 + a12 x2 + . . . + a1n xn a22 x2 + . . . + a2n xn . . . = b1 = b2 . . .

(2.4)

ann xn = bn Denio 2.10: Um sistema quadrado dito triangular inferior, quando os coecientes aij = 0 para i < j , com i, j = 1, 2, . . . , n. a11 x1 a21 x2 . . . = b1 = b2 . . .

+ a22 x2 . . .

(2.5)

an1 xn + an2 x2 + . . . + ann xn = bn Existem vrias tcnicas para encontrarmos a soluo de um sistema linear, a mais conhecida de todas, com certeza , o mtodo de eliminao, onde normalmente temos contato com este mtodo no perodo escolar. Outros dois mtodos que so igualmente ecientes o mtodo de reduo de GaussJordan e o mtodo de eliminao de Gauss ou eliminao gaussiana. Na prtica, estes dois mtodos so pouco utilizados, a tcnica mais usada a que envolve a fatorao LU da matriz dos coecientes.
E XEMPLO 6: Considere o sistema: 2x x + utilizando o mtodo de eliminao, fazemos: 2x x + y y = = 1 5 (I ) (II ) y y = = 1 5

isolando a varivel y na primeira equao, camos com y = 2x 1. Substituindo o valor de y na equao (II), camos com: x + (2x 1) = 5 3x = 6 x = 2 encontrado o valor de x, basta substitu-lo em qualquer uma das duas equaes, por exemplo, na equao (II), temos: 2 + y = 5 y = 3. Logo a soluo do sistema o par ordenado (2, 3), ou seja, x = 2 e y = 3.

Olhando do ponto de vista geomtrico, estamos procurando a interseo das retas r1 : 2x y = 1 e r2 : x + y = 5. Substituindo valores para x e y nas equaes, podemos representar as retas no plano cartesiano e, com isso, observar que o ponto de interseo das retas, o ponto (2, 3), o qual, a soluo encontrada para o sistema. ..............................................................................................

2 Sistemas Lineares

23 y r1 r2

(2,3)

Podemos ainda, representar o sistema linear do exemplo anterior como: x 2 1 1 +y = 1 5 1


a1 a2 b

com isso, estamos procurando uma combinao linear dos vetores a1 , a2 que resulte no vetor b. Denio 2.11: Sejam a1 , a2 , . . . , an vetores em Rn . Um vetor a Rn dito uma combinao linear de a1 , a2 , . . . , an Rn , se existe nmeros reais c1 , c2 , . . . , cn tais que a = c1 a1 + c2 a2 + . . . + cn an . (2.6)

Observao: Quando nos referimos a uma combinao linear, vamos escrever somente as siglas C.L. Voltando a C.L. do exemplo anterior, ento, estamos procurando valores para x, y R que multiplicam os vetores a1 , a2 R2 , respectivamente, de modo que, xa1 + ya2 = ya2 + xa1 = (1, 5), onde (1, 5) o vetor b R2 . Esta idia ilustrada na gura a seguir, onde 2a1 somado com 3a2 . Geometricamente, isto conhecido como, a regra do paralelogramo. y (1,5)

x
E XEMPLO 7: Considere o sistema linear: 2x + 4x 2x + y 6y 7y + z = = = 5 2 9

+ 2z

..............................................................................................

2 Sistemas Lineares reescrevendo este sistema, temos: 1 1 5 2 x 4 +y 6 +z 0 = 2 2 7 2 9


a1 a2 a3 b

24

logo, temos que encontrar um C.L. dos vetores a1 , a2 , a3 R3 com as incgnitas x, y, z R, de modo que, o resultado seja b R3 . A soluo desta C.L. fcil de encontrar, que o ponto (1, 1, 2), tambm podemos encontrar este resultado aplicando o mtodo de eliminao.

Vamos pensar agora, no sistema linear para o caso geral: a11 x1 a21 x1 . . . + + a12 x2 a22 x2 . . . + ... + + ... + .. . a1n xn a2n xn . . . = = b1 b2 . . .

am1 x1 + am2 x2 + . . . + amn xn

= = bm

(2.7)

reescrevendo este sistema, estamos procurando o ponto na interseo dos m planos em Rn . Dessa forma temos a11 a12 a1n b1 a21 a a n b2 +x2 22 + . . . + xn 2 = . (2.8) x1 . . . . . . . . . . . am1
A1

am2
A2

amn
An

bm
b

e, ao resolver (2.8), estamos procurando coecientes da C.L. de A1 , A2 , . . . , An Rm que resulte em b Rm . x1 x2 o vetor cujos elementos formam esta C.L. que resulta em b, ento, Observe que, se x = . . . xn x est na interseo dos m planos em Rn . A, onde | | A = A1 A2 | | Note que, podemos representar os vetores-coluna dos coecientes da C.L. acima em uma matriz a11 a12 . . . a1n | a21 a22 . . . a2n . . . . An = . . . .. . . . . . . . | am1 am2 . . . amn

Lembrando da multiplicao de matrizes, que denimos no captulo anterior, se zermos a matriz A multiplicada pelo vetor x, temos: Ax = x1 A1 + x2 A2 + . . . + xn An = b ..............................................................................................

2 Sistemas Lineares

25

logo, podemos representar o sistema linear em (2.7) como uma equao matricial da forma Ax = b onde, a11 a21 A= . . . a12 a22 . . . ... ... .. . a1n a2n Rmn a matriz dos coecientes; . . . (2.9)

am1 am2 . . . amn

x1 x2 Rn o vetor das incgnitas; x= . . . xn

b1 b2 Rm o vetor dos termos independentes. b= . . . bm Juntando-se a matriz A dos coecientes o vetor b dos termos independentes, obtemos a11 a21 A= . . . a12 a22 . . . ... ... .. . a1n a2n . . . b1 b2 . . .

(2.10)

am1 am2 . . . amn

bm

chamada de matriz aumentada do sistema linear (2.7). A matriz aumentada de (2.7) ser denotada por [A|b]. Denio 2.12: Dizemos que um sistema Ax = b singular quando no tem soluo ou a soluo nica, caso contrrio dito no-singular. No que segue, vamos trabalhar com sistemas lineares em sua forma matricial e, vamos apresentar o mtodo de resoluo de sistemas lineares conhecido como mtodo de Gauss ou eliminao gaussiana. Observao: Os mtodos de resoluo de sistemas lineares esto divididos em duas classes: mtodos diretos; mtodos iterativos. ..............................................................................................

2 Sistemas Lineares

26

Ns iremos tratar apenas dos mtodos diretos, que so os que determinam a soluo de um sistema linear atravs de um nmero nito de passos. Os mtodos iterativos, que so vistos em um curso de Clculo Numrico ou um curso avanado de lgebra Linear, requerem um nmero innito de passos. Denio 2.13: Dizemos que dois sistemas lineares so equivalentes, se o conjunto soluo de um deles tambm for soluo do outro. Portanto, j que sistemas lineares equivalentes tem a mesma soluo, o que vamos fazer para resolver um sistema linear, substituir o sistema original por outro equivalente. Uma maneira de encontrarmos um sistema linear equivalente aplicando sucessivamente uma srie de operaes, que no alteram a soluo do sistema. Estas operaes so chamadas de operaes elementares, denidas a seguir: Denio 2.14: Existem trs operaes elementares sobre as linhas de uma matriz: i) Trocar a posio de duas linhas da matriz, isto , troca-se ar1 , ar2 , . . . , arn por as1 , as2 , . . . , asn e as1 , as2 , . . . , asn por ar1 , ar2 , . . . , arn . Vamos denotar a troca de linhas por: lr ls , com r, s = 1, 2, . . . , m. ii) Multiplicar uma linha da matriz por um escalar k no nulo (k R {0}). Denotaremos por: lr klr , com r = 1, 2, . . . , m. iii) Substituir a r -sima linha da matriz pela r -sima linha adicionada de k vezes a s-sima linha da matriz, com k R {0}, r, s = 1, 2, . . . , m e r = s; em outras palavras, troca-se ar1 , ar2 , . . . , arn por ar1 + kas1 , ar2 + kas2 , . . . , arn + kasn . Denotaremos por: lr lr + kls (lr recebe lr + kls ). Observao: Note que, ao aplicarmos operaes elementares em um sistema linear, devemos considerar a matriz aumentada [A|b] para que o conjunto soluo no seja alterado. O mtodo de eliminao gaussiana consiste em aplicar operaes elementares s linhas da matriz aumentada at transformarmos o sistema linear em um sistema triangular superior, e ento aplicarmos uma substituio regressiva para encontrarmos o conjunto soluo. Observao: Por um momento, vamos nos limitar apenas a resoluo de sistemas quadrado, mais adiante falaremos um pouco sobre a resoluo de sistemas no quadrado. Vamos mostrar agora, atravs de um exemplo, o mtodo de eliminao gaussiana.
E XEMPLO 8: Considere o seguinte sistema 2x1 4x1 2x1 + 3x2 + 4x2 3x2 x3 3x3 + x3 = = = 5 3 1

..............................................................................................

2 Sistemas Lineares escrevendo a matriz aumentada do sistema, temos: 2 3 1 4 4 3 2 3 1

27

portanto, para deixarmos este sistema na forma de um sistema triangular superior, devemos aplicar operaes elementares de tal forma, que os elementos a21 , a31 e a32 da matriz aumentada sejam nulos; para isso, vamos pegar a linha 1 (l1 ) da matriz aumentada e aplicar a operao elementar (iii). Dessa forma, o elemento a11 ser o piv da coluna 1 e vamos calcular os multiplicadores para os elementos a21 e a31 : m21 = a21 4 a31 2 = = 2 e m31 = = = 1 a11 2 a11 2

5 3 1

Aplicando agora as operaes elementares, temos: 2 3 1 l1 l1 5 7 l2 l2 + (2)l1 0 2 1 0 6 2 6 l3 +(1)l1

observe que, todos os elementos abaixo do piv da primeira coluna j foram anulados. Vamos repetir o processo agora para a coluna 2, tomando como piv o elemento a22 e calculando o multiplicador para o elemento a32 : a32 6 m32 = = = 3 a22 2 aplicando novamente operaes elementares, temos: 2 3 1 5 l1 l1 0 2 1 7 l2 l2 0 0 5 15 l3 +(3)l2 2x1 + 3x2 2x2 x3 x3 5x3 = = = 5 7 15

Portanto, j encontramos nosso sistema triangular superior, o qual equivalente ao sistema inicial:

aplicando agora uma substituio regressiva, temos 2x1 + 3x2 2x2 2x1 + 3x2 2x2

x3 x3 5x3 x3 x3 x3 x3 x3

= = = = = =

5 7 15 5 7 3

2x1

+ 3x2 x2

= 5 = 2 = 3

..............................................................................................

2 Sistemas Lineares x1 1 isto , nosso conjunto soluo x = 2. 3 = 1 = 2 = 3

28

x2 x3

Vamos formalizar agora, o mtodo de eliminao gaussiana: i) Para aplicarmos a eliminao gaussiana, precisamos que todas as linhas nulas do sistema ocorram abaixo de todas as linhas no-nulas. ii) Usaremos mltiplos para a primeira equao de modo que, atravs de uma operao elementar todos os elementos abaixo do primeiro piv sejam zero. iii) Aps feito o segundo passo, ca fcil encontrar o segundo piv, e ento, aplicamos novamente uma operao elementar, de modo que todos os elementos abaixo do segundo piv sejam nulos. iv) Aplicando sucessivamente o passo ii) at a n-sima equao, encontraremos o ltimo piv, o qual estar multiplicando a ltima varivel xn . v) Encontrado o sistema triangular superior, que equivalente ao sistema inicial, basta aplicarmos ento, uma substituio regressiva, isto , comeando pela ltima varivel at chegarmos a primeira, encontrando assim o conjunto soluo do sistema. Observao: Por denio, os pivs nunca podem ser zeros. Note que: i) Se, ao aplicarmos a eliminao gaussiana, ocorrer uma equao do tipo 0x1 + 0x2 + . . . + 0xn = 0 esta linha dever ser deslocada para o m do sistema, ou ento, poder ser suprimida do sistema. ii) Se encontrarmos uma equao do tipo 0x1 + 0x2 + . . . + 0xn = b (b = 0)

aps aplicarmos a eliminao gaussiana, ca evidente que o sistema impossvel. Denio 2.15: Dado um sistema linear com n equaes e n incgnitas (sistema quadrado), dizemos que o sistema tem um conjunto completo de pivs se tem n pivs no-nulos. Observao: Um sistema no-singular se, e somente se, ele tem um conjunto completo de pivs. ..............................................................................................

2 Sistemas Lineares

29

No que segue, vamos discutir uma variao do mtodo de eliminao gaussiana para encontrarmos a soluo de um sistema linear. Este mtodo consiste em decompor uma matriz como um produto de duas matrizes, sendo uma das matrizes triangular inferior e a outra, triangular superior. Esta decomposio nos leva ao algoritmo mais utilizado para a resoluo de sistemas lineares da forma Ax = b usando computadores, pois este mtodo fornece a maneira mais barata de resolver um sistema linear onde necessitamos mudar, repetidamente, o lado direito. Nota: Para este tipo de decomposio, vamos modicar o mtodo de eliminao gaussiana: i) No ser permitida a troca de linhas; ii) A nica operao elementar permitida ser a (iii), isto , somar o mltiplo de uma linha a uma outra linha. Observao: Devido restrio i) descrita acima, se em algum momento algum dos pivs for zero, ento a decomposio no funciona. Para este caso, apresentaremos uma outra tcnica de decomposio, onde ser permitida a troca de linhas. Para o mtodo que utilizaremos agora, no vamos mais usar a matriz aumentada do sistema para aplicar a eliminao gaussiana, utilizaremos apenas a matriz dos coecientes. Denio 2.16: Suponha que a matriz Ann possa ser escrita como a multiplicao de uma matriz triangular inferior denotada por L (abreviatura de lower triangular, do ingls), por uma matriz triangular superior, denotada por U (abreviatura de upper triangular, do ingls), isto , A = LU Neste caso, dizemos que A tem uma fatorao LU ou, uma decomposio LU da matriz. A matriz U j sabemos como encontrar, isto , a matriz resultante da eliminao gaussiana, ou seja, uma matriz triangular superior, onde todos os elementos abaixo da diagonal so zeros e nenhum elemento da diagonal nulo. A matriz L, como j havamos mencionado, ser uma matriz triangular inferior, onde todos os elementos da diagonal so 1s e os elementos que aparecem abaixo da diagonal so exatamente os multiplicadores, com os sinais trocados, utilizados em cada eliminao feita no momento de encontrarmos a matriz U .
E XEMPLO 9: Considere o sistema linear: 6x1 3x1 12x1 6x1 2x2 3x2 + 8x2 4x3 6x3 + 21x3 10x3 + + + 4x4 x4 8x4 7x4 = 2 = 4 = 8 = 43

(2.11)

..............................................................................................

2 Sistemas Lineares escrevendo a matriz dos coecientes, temos: 6 2 4 4 3 3 6 1 A= 12 8 21 8 6 0 10 7

30

para encontrarmos a decomposio LU da matriz, vamos utilizar o mtodo de eliminao gaussiana, com as restries dadas, para encontrar a matriz U . Ao mesmo tempo vamos construir a matriz L, triangular inferior com 1s na diagonal, que carrega os multiplicadores, com os sinais trocados, das operaes elementares aplicadas na matriz A para encontrarmos a U . 1o Passo: O primeiro piv o elemento a11 = 6. Vamos calcular os multiplicadores dos elementos a21 , a31 e a41 . a21 3 1 m21 = = = a11 6 2 a31 12 m31 = = =2 a11 6 a41 6 m41 = = =1 a11 6 Agora vamos aplicar as seguintes operaes elementares para encontrarmos a matriz U1 . 6 0 U1 = 0 0 l1 l1 2 4 4 1 2 4 1 l2 l2 + ( )l1 2 4 13 0 l3 l3 + 2 l1 2 14 11 l4 l4 + 1 l1

Feito o primeiro passo da eliminao gaussiana, vamos comear a construir a matriz L 1 1 L1 = 2 2 1 0 1 0 0 . 1 0 1 0 0

2o Passo: Olhando para a matriz U1 , temos que o segundo piv o elemento a22 = 2. Vamos calcular ento, os multiplicadores para os elementos a32 e a42 . m32 = m42 = 6 2 0 2 U2 = 0 0 0 0 a32 4 = =2 a22 2

Aplicando as seguintes operaes elementares, encontramos a matriz U2 . 4 4 l1 l1 4 1 l2 l2 5 2 l3 l3 + 2l2 10 12 l4 l4 + (1)l2

a42 2 = = 1 a22 2

..............................................................................................

2 Sistemas Lineares

31

aps o segundo passo da eliminao gaussiana, continuamos a construo da nossa matriz L: 1 0 1 1 L2 = 2 2 2 1 1 0 0 1 0 0 . 0 1

3o Passo: Da matriz U2 , temos o terceiro piv, que o elemento a33 = 5 e calculamos ento, o multiplicador para o elemento a43 . a43 10 m43 = = =2 a33 5 fazendo a ltima operao elementar, temos a matriz U3 . 6 0 U3 = 0 0 2 4 4 2 4 1 0 5 2 0 0 8 l4 l1 l1 l2 l2 l3 l3 l4 + 2 l3 0 0 0 1

aps o ltimo passo da eliminao gaussiana, temos a nossa matriz L: 1 0 0 1 1 0 L3 = 2 2 2 1 1 1 2

Chamando L3 de L e U3 de U , temos a decomposio da matriz original A na forma LU , isto , A = LU (verique!). Agora, vamos encontrar a soluo do sistema linear usando a fatorao LU . Substituindo A por LU no sistema linear Ax = b, temos (LU )x = b = L(U x) = b. Fazendo U x = z , temos a seguinte equao matricial Lz = b isto , 0 z1 2 z2 4 0 = z 8 3 0 43 1 z4 = + + z2 2z2 z2 + z3 2z3 + z4 = 2 4

resolvendo essa equao matricial, temos: z1 1 z1 2 2z1 z1

1 0 0 1 1 0 2 2 2 1 1 1 2

= 8 = 43

..............................................................................................

2 Sistemas Lineares fazendo agora uma substituio progressiva, isto , de cima para baixo, temos: z1 z2 z3 z4 Agora, vamos resolver U x = z 6 0 0 0 6x1 2 4 4 x1 2 2 4 1 x2 5 = 0 5 2 x3 2 0 0 8 x4 32 4x3 4x3 5x3 + 4x4 x4 2x4 8x4 = = = = 2 5 2 32 = 2 = 5 = 2 = 32

32

da, temos:

2x2 2x2

aplicando uma substituio regressiva, temos x1 x2 x3 x4


9 2 69 10 6 . 5

Logo, a soluo do sistema linear dado , x =

= 9 2 = 69 10 6 = 5 = 4

Observao: No exemplo acima, foi imposta a construo da matriz L do ponto de vista prtico e usual, mas o modo como realmente construda a L, pode ser encontrado no apndice B. Resumindo: Considere o sistema linear Ax = b e suponha que A tenha uma fatorao LU , isto , A = LU onde, L triangular inferior com 1s na diagonal e, U triangular superior, ento, substituindo A = LU no sistema Ax = b, temos LU x = b fazendo U x = z , temos Lz = b e aplicando um substituio progressiva encontramos z e, da, aplicando uma substituio regressiva em U x = z , encontramos x que a soluo do sistema. ..............................................................................................

2 Sistemas Lineares E XEMPLO 10: Considere o sistema x1 x1 + x2 2x2 x2 + x3 2x3 x3 = = = = 1 1 1 1

33

x4 + 2x4

temos,

decompondo A em LU ,

1 1 0 0 x1 1 1 2 1 0 x 1 2 A= , x = , b = 0 1 2 1 x3 1 0 0 1 2 x4 1 0 0 , 0 1 1 0 0 0 1 1 0 U = 0 0 1 1 0 0 0 1

assim, de Lz = b, temos

1 0 0 1 1 0 L= 0 1 1 0 0 1 1 0 0 1 1 0 0 1 1 0 0 1 1 0 0 0

e, fazendo U x = z , temos

1 1 0 z1 0 z2 1 2 = = z = 3 0 z3 1 1 4 1 z4

10 1 x1 1 0 0 1 1 0 x2 2 9 = = x = 7 0 1 1 x3 3 4 4 x4 0 0 1

Observao: Muitos podem estar pensando que este mtodo d mais trabalho do que a eliminao gaussiana aplicada diretamente no sistema linear Ax = b e ento fazendo uma nica substituio regressiva. Pois bem, lembre que o vetor b o vetor das variveis independentes, logo, ele quem determina se o sistema tem soluo ou no, ento para cada mudana de valor efetuada em qualquer um dos elementos do vetor b, teremos que aplicar novamente a eliminao gaussiana em todo sistema linear Ax = b e aplicar novamente uma substituio regressiva para encontrarmos a soluo. Observe que, na fatorao LU , o nico trabalho que teremos a cada modicao do vetor b, aplicar uma substituio progressiva e uma substituio regressiva e pronto (Por qu?). Isto segue diretamente do fato, que a matriz A dos coecientes no alterada, logo L e U tambm no, portanto o nico trabalho que temos a fazer a cada mudana de b , calcular, Lz = b e depois U x = z. Observe que, a forma LU assimtrica, isto porque a matriz U contm os pivs ao longo da diagonal enquanto a matriz L tem sempre 1s. Com isso, podemos dividir a matriz U em duas matrizes, sendo uma delas diagonal e a outra triangular superior com 1s na diagonal. ..............................................................................................

2 Sistemas Lineares

34

Assim, escrevemos a matriz U como u11 u12 . . . u1n u11 0 . . . 0 1 0 u22 . . . u2n 0 u22 . . . 0 0 = . . U = . . . . .. .. . . . . . . . . . . . . . . . . . 0 0 . . . unn 0 0 . . . unn 0
D =diag (u11 ,...,unn )

u12 u11

1 . . . 0

... ... .. . ...


U

u1n u11 u2n u22

. . . 1

isto , a matriz D, matriz que carrega os pivs ao longo da diagonal e o restante dos elementos so nulos, e a matriz U encontrada dividindo todos os elementos de cada uma das linhas pelo piv correspondente da linha. Dessa forma, temos a matriz D, diagonal com os pivs na diagonal, e U triangular superior com 1s na diagonal, de modo que U = DU . Fazendo um abuso de notao, vamos reescrever U como U e, assim obtemos que A = LDU (2.12) tambm conhecido como, decomposio LDU ou fatorao LDU.
E XEMPLO 11: Considere a matriz

Fazendo a fatorao LU , temos: 1 0 L = 0 1 3 0

2 3 A = 0 5 6 9 0 0 1

3 7 8 2 U = 0 0
3 2

agora, vamos decompor a matriz U , como: 2 U = 0 0

3 3 5 7 0 1
3 2 7 5

1 0 0 5 0 0 0 1 0
D

1 0
U

reescrevendo U como U , temos A = LDU , isto , 1 0 0 2 0 0 1 A = 0 1 0 0 5 0 0 3 0 1 0 0 1 0

3 2

1 0

3 2 7 5

Observao: A ordem em que aparece a multiplicao das matrizes na decomposio LDU no deve ser alterada. Teorema 2.17 (Unicidade da LU): Se A = L1 D1 U1 e A = L2 D2 U2 onde L1 , L2 so triangulares inferiores com 1s na diagonal, U1 , U2 so triangulares superiores com 1s na diagonal e, D1 , D2 matrizes diagonais com os elementos da diagonal no nulos, ento L1 = L2 , U1 = U2 e D1 = D2 . ..............................................................................................

2 Sistemas Lineares

35

Conseqentemente, a fatorao LDU e LU so unicamente determinadas, quando a eliminao pode ser completada sem a troca de linhas. Nota: Seja A uma matriz simtrica. Considere A = LDU a fatorao LDU de A. Da, AT = (LDU )T = U T DT LT como A simtrica, A AT assim, temos que U T , L : triangular inferior com 1s na diagonal LT , U : triangular superior com 1s na diagonal DT , D : diagonal que contm os pivs Supondo que A no-singular, e portanto tem um conjunto completo de pivs, segue da unicidade da fatorao LDU que U = LT conseqentemente, nossa matriz A pode ser fatorada como A = LDLT onde L diagonal inferior com 1s na diagonal e D diagonal que contm os pivs.
E XEMPLO 12: 1 2 Seja A = . 2 8 Aplicando a fatorao LDU , temos as matrizes L= logo 1 2 1 = 2 8 2 0 1 1 0 0 4 1 2 . 0 1 1 0 2 1 e D= 1 0 0 4

= LDU T = U D T LT

(2.13)

A seguir, vamos tratar do caso, em que a fatorao LU no pode ser encontrado sem que haja uma troca de linhas. Para isto, no vamos mais utilizar a igualdade A = LU e sim, P A = LU , onde P uma matriz de permutao. Denio 2.18 (Matriz de Permutao): Uma matriz Pnn denominada matriz de permutao, quando esta resulta da reordenao, de alguma forma, das linhas da matriz identidade In . Proposio 2.19: Toda matriz de permutao P invertvel, e sua inversa P 1 a prpria matriz P . ..............................................................................................

2 Sistemas Lineares

36

A demonstrao segue diretamente do fato que P obtida atravs de um produto de matrizes elementares (ver apndice B).
E XEMPLO 13: Seja a matriz

que troca a linha i com a linha j da matriz identidade. Se a11 . . . aj 1 A= a i1 . . . an1 a11 . . . ai1 Pij A = a j1 . . . a1m . . . . . . ajm j . . . aim i . . . . . . anm ... ...

0 . . . Pij =

0 ..

... . 1 0 1 1 0 1 .. ... . 0

j i . . . 0 1

ento

an1

troca a linha i com a linha j na matriz A.

a1m . . . . . . aim i . . . ajm j . . . . . . anm

Note que, os sistemas Ax = b e P Ax = P b tm as mesmas solues, e com isso, podemos resolver o sistema pela decomposio LU . Como P possui inversa P 1 , em alguns livros a equao P A = LU pode escrita como A = P LU (2.14)

conhecido tambm como decomposio PLU da matriz A, onde a P trocada pela P 1 . Logo, isto nos sugere a seguinte denio: Denio 2.20: Para um sistema Ax = b (A Rnn ),

..............................................................................................

2 Sistemas Lineares

37

se conhecemos as linhas que preciso trocar em A de maneira que no aparea nenhum zero na posio dos pivs, ento podemos construir a matriz de permutao P de modo que, podemos obter a fatorao LU de P A. Assim temos, P A = LU (2.15)
E XEMPLO 14: Seja

vamos aplicar a eliminao gaussiana na matriz A e efetuar as trocas de linhas necessrias para encontrarmos a matriz U . Ento, 1 1 1 l2 l2 + (1)l1 A = 1 1 3 2 5 8 l3 l3 + (2)l1 resulta em 1 1 U1 = 0 0 0 3 1 2 6

1 1 A = 1 1 2 5

1 3 8

observe que na posio do segundo piv apareceu o troca de linhas, l2 l3 e assim, obtemos: 1 1 U = 0 3 0 0

elemento nulo, logo vamos efetuar a seguinte 1 6 2

Note que, foi preciso efetuar uma permutao de linhas no processo da eliminao gaussiana, ento, efetuando essa mesma troca de linhas na matriz identidade I3 , obtemos a nossa matriz de permutao 1 0 0 P = 0 0 1 0 1 0

agora nos resta construir a matriz L que carrega os multiplicadores, mas preciso car atento em relao a troca de linhas no processo da eliminao gaussiana, assim, para cada troca de linhas efetuadas no procedimento para obter a matriz U quando construirmos a matriz L, precisamos efetuar as mesmas trocas de posies somente para os multiplicadores que foram obtidos antes da primeira troca de linhas (lembre-se, somente para os multiplicadores no para a linha toda), logo, a matriz L correspondente 1 0 0 L = 2 1 0 1 0 1 observe que o multiplicador que foi usado para a linha 3 aparece na linha 2 na matriz L, e o multiplicador da linha 1 aparece na linha 3. Agora podemos representar A como P A = LU .

..............................................................................................

2 Sistemas Lineares E XEMPLO 15: Encontre a fatorao P A = LU e resolva o sistema linear Ax = b usando P e os fatores L e U para 0 1 1 3 1 1 1 1 2 2 A= , b = 0 1 1 1 1 0 0 1 1 1

38

Na posio do primeiro piv j apareceu o zero, logo vamos fazer a seguinte troca de linha l1 l2 1 1 1 0 1 1 U1 = 0 1 1 0 0 1 2 3 1 1

apareceu o zero na posio do terceiro piv, ento vamos fazer a seguinte troca de linha l3 l4 e com isso, vamos encontrar a matriz U nal. 1 1 1 2 0 1 1 3 U = . 0 0 1 1 0 0 0 4 Vamos construir a matriz P , 1 0 0 0 1 0 P = 0 0 1 0 0 0 0 0 l1 l2 = 0 1 0 1 0 0 0 0 l3 l4 = 0 1 0 1 0 0 0 0 1 0 0 0 0 1

aplicando a seguinte operao elementar l3 l3 + l2 , obtemos 1 1 1 2 0 1 1 3 U2 = 0 0 0 4 0 0 1 1

1 0 0 0

0 0 1 0

1 0 0 0

0 0 0 1

por ltimo, vamos construir a matriz L tomando os cuidados necessrios, 1 0 0 0 1 0 0 0 1 0 0 1 0 0 0 1 0 0 0 1 l3 l4 L= = = 0 1 0 0 1 1 0 0 0 0 1 0 0 1 0 1

0 0 1 0

portanto, j podemos reescrever a matriz A como P A = LU e vamos ento resolver o sistema linear da forma P Ax = P b.

Substituindo P A por LU , obtemos LU x = P b e aplicando agora os mesmos passos da fatorao LU , temos Lz = P b

..............................................................................................

2 Sistemas Lineares 1 0 0 0 0 1 0 1 0 0 1 0 0 z1 0 1 z 1 0 0 2 = 0 z3 0 0 1 z4 0 0 2 1 z= 1 0 e por m, fazemos Ux = z 1 1 1 2 x1 2 0 1 1 3 x 1 2 = 0 0 1 1 x3 1 0 0 0 4 x4 0 1 0 x= 1 0 0 0 0 1 0 1 2 0 1 1 0 1

39

fazendo os clculos necessrios, encontramos que

e assim, encontramos a soluo do sistema, que

..............................................................................................

3
Espaos Vetoriais
Denio 3.1: Um conjunto V chamado espao vetorial quando, V no-vazio e V munido de duas operaes: adio: :V V V (u, v ) u + v que a cada par de vetores u, v V faz corresponder um novo vetor u + v V , chamado a soma de u e v , e, multiplicao por escalar: :R V V (, v ) v que a cada R e a cada vetor v V faz corresponder um vetor v , ou v V , chamado o produto de v . Essas operaes, ainda devem satisfazer para quaisquer , R e u, v, w V , as seguintes condies, chamadas de axiomas de espao vetorial. A1) u v = v u A2) (u v ) w = u (v w) A3) 0 V , tal que u 0 = 0 v = u, u V (comutatividade). (associatividade da adio). (vetor nulo).

A4) Para cada u V , u V , chamado o inverso aditivo, ou simtrico de u, tal que u (u) = u u = 0 (inverso aditivo).

3 Espaos Vetoriais

41 (distributividade em relao a adio). (distributividade em relao a multiplicao). (associatividade da multiplicao). (existncia do elemento neutro).

D1) (u v ) = u v D2) ( + ) u = u u M1) ( ) u = ( u) M2) 1 u = u 1 = u, u V

Observao: Quando os escalares so nmeros complexos, obtm-se um espao vetorial complexo. Nos nossos estudos, vamos nos limitar apenas aos espaos vetoriais reais.
E XEMPLO 16: Considere o conjunto V de todas as triplas ordenadas de nmeros reais da forma (x, y, 0) e dena as operaes e atravs das frmulas (x1 , y1 , 0) (x2 , y2 , 0) = (x1 + x2 , y1 + y2 , 0) r (x, y, z ) = (rx, ry, 0) V com estas operaes um espao vetorial, pois satisfaz todas as propriedades da Denio 3.1. E XEMPLO 17: Considere o conjunto V das triplas ordenadas de nmeros reais munido das operaes e denidas por (x, y, z ) (x , y , z ) = (x + x , y + y , z + z ) r (x, y, z ) = (rx, y, z ). fcil vericar que as propriedades A1), A2), A3), A4) so vlidas. Neste caso, 0 = (0, 0, 0) e o negativo do vetor (x, y, z ) o vetor (x, y, z ). Como exemplo, vamos mostrar que vale a propriedade D1) e mostrar que a propriedade D2) no vlida e assim, conclumos que V no um espao vetorial quando munido das operaes descritas acima (a propriedades M1) e M2) tambm so vlidas). D1) [(x, y, z ) (x , y , z )] = (x + x , y + y , z + z ) = ((x + x ), y + y , z + z ) alm disso, (x, y, z ) (x , y , z ) = (x, y, z ) (x , y , z ) = (x + x , y + y , z + z ) = ((x + x ), y + y , z + z ) portanto, vale a propriedade D1). D2) ( + ) (x, y, z ) = (( + )x, y, z ). Por outro lado, (x, y, z ) (x, y, z ) = (x, y, z ) (x, y, z ) = (x + x, y + y, z + z ) = (( + )x, 2y, 2z ) logo, no vale a propriedade D2).

..............................................................................................

3 Espaos Vetoriais

42

E XEMPLO 18: O conjunto V = M Rmn das matrizes m n um espao vetorial com a adio e multiplicao por escalar usuais de matrizes. E XEMPLO 19: Seja V a coleo de todas as funes com valores reais denidas no intervalo [a, b]. Se f e g pertencem a V , denimos f g pela frmula (f g )(t) = f (t) + g (t) Se f est em V e um escalar, denimos f pela frmula (r f )(t) = rf (t) Ento V um espao vetorial. E XEMPLO 20: Sejam os polinmios p(t) = an tn + an1 tn1 + + a1 t + a0 e q (t) = bn tn + bn1 tn1 + + b1 t + b0 Denimos p(t) q (t) pela frmula p(t) q (t) = (an + bn )tn + (an1 + bn1 )tn1 + + (a1 + b1 )t + (a0 + b0 ) Se R, denimos c p(t) pela frmula p(t) = (an )tn + (an1 )tn1 + + (a1 )t + (a0 ) Vamos provar agora que Pn um espao vetorial. Para vericar A1), note que q (t) p(t) = (bn + an )tn + (bn1 + an1 )tn1 + + (b1 + a1 )t + (b0 + a0 ), e, como ai + bi = bi + ai , pois a, b R, podemos concluir que p(t) q (t) = q (t) p(t). A propriedade A2) pode ser mostrada de maneira anloga. O polinmio nulo o elemento 0 necessrio para a propriedade A3). Se p(t) como acima, ento seu negativo, p(t), an tn an1 tn1 + a1 t a0 Vamos mostrar agora a propriedade D2) e deixar a demonstrao das outras propriedades como exerccio. ( + ) p(t) = ( + )an tn + ( + )an1 tn1 + + ( + )a1 t + ( + )a0

= (an tn + an1 tn1 + + a1 t + a0 ) + (an tn + an1 tn1 + + a1 t + a0 ) = p(t) p(t)

= an tn + an tn + an1 tn1 + an1 tn1 + + a1 t + a1 t + a0 + a0

Observao: Para simplicar a notao, escreveremos u v como u + v e v como v . ..............................................................................................

3 Espaos Vetoriais

43

Teorema 3.2: Se V um espao vetorial, ento: i) 0u = 0, u V . ii) 0 = 0, R. iii) se u = 0, ento = 0 ou u = 0. iv) (1)u = u. Demonstrao: i) Temos 0 = 0 + 0. Da, 0u = (0 + 0)u = 0u + 0u, isto , 0u = 0u + 0u. Somando 0u e usando A4)e A3) obtemos 0 = (0u + 0u) 0u = 0u + (0u 0u) = 0u + 0 = 0u, ou seja, 0 = 0u. ii) 0 = (0 + 0) = 0 + 0, isto , 0 = 0 + 0. Somando 0 e usando A4) e A3), obtemos 0 = 0. iii) Suponhamos que u = 0, com = 0. Ento, u = 1u =
M2 M2 A3 D1 D2)

1 ii) hip 1 M1 1 u = (u) = 0 = 0.


D2 i)

iv) Note que, (1)u + u = (1)u + 1u = (1 + 1)u = 0u = 0, isto , (1)u + u = 0. Somando u e usando A4) e A3), obtemos (1)u = u. Denio 3.3: Seja V um espao vetorial munido das operaes : V V V e : RV V , e W um subconjunto de V , no-vazio. Dizemos que W V um subespao vetorial de V, se A) u, v W tem-se u v W ; M) R e u W tem-se u W . Observao: Note que W visto como um conjunto prprio um espao vetorial. Alm disso, pela propriedade M), para que W seja um subespao vetorial necessrio que 0 W . Ainda mais, todo espao vetorial V admite pelo menos dois subespaos chamados subespaos triviais, o conjunto {0} e o prprio V . ..............................................................................................

3 Espaos Vetoriais

44

E XEMPLO 21: V = R5 e W = {(0, x2 , x3 , x4 , x5 )|xi R}. Isto , W o conjunto dos vetores de R5 , cuja primeira ordenada nula. Veriquemos as condies A) e M). A) u = (0, x2 , x3 , x4 , x5 ), v = (0, y1 , y2 , y3 , y4 , y5 ) W . Ento u + v = (0, x2 + y2 , x3 + y3 , x4 + y4 , x5 + y5 ) que ainda pertence a W , pois tem a primeira coordenada nula. M) ku = (0, kx2 , kx3 , kx4 , kx5 ) W , pois a primeira coordenada nula para todo k R. Portanto, W uma subespao vetorial de R5 . E XEMPLO 22: V = R2 , onde W uma reta deste plano que no passa pela origem.

u+v /W u v

W no subespao de V , pois existem u e v em W , tal que u + v / W . Outra maneira de ver que W no subespao de V notar que o vetor nulo no pertence a W . E XEMPLO 23: V = R2 e W = {(x, x2 )|x R}. Se escolhermos u = (1, 1) e v = (2, 4), temos u + v = (3, 5) W . Assim, W no subespao vetorial de V pois, caso contrrio, a condio A) deveria ser satisfeita para quaisquer u, v W , e isto no ocorre neste exemplo.
W

(0, 0) W

E XEMPLO 24: Seja V o conjunto de todos os polinmios de grau exatamente igual a 2, ento V um subconjunto de P2 , mas no um subespao de P2 , j que a soma dos polinmios 2t2 + 3t + 1 e 2t2 + t + 2 um polinmio de grau 1, que no est em V .

Teorema 3.4 (Interseo de Subespaos): Dados W1 e W2 subespaos de um espao vetorial V , a interseo W1 W2 tambm um subespao de V . Demonstrao. Observe que, W1 W2 nunca vazio pois ambos os subespaos contm o vetor nulo ..............................................................................................

3 Espaos Vetoriais

45

de V . Logo, precisamos provar os itens A) e M) da denio anterior para mostrar que W1 W2 subespao vetorial de V . A) Dados u, v W1 W2 , ento u, v W1 e u, v W2 . Logo, u + v W1 e u + v W2 , sendo W1 e W2 subespaos de V . Portanto u + v W1 W2 . M) Seja u W1 W2 , ento u W1 e u W2 . Dado R, temos que u W1 e u W2 , pois W1 e W2 so subespaos de V . Logo, u W1 W2 . Portanto, W1 W2 um subespao vetorial de V .
E XEMPLO 25: V = Mnn

W1 = {matrizes triangulares superiores} W2 = {matrizes triangulares inferiores}

Ento, W1 W2 = {matrizes diagonais}.

Observao: A Reunio de dois subespaos vetoriais W1 W2 , nem sempre um subespao vetorial.


E XEMPLO 26: V = R3 . W2 u u+v

W1

W1 e W2 so retas que passam pela origem ento W1 W2 = {0} e W1 W2 o feixe formado pelas duas retas, que no subespao vetorial de R3 . De fato, se somarmos os dois vetores u, v W1 W2 vemos que u + v est no plano que contm W1 e W2 , mas u + v / W1 W2 .

Teorema 3.5 (Soma de Subespaos): Sejam W1 e W2 subespaos de um espao vetorial V . Ento, o conjunto W1 + W2 = {v V |v = w1 + w2 , onde w1 W1 e w2 W2 } subespao de V . Demonstrao. Devemos novamente provar os itens A) e M) da denio 3.3 para mostrar que W1 + W2 subespao vetorial de V . ..............................................................................................

3 Espaos Vetoriais

46

+ w W + W , onde w , w W e w , w W . A) Dados u = w1 + w2 W1 + W2 e v = w1 1 2 1 1 2 2 2 1 2 Temos que u + v = (w1 + w2 ) + (w1 + w2 ) = (w1 + w1 ) + (w2 + w2 ). W , ento w + w W , do mesmo modo w , w W , ento w + w W , Como w1 , w1 1 1 1 2 2 2 2 1 2 2 logo (w1 + w1 ) + (w2 + w2 ) W1 + W2 , ou seja, u + v W1 + W2 .

M) Seja u = w1 + w2 W1 + W2 , com w1 W2 e w2 W2 . Seja R, ento u = (w1 + w2 ) = w1 + w2 . como w1 W1 segue que w1 W1 , do mesmo modo w2 W2 , logo w2 W2 e da, w1 + w2 W1 + W2 , ou seja, u W1 + W2 . Portanto, W1 + W2 um subespao vetorial de V .

Observao: Quando W1 W2 = {0}, ento W1 + W2 chamado soma direta de W1 com W2 , denotado por W1 W2 . No que segue, vamos trabalhar novamente com sistemas lineares, mas dessa vez vamos considerar sistemas Ax = b com A Rmn , x Rn e b Rm , onde m < n, m > n ou m = n. Observao: O sistema Ax = b tem soluo se, e somente se, for possvel escrever b como uma C.L. das colunas da matriz A. logo, isto nos sugere a seguinte denio: Denio 3.6: O conjunto formado por todas C.L possveis das colunas da matriz A chamado de espao coluna da matriz A, e denotaremos por R(A), em smbolos, temos: R(A) = {y Rm |y = AX para algum x Rn } = {Ax|x Rn } . Note que R(A) Rm . Proposio 3.7: R(A) um subespao vetorial de de Rm . Demonstrao. A) Tome u, v R(A), onde u = Ax1 para algum x1 Rn e v = Ax2 para algum x2 Rn . Ento, u + v = Ax1 + Ax2 = A(x1 + x2 ) = Az, Portanto, u + v R(A). .............................................................................................. z = x1 + x2 .

3 Espaos Vetoriais

47

M) Dado u R(A), onde u = Ax, para algum x Rn . Tome R, ento: u = Ax = A(x) = Az, Portanto, v R(A). Logo, R(A) um subespao vetorial de Rn .
E XEMPLO 27: A= E XEMPLO 28: 1 0 A = 0 1 , 0 0 0 c c R(A) = 0 + d c, d R = d c, dR 0 0 0 1 , 1 R(A) = c c cR .

z = x.

E XEMPLO 29: Se Ann invertvel, encontre R(A).

Como A invertvel, sabemos que A no-singular, e portanto o sistema Ax = b tem soluo nica para qualquer b. Temos que R(A) Rn , ento tome b Rn qualquer. Logo, como o sistema linear nox1 | | . . singular, existe um nico x tal que Ax = b. Assim, se x = 1 . . . An , temos que . e A = A | | xn b = A1 x1 + + An xn , isto , b R(A). Temos ento que Rn R(A) e tambm que R(A) Rn , portanto R(A) = Rn .

Considere o sistema homogneo Ax = 0, onde A Rmn . Uma soluo particular que satisfaz o sistema x = 0, a soluo trivial, mas ainda assim, pode existir uma innidade de outras solues, e no que segue vamos estudar o conjunto formado por todas as solues do sistema homogneo. Denio 3.8: O conjunto formado por todas as solues do sistema homogneo denominado de ncleo de A, e denotado por N (A). Assim, N (A) = {x Rn |Ax = 0} Rn . Observao: Alguns livros denotam N (A) por kernel(A) ou ker(A). Proposio 3.9: N (A) um subespao vetorial de Rn . Demonstrao. A) Seja x, y N (A), logo x + y N (A). Tome Ax = 0 e Ay = 0. Ento, A(x + y ) = Ax + Ay = 0 + 0 = 0, logo, x + y uma soluo. ..............................................................................................

3 Espaos Vetoriais

48

M) Seja x N (A) e R. Ento, A(x) = (Ax) = 0 = 0. Portanto, N (A) um subespao vetorial de Rn . Observao: Se A no-singular ento N (A) = {0}, o mesmo vale para A sendo invertvel.
E XEMPLO 30: 1 2 A = 2 0 encontre N (A). 0 1

Fazendo Ax = 0, temos x + x2 1 2 0 1 x1 = 0 = 2x1 2 0 x2 0 0 1 + x2 2 1 0 2, qual N (A)? 1 0 =0 = 0 = x1 = x2 = 0. =0

logo N (A) = {0}. E XEMPLO 31:

1 Considere a matriz A = 2 1 ,

Observe que, a segunda coluna da matriz pode ser escrita como uma C.L. das outras duas, isto 2 1 1 1 2 1 0 0 = 2 + 2 = 2 0 + 2 = 0 1 1 0 1 1 0 0 1 2 1 1 0 2 1 0 2 1 = 0 1 0 1 0

Portanto,

1 assim, 1 N (A). 1

1 Observe que, para um R qualquer, 1 N (A), logo, 1 R N (A).

No caso em que temos uma matriz Ann , j sabemos que: ..............................................................................................

3 Espaos Vetoriais

49

Se A invertvel, ento Ax = b tem soluo nica. Se A no invertvel, ento Ax = b no tem soluo ou tem innitas solues. E no caso em que temos uma matriz Amn , com m < n ou m > n? Antes de discutirmos este caso, vamos denir o que uma matriz na forma escalonada ou

escada.
Denio 3.10 (Matriz Na Forma Escalonada ou Escada): Uma matriz Amn dita estar na forma escalonada ou na forma escada se ela satisfaz as seguintes condies: i) Todas as linhas nulas ocorrem , se existirem, ocorrem abaixo de todas as linhas nulas. ii) Os elementos abaixo das colunas que contm os pivs devem ser todos iguais a zero. iii) O piv da linha i + 1 deve sempre estar a direita do piv da linha i. Observao: Se no meio das linhas no-nulas aparecer uma linha nula, ento podemos atravs de permutaes deix-la como a ltima linha. Abaixo temos a representao de uma matriz na forma escalonada, onde os elementos dentro do box so os pivs. 0 U = 0 0 0 0 0 0 0 0 0 0 0 0 0 0 0 0 0 0 0 0 0 Observe que, quando aplicamos a fatorao A=LU, temos L triangular superior com 1s na diagonal e U na forma escalonada. De maneira geral, para qualquer matriz A Rmn existir uma matriz de permutao P de maneira que as operaes elementares possam ser desempenhadas deixando as linhas nulas no nal e, ento, podemos escrever PA=LU, onde L Rnn triangular inferior com 1s na diagonal e U Rmn est na forma escalonada.
E XEMPLO 32: Considere o sistema linear com 0 b = 0 0 1 A= 2 1 3 3 2 6 9 5 3 3 0

..............................................................................................

3 Espaos Vetoriais Aplicando a eliminao gaussiana, encontramos 1 U = 0 0 resolver U x = 0, logo 1 0 0 3 0 0 3 3 0

50

3 3 0 3 0 0

Observe que de Ax = b temos LU x = b ou ainda L1 b. Portanto, resolver Ax = 0 equivalente a


0

2 1 0

Vamos ento dividir as variveis x1 , x2 , x3 , x4 em dois grupos:

x1 0 2 x2 0 1 = x3 0 0 x4 0

1o Grupo: Ser o grupo das variveis que vamos chamar de variveis bsicas. As variveis bsicas sero aquelas associadas as pivs. 2o Grupo: Ser o grupo das variveis que vamos chamar de variveis livres. As variveis livres sero as demais variveis existentes, ou seja, as que no sero variveis bsicas. Vamos montar o sistema U x = 0: 1 x1 + 3x2 + 3x3 + 2x4 + 3 x3 + x4 =0 =0

logo, as nossas variveis bsicas so: x1 , x2 , e as variveis livres so: x2 , x4 . O objetivo deixar as variveis bsicas em funo das variveis livres, logo x4 e x1 = 3x2 x4 3 Assim, a soluo do sistema linear ser: 3x2 x4 3 1 x2 1 0 = x + x 2 4 x 3 1 0 4 3 x3 = x4 0 1 Temos um outro mtodo para encontrar a soluo do sistema U x = 0 atravs das variveis bsicas e livres. Este mtodo prtico no caso de termos sistemas com um nmero maior de variveis. Temos ento, que: 1 3 3 2 0 x1 0 + x2 0 + x3 3 + x4 1 = 0 0 0 0 0 0

onde, x1 e x3 so as variveis bsicas e x2 e x4 so as variveis livres.

..............................................................................................

Primeiro, faremos x2 = 0 e x4 = 1, logo 1 0 3 2 0 1 3 2 x1 0 + 0 + x3 3 + 1 = 0 = x1 0 + x3 3 = 1 0 0 0 0 0 0 0 0

3 Espaos Vetoriais Ento, x1 + x3 + 3x3 1 1 0 e, da, x1 = 1 e x3 = e uma soluo : 1 3 3 1 = 2 = 1

51

Ento,

Do mesmo modo, fazendo x2 = 1 e x4 = 0, temos 1 3 3 0 0 1 3 3 x1 0 + 0 + x3 3 + 0 = 0 = x1 0 + x3 3 = 0 0 0 0 0 0 0 0 0 x1 + x3 = 3

+ 3x3 =0 3 1 da, x1 = 3 e x3 = 0 e uma outra soluo : conseqentemente, o conjunto de todas as 0 0 solues ser da forma: 3 1 1 0 x = x2 + x4 1 0 3 0 1

ou seja,

3 1 1 0 N (U ) = x2 + x4 1 x2 , x4 R = N (A) 3 0 1 0

O Caso Geral Para Sistema Homogneos


i) Encontrar a matriz U na forma escalonada atravs da aplicao de operaes elementares sobre a matriz A. ii) Detectar as variveis bsicas e as variveis livres. variveis bsicas: aquelas associadas as colunas de U que contm os pivs. variveis livres: as restantes. iii) Encontrar a soluo geral do sistema deixando as variveis bsicas em funo das variveis livres. No exemplo anterior mostramos dois modos de encontrar a soluo geral de um sistema m n homogneo. Portanto, vamos formalizar agora o segundo procedimento para encontrar a soluo geral. ..............................................................................................

3 Espaos Vetoriais

52

1o ) Escolhemos uma das variveis livres e fazemos igual a 1 e as demais iguais a 0. Ento, resolvemos o sistema resultante, que ser triangular superior, e encontramos os valores das variveis bsicas. obtendo assim, um vetor que ser o primeiro da soluo geral do sistema Ax = 0. 2o ) Repetimos o procedimento do primeiro passo para cada uma das outras variveis livres, encontrando assim, sempre um vetor correspondente a cada varivel livre. 3o ) Aps isso, a soluo geral de Ax = 0 a combinao linear dos vetores obtidos no procedimento acima, onde cada vetor est associado a varivel livre correspondente. Note que, a soluo geral encontrada pelos dois procedimentos anteriores o conjunto de todas as solues do sistema, logo, aplicando este procedimento encontramos N (A).
E XEMPLO 33: Vamos pensar agora no caso em que b = 0, ou seja, em um sistema no-homogneo. Consideremos o mesmo sistema do exemplo anterior mas com b = 0. x1 b1 1 3 3 2 x 2 b2 6 9 5 = 2 x3 b3 1 3 3 0 x4 b4 Ax = b Da matriz A, obtemos P Ax = P b LU x = P b 1 3 U = 0 0 0 0 3 2 3 1 0 0 U x = L1 P b U x = C.
C

e vamos fazer a seguinte transformao:

como no houve troca de linhas a matriz P igual a identidade, portanto podemos omiti-la. Fazendo ento C = L1 b, temos b1 C = b2 2b1 b3 2b2 + 5b1 de U x = C , 1 3 0 0 0 0 x1 3 2 b1 x2 3 1 = b2 2b1 x3 0 0 b3 2b2 + 5b1 x4

1 0 L= 2 1 1 2

0 0 1

temos que o sistema Ax = b tem soluo se, e somente se, b3 2b2 + 5b1 = 0. 1 Vamos tomar b = 5, pois este satisfaz a condio acima. Assim, fazendo C = L1 b, encon5 tramos 1 C = 3 0

..............................................................................................

3 Espaos Vetoriais ento x1 2 1 x2 1 = 3 x3 0 0 x4

53

1 0 0

3 3 0 3 0 0

logo, a ltima equao 0 = 0, como era esperado, e as outras equaes so dadas por 3x3 + x4 x1 + 3x2 + 3x3 + 2x4 =3 =1 ou 1 x3 = 1 x4 3 x1 = 2 3x2 x4

O nosso objetivo agora encontrar uma soluo da forma x = xp + xh onde xp representa uma soluo particular para o sistema e xh uma soluo geral ao sistema homogneo associado. Para encontrarmos a soluo particular xp , fazemos todas as variveis livres iguais a zero e resolvemos o sistema resultante. No nosso caso, temos 1 3 1 x1 0 + x3 3 = 3 0 0 0 2 0 xp = 1 0

o que nos d x1 = 2, x3 = 1 e x2 = x4 = 0, logo a soluo particular para o nosso sistema

A soluo geral para o sistema homogneo associado, xh , j sabemos calcular, e do exemplo anterior temos que 3 1 1 0 xh = x2 + x4 1 0 3 0 1 Portanto a soluo geral de Ax = b : x = xp + xh 2 3 1 0 1 0 x = + x2 + x4 1 1 0 3 0 0 1

O Caso Geral Para Sistemas No-Homogneos


1o ) Encontrar a fatorao PA=LU; ..............................................................................................

3 Espaos Vetoriais

54

2o ) Encontrar as variveis bsicas e as variveis livres; 3o ) Resolver o sistema C = L1 P b para encontrar o vetor C e ento resolver U x = C ; 4o ) Achar xp (soluo particular) fazendo todas as variveis livres iguais a zero; 5o ) Calcular xh (soluo geral do sistema homogneo associado) em funo das variveis livres pelo procedimento visto para a resoluo do sistema homogneo; 6o ) Montar a soluo geral x = xp + xh . Observao: i) Se A Rmn tem r pivs, ento temos r variveis bsicas e n r variveis livres.

ii) Se r = n, ento no existe variveis livres, logo N (A) = {0} e portanto Ax = b tem soluo nica, isto , A no-singular, e no caso em que A quadrada, ento A invertvel. Portanto, das observaes acima podemos concluir que: Ax = b, com b R(A) tem soluo nica se, e somente se, o nmero de pivs igual ao nmero de incgnitas se, e somente se N (A) = {0}. Denio 3.11: Seja A Rmn com r pivs. Chamamos de posto de A, o nmero de pivs de A, denotado por posto(A), isto , posto(A) = r . Observao: O nmero de variveis livres dadas por n r tambm chamado de nulidade de A. Denio 3.12: Dizemos que A Rmn tem posto coluna completo, se posto(A) = n. Dizemos que A Rmn tem posto linha completo, se posto(A) = m.

..............................................................................................

4
Independncia Linear, Base e Dimenso
Denio 4.1 (C.L. Em Um Espao Vetorial V ): Sejam v1 , v2 , . . . , vp elementos de V . Dizemos que um vetor v V uma combinao linear de v1 , v2 , . . . , vp se existirem escalares a1 , a2 , . . . , ap R tais que v = a1 v1 + a2 v2 + + ap vp . Denio 4.2: Seja V um espao vetorial e v1 , v2 , . . . , vp V , dizemos que v1 , v2 , . . . , vp so linearmente dependentes (LD) se existirem constantes a1 , a2 , . . . , ap R, nem todas nulas, tais que a1 v1 + a2 v2 + + ap vp = 0 (4.1)

Caso contrrio, v1 , v2 , . . . , vp so ditos linearmente independentes (LI), isto , v1 , v2 , . . . , vp so LI se sempre que a1 v1 + a2 v2 + + ap vp = 0, tivermos a1 = a2 = . . . = ap = 0. Denio 4.3: Dizemos que {v1 , . . . , vp } um conjunto linearmente independente se v1 , . . . , vp so LI. Dizemos que {v1 , . . . , vp } um conjunto linearmente dependente se v1 , . . . , vp so LD. Teorema 4.4: Um conjunto {v1 , . . . , vp } V LD se, e somente se, existe pelo menos um vi , i = 1, . . . , p, que pode ser escrito como uma C.L. dos outros Demonstrao. Suponhamos que v1 , . . . , vp LD, ento pela Denio (4.2) existem a1 , . . . , ap R, nem todos nulos, tais que a1 v1 + a2 v2 + + ap vp = 0

4 Independncia Linear, Base e Dimenso

56

Considere ai = 0, para algum i {1, . . . p}. Ento, a1 v1 + + ai1 vi1 + ai+1 vi+1 + + aP vp = ai vi e, da, v1 = a1 ai1 ai+1 ap v1 + vi1 vi+1 + vp . ai ai a1 ai

e portanto, vi uma C.L. dos outros vetores. Por outro lado, suponhamos que vi pode ser escrito como uma C.L. dos outros vetores, ento, existem escalares a1 , . . . , ai1 , ai+i , . . . , ap tais que vi = a1 v1 + + ai1 vi1 + ai+1 vi+1 + + ap vp ou seja, a1 v1 + + ai1 vi1 vi + ai+1 vi+1 + + ap vp = 0 Portanto, {v1 , . . . , vp } um conjunto LD.

E XEMPLO 34: V = R3 . Sejam v1 , v2 V . Ento {v1 , v2 } LD se e somente se v1 e v2 estiverem na mesma reta, que passa pela origem. (v1 = v2 ).

v1

v2 E XEMPLO 35: V = R2 , e1 = (1, 0) e e2 = (0, 1). e1 e e2 so LI, pois a1 e 1 + a2 e 2 = 0 a1 (1, 0) + a2 (0, 1) = (0, 0) (a1 , a2 ) = (0, 0) a1 = 0 e a2 = 0 E XEMPLO 36: V = R2 {(1, 1), (1, 0), (1, 1)} LD, pois E XEMPLO 37: 1 3 Seja A = 2 6 1 3 A1 A2 3 9 3 A3 2 L1 5 L2 0 L3 A4

1 1 (1, 1) 1(1, 0) + (1, 1) = (0, 0). 2 2

..............................................................................................

4 Independncia Linear, Base e Dimenso As colunas de A so LI ou LD? e as linhas? Note que a coluna A2 = 3A1 + 0A3 + 0A4 . Portanto, {A1 , A2 , A3 , A4 } LD. Para as linhas, temos 1 2 1 0 3 6 3 0 x1 + x2 + x3 = 3 9 3 0 2 5 0 0 aplicando a eliminao gaussiana, encontramos x2 = 2x3 e x1 = 5x3 , logo L3 = 2L2 5L1 Portanto as linhas tambm so LD. E XEMPLO 38: 1 3 1 Seja A = 0 2 0 as colunas de A so LI ou LD? e as linhas? 0 0 1

57

logo AT x x = 0, portanto as linhas tambm so LI.

Observe que Ax = 0 x = 0, logo as colunas so LI. Para as linhas de A, temos 1 0 0 AT = 3 2 0 1 0 1

Observao: Uma matriz triangular inferior ou superior, com os elementos da diagonal todos diferentes de zero apresenta colunas e linhas LI. Teorema 4.5: Seja A Rmn . As colunas da matriz A formam um conjunto LI se, e somente se N (A) = {0}. x1 . . Demonstrao. Suponhamos, por absurdo, N (A) = {0}, ento x = . , com algum x1 = 0, tal xn que Ax = 0. Logo, x1 A1 + x2 A2 + + xn An = 0 com xi = 0, isto , as colunas de A so LD, contradio, pois por hiptese as colunas so LI. Suponhamos agora, por absurdo novamente, que as colunas de A so LD. Ento, existem x1 , . . . , xn , nem todos nulos, tais que x1 A1 + x2 A2 + + xn An = 0 ..............................................................................................

4 Independncia Linear, Base e Dimenso

58

x1 . . isto , Ax = 0 e x = . = 0. Portanto, x N (A) e N (A) = {0}, o que uma contradio. xn Portanto segue a tese. Denio 4.6: Seja V um espao vetorial e w1 , w2 , . . . , wl V . Se para qualquer v V existir escalares a1 , a2 , . . . , al , tais que v = a1 w1 + a2 w2 + + al wl dizemos que w1 , w2 , . . . , wl geram o espao vetorial V . O conjunto {w1 , w2 , . . . , wl } denominado de conjunto gerador de V , e denotado por ger{w1 , w2 , . . . , wl }. Observao: i) Se {w1 , w2 , . . . , wl } um conjunto gerador de V , ento V o conjunto formado por todas as combinaes lineares de w1 , w2 , . . . , wl . ii) Alguns livros usam a notao [w1 , w2 , . . . , wl ], e alguns livros em ingls tambm denotam por span{w1 , w2 , . . . , wl }. iii) Se {w1 , w2 , . . . , wl } um conjunto gerador de V , ento ger{w1 , w2 , . . . , wl } = V. Temos o seguinte procedimento para determinar se os vetores w1 , w2 , . . . , wl geram o espao vetorial V . Etapa 1: Escolha um vetor arbitrrio v V . Etapa 2: Verique se v uma C.L dos vetores dados. Se for, os vetores dados geram V , caso contrrio eles no geram V .
E XEMPLO 39: Seja V o espao vetorial de R3 e sejam v1 = (1, 2, 1), Os vetores v1 , v2 e v3 geram V ? Etapa 1: Seja v = (a, b, c) um vetor arbitrrio em R3 , onde a, b e c so nmeros reais quaisquer. Etapa 2: Temos que vericar se existem constantes c1 , c2 e c3 tais que c1 v1 + c2 v2 + c3 v3 = v. Isso nos leva ao seguinte sistema linear: c1 2 c1 c1 + c2 + +2c2 + c3 + c3 =a =b =c v2 (1, 0, 2), e v3 = (1, 1, 0).

..............................................................................................

4 Independncia Linear, Base e Dimenso Uma soluo

59

2a + 2b + c ab+c 4a b 2c , c2 = , c3 . 3 3 3 Como obtivemos uma soluo para qualquer escolha de a, b e c, conclumos que v1 , v2 , e v3 geram R3 , o que equivalente a ger{v1 , v2 , v3 } = R3 . c1 = E XEMPLO 40: Mostre que S= 1 0 0 0 1 0 0 , , 0 1 0 0 1

gera o subespao de M22 formado por todas as matrizes simtricas. Etapa 1: Uma matriz simtrica arbitrria tem a forma A= onde a, b e c so nmeros reais quaisquer. Etapa 2: Precisamos encontrar constantes d1 , d2 e dn tais que d1 1 0 0 + d2 0 0 1 1 0 + d3 0 0 0 a =A= 1 b b , c a b b , c

o que nos leva a um sistema linear cuja soluo d1 = a, d2 = b, d3 = c.

Como encontramos uma soluo para qualquer escolha de a, b, e c, conclumos que S gera o subespao dado. E XEMPLO 41: O conjunto {tn , tn1 , . . . , t, 1} gera Pn , j que todos os polinmios em Pn so da forma a0 tn + a1 tn1 + + an1 t + an , que uma C.L. de elementos de S . E XEMPLO 42: Sejam, w1 = (1, 0, 0), w2 = (0, 2, 0) e w3 = (1, 1, 0). Ento ger{w1 , w2 , w3 } = {(a, b, 0)/a, b R} E XEMPLO 43: Sejam w1 = (1, 0, 0) e w2 = (0, 2, 0). Ento ger{w1 , w2 , 0} = {(a, b, 0)/a, b R} E XEMPLO 44: Seja A Rmn , A = A1 , A2 , . . . , An . Ento
| | | | | |

ger{A1 , A2 , . . . , An } = R(A) ou seja, espao gerado pelas colunas de A.

..............................................................................................

4 Independncia Linear, Base e Dimenso

60

Denio 4.7: Seja V um espao vetorial. Dizemos que v1 , v2 , . . . , vk uma base de V, se: i) ger{v1 , v2 , . . . , vk } = V ii) {v1 , v2 , . . . vk } um conjunto LI.
E XEMPLO 45: V = R2 , e1 = (1, 0) e e2 = (0, 1). Temos que {e1 , e2 } base de V , conhecida tambm como base cannica de R2 . E XEMPLO 46: O conjunto {(1, 1), (0, 1)} tambm uma base de V = R2 . De fato, se (0, 0) = a(1, 1) + b(0, 1) = (a, a + b), ento a = b = 0. Isto , {(1, 1), (0, 1)} LI. Ainda, ger{(1, 1), (0, 1)} = V , pois dado v = (x, y ) V , temos (x, y ) = x(1, 1) + (y x)(0, 1), ou seja, v um vetor de R2 e tambm uma C.L. dos vetores (1, 1) e (0, 1). E XEMPLO 47: {(0, 1), (0, 2)} no base de R2 , pois um conjunto LD. Se (0, 0) = a(0, 1) + b(0, 2), temos a = 2b e, a e b no so necessariamente zero. (0,2) (0,1)

E XEMPLO 48: V = R3 . {(1, 0, 0), (0, 1, 0), (0, 0, 1)} uma base de R3 . Esta a base cannica de R3 , logo (0, 0, 0) = a(1, 0, 0) + b(0, 1, 0) + c(0, 0, 1) = (a, b, c) logo, a = b = c = 0, isto , {(1, 0, 0), (0, 1, 0), (0, 0, 1)} LI, e ainda ger{(1, 0, 0), (0, 1, 0), (0, 0, 1)} = V, pois dado v = (x, y, z ) V , temos (x, y, z ) = x(1, 0, 0) + y (0, 1, 0) + z (0, 0, 1). E XEMPLO 49: {(1, 0, 0), (0, 1, 0)} no base de R3 . LI, mas no gera todo R3 , isto , ger{(1, 0, 0), (0, 1, 0)} = R3 . E XEMPLO 50: V = M22 1 0 0 1 0 0 , , 1 0 0 0 0 0 , 0 0 0 1

uma base de V .

..............................................................................................

4 Independncia Linear, Base e Dimenso E XEMPLO 51: Vimos no Exemplo 41 que o conjunto de polinmios {tn , tn1 , . . . , t, 1}

61

gera Pn . Alm disso, podemos mostrar que {tn , tn1 , . . . , t, 1} linearmente independente fazendo a C.L. dos vetores a1 tn + a2 tn1 + + an t + an+1 = 0 Para que essa equao seja verdadeira para todos os valores de t, o polinmio esquerda do sinal de igualdade tem que ter innitas razes. Como o nico polinmio com innitas razes o polinmio nulo, a equao acima s vlida se, e somente se, a1 = a2 = = an = an+1 = 0 Portanto, {tn , tn1 , . . . , t, 1} um conjunto LI, logo uma base para Pn , chamada de base natural ou base cannica para Pn . E XEMPLO 52: 1 2 1 A = 0 0 2 0 0 0 2 3 0

A1 A2 A3 A4

base de R(A) = {A1 , A3 }. E XEMPLO 53: A Rmn com colunas LI: base de R(A) = { conjunto das colunas de A}.

Teorema 4.8: Sejam v1 , v2 , . . . , vn elementos de V , no nulos, tais que ger{v1 , v2 , . . . , vn } = V . Ento podemos extrair uma base de {v1 , v2 , . . . , vn } para V . Demonstrao. Se v1 , v2 , . . . , vn so um conjunto LI, ento eles satisfazem as condies para uma base, e nada temos a provar. Suponhamos ento, que {v1 , v2 , . . . , vn } um conjunto LD. Ento existem a1 , a2 , . . . , an R, nem todos nulos, tais que a1 v1 + a2 v2 + + an vn = 0 Suponhamos, sem perda de generalidade, que an = 0. Ento podemos escrever vn = a1 a2 an1 v1 + v2 + + vn1 an an an

ou seja, vn uma C.L. de v1 , v2 , . . . , vn1 e, portanto v1 , v2 , . . . , vn1 ainda geram V . Se v1 , . . . , vn1 for LD, ento existe uma C.L. deles dando o vetor nulo e com algum coeciente diferente de zero; assim, podemos extrair aquele vetor que corresponde a este coeciente. Seguindo desta forma, aps uma quantidade nita de passos, chegaremos a um subconjunto de {v1 , . . . , vn } formado por r (r n) vetores LI que ainda geram V , isto , formaremos uma base. ..............................................................................................

4 Independncia Linear, Base e Dimenso

62

Teorema 4.9: Seja {v1 , . . . , vn } V , tal que ger{v1 , . . . , vn } = V . Ento, qualquer conjunto com mais de n vetores necessariamente um conjunto LD. Demonstrao. Como ger{v1 , . . . , vn } = V , pelo Teorema 4.8, podemos extrair uma base para v de {v1 , . . . , vn }. Seja {v1 , . . . , vr }, com r n, tal base. Consideremos agora {w1 , . . . , wm } V , com m > n. Ento, existem aij R, i = 1, . . . , m e j = 1, . . . , r , tais que w1 = a11 v1 + a12 v2 + + a1r vr w2 = a21 v1 + a22 v2 + + a2r vr (4.2) . . . w = a v + a v + + a v m m1 1 m2 2 mr r Consideremos agora uma C.L. de w1 , . . . , wm , tal que c1 w1 + c2 w2 + + cm wm = 0 substituindo os valores de w em (4.3), e coletando os termos, obtemos 0 = (a11 c1 + a21 c2 + + am1 cm )v1 + + (a12 c1 + a22 c2 + + am2 cm )v2 + + (a1r c1 + a2r c2 + + amr cr )vr . Como v1 , v2 , . . . , vr so LI, ento a c + a21 c2 + + am1 cm = 0 11 1 . . . a c + a c + + a c = 0 1r 1 2r 2 mr r (4.3)

ou seja, temos um sistema linear homogneo com r equaes e m incgnitas c1 , . . . , cm . Como r n < m, ele admite uma soluo no trivial, ou seja, existe uma soluo com algum ci = 0. Logo {w1 , . . . , wm } um conjunto LD. Corolrio 4.10: Dado um espao vetorial V . Ento, toda base de V tem sempre o mesmo nmero de elementos. Demonstrao. Tome {v1 , . . . , vr } e {w1 , . . . , wq } bases de V . Devemos mostrar que q = r . Suponhamos que r > q , temos ento que ger{w1 , . . . , wq } = V e {v1 , . . . , vr } V ..............................................................................................

4 Independncia Linear, Base e Dimenso

63

logo, pelo Teorema 4.9, {v1 , . . . , vr } LD, o que uma contradio, pois {v1 , . . . , vr } uma base de V . Suponhamos ento, que r < q , logo ger{v1 , . . . , vr } = V e {w1 , . . . , wq } V e, pelo Teorema 4.9, {w1 , . . . , wq } LD, contradio novamente, pois {w1 , . . . , wq } uma base de V . Portanto, r = q . Denio 4.11: Chamamos de dimenso de um espao vetorial V no-nulo, o nmero de vetores de uma base de V . Denotaremos a dimenso de um espao vetorial V por: dim V . Observao: Como o conjunto {0} LD, comum dizer que o espao vetorial nulo, {0}, tem dimenso zero ou nula.
E XEMPLO 54: A dimenso de R2 2, a dimenso de R3 3; e em geral, a dimenso de Rn n. E XEMPLO 55: A dimenso de P2 3, a dimenso de P3 4; e em geral, a dimenso de Pn n + 1. E XEMPLO 56: Seja {v1 , v2 , v3 , v4 , v5 } um conjunto de vetores em R4 , onde v1 = (1, 2, 2, 1), v2 = (3, 0, 4, 3), v3 = (2, 1, 1, 1)

v4 = (3, 3, 9, 6),

v5 = (9, 3, 7, 6)

Encontre um subconjunto de {v1 , v2 , v3 , v4 , v5 } que seja uma base para ger{v1 , v2 , v3 , v4 , v5 } = W e tambm a dimenso de W . Para encontrarmos uma base para W , vamos escrever a C.L. dos vetores, logo 1 3 2 3 9 0 2 0 1 3 3 0 c1 + c2 + c3 + c4 + c5 = 2 4 1 0 7 0 1 3 1 6 6 0 c1 0 9 c2 3 0 c3 = 7 0 c4 6 0 c5

Assim, temos o seguinte sistema homogneo

1 3 2 3 2 0 1 3 2 4 1 9 1 3 1 6
A

..............................................................................................

4 Independncia Linear, Base e Dimenso Aplicando a eliminao gaussiana na matriz A, temos 1 3 2 3 9 1 3 0 0 6 6 3 9 15 0 10 5 15 25 0 0 0 6 3 9 15 0 0

64

2 3 9 3 9 15 =U 0 0 0 0 0 0

Portanto, a base que estvamos procurando so as colunas da matriz A onde aparecem os pivs na matriz U , logo 1 3 2 0 base de W = , 2 4 1 3 e dim W = 2.

Teorema 4.12: Qualquer conjunto {v1 , v2 , . . . , vr } linearmente independente de um espao vetorial V com dimenso nita, pode ser completado (extendido) de modo a formar uma base de V . n. Se Demonstrao. Seja v1 , v2 , . . . , vr vetores LI e dim V = n, assim pelo Teorema 4.9, r ger{v1 , v2 , . . . , vr } = V , ento {v1 , . . . , vr } forma uma base de V , e no temos mais nada a fazer, isto para o caso em que n = r . Agora, se ger{v1 , v2 , . . . , vr } = V , ento existe algum vetor vr+1 V que no uma C.L. de v1 , v2 , . . . , vr , isto , vr+1 / ger{v1 , v2 , . . . , vr } e o conjunto {v1 , v2 , . . . , vr } LI, pois nenhum dos seus vetores pose ser escrito como uma C.L. dos vetores que o antecedem. Se ger{v1 , v2 , . . . , vr , vr+1 } = V , ento {v1 , v2 , . . . , vr , vr+1 } a base procurada. Caso contrrio, existe / ger{v1 , v2 , . . . , vr , vr+1 } e, ento {v1 , v2 , . . . , vr , vr+1 , vr+2 } LI, algum vetor vr+2 V e vr+2 da, se ger{v1 , v2 , . . . , vr , vr+1 , vr+2 } = V , nossa prova est concluda. Se no, continuamos usando o mesmo argumento. Como no podemos ter mais do que n vetores LI em V (Teorema 4.9), aps um nmero nitos de passos encontraremos a base de V que contm os vetores dados. Corolrio 4.13: Se dim V = n, qualquer conjunto de n vetores LI formar uma base de V . Demonstrao. Suponhamos, que no forme uma base, assim poderamos completar o conjunto at forma-l, e portanto, teramos uma base com mais do que n vetores em V , o que uma contradio. Por exemplo, se sabemos que a dim V = 2, e encontrarmos um conjunto de dois vetores LI, podemos armar que este conjunto uma base e portanto gera V . Denio 4.14 (Os Quatro Subespaos Fundamentais): Seja A Rmn , ento i) R(A) = {y Rm |y = Ax, para algum x Rn } = {Ax|x Rn }, o conjunto formado por todas as C.L.s possveis das colunas de A, denominado espao coluna de A ..............................................................................................

4 Independncia Linear, Base e Dimenso

65

ii) N (A) = {x Rn |Ax = 0}, o conjunto formado por todas as solues do sistema homogneo, denominado espao nulo de A ou ncleo de A. iii) R(AT ) = {z Rn |z = AT w, para algum w Rn }, o conjunto formado por todas as C.L.s possveis das linhas de A, denominado espao linha de A. iv) N (AT ) = {w Rn |AT w = 0} = {w Rn |wT A = 0}, denominado de espao nulo esquerda de A ou ncleo esquerda de A. Observao: Note que, R(A), N (AT ) Rm , enquanto que R(AT ), N (A) Rn .
E XEMPLO 57: Seja a matriz A = 1 0 0 0 , ento: 0 0 1 0 0 0 0 N (A) = x2 x2 , x3 R = ger 1 , 0 x3 0 1 N (AT ) = 0 w2 w2 R = ger 0 1

R(A) = ger

A seguir, vamos discutir como podemos extrair uma base para cada uma dos quatro subespaos fundamentais de A Rmn . Para isso, vamos usar a fatorao PA=LU e encontrar qual a relao existente entre os subespaos de U com os subespaos de A. Vamos denir no prximo exemplo uma matriz A e a matriz U resultante da aplicao da fatorao PA=LU na matriz A. Usaremos estas duas matrizes nos prximos exemplos sem precisar reescrevelas toda vez.
E XEMPLO 58 (Resultado Para os Prximos Exemplos): 1 3 3 A= 2 6 9 1 3 3 1 U = 0 0 3 3 0 3 0 0 2 5 0

1 T R(A ) = ger 0 , 0

1 0 0 L = 2 1 0 1 2 1

2 1 0

A matriz P a prpria identidade, pois no houve troca de linhas na fatorao.

..............................................................................................

4 Independncia Linear, Base e Dimenso

66

Proposio 4.15 (Base Para R(AT )): Dada A Rmn e a fatorao PA=LU, U na forma escalonada, uma base para R(AT ) o conjunto formado pelas linhas no nulas de U que contm os pivs. Com efeito, ao fazer a eliminao gaussiana estamos fazendo a C.L. da linhas, ou seja, estamos sempre preservando o espao linha da matriz A, isto , R(AT ) = R(U T ) Observao: Se a matriz A Rmn tem posto(A) = r ento, dim R(AT ) = r.
E XEMPLO 59: Considere as matrizes do Exemplo 58, ento 1 0 3 0 T base R(A ) = , = { linhas no-nulas de U } 3 3 2 1 dim R(AT ) = 2

(4.4)

(4.5)

Proposio 4.16 (Base Para N (A)): Dada A Rmn e a fatorao PA=LU, uma base para N (A) pode ser construda reduzindo U x = 0, onde existem n r variveis livres que correspondem as colunas de U que no contm pivs. Ento, damos para cada varivel livre o valor 1 e para todas as outras variveis livres o valor 0, e resolvemos U x = 0 atravs de uma substituio regressiva para as variveis bsicas restantes. Assim, os n r vetores produzidos deste modo so a base para N (A), ou seja, N (A) = N (U ) Observao: Se A Rmn e tem posto(A) = r , ento dim N (A) = n r.
E XEMPLO 60: No exemplo 58, base para N (A) = base para N (U ), logo, Ax = 0 P Ax = 0 LU x = 0 U x = L1 0 U x = 0 Assim, temos o seguinte sistema 1 0 0 3 0 0 3 3 0 x1 0 2 x 2 0 1 = x3 0 0 x4 0

(4.6)

(4.7)

Temos 2 pivs, logo pela Equao (4.7), sabemos que a base N (A) ter n r = 4 2 = 2 vetores.

..............................................................................................

4 Independncia Linear, Base e Dimenso

67

Vamos encontrar o primeiro vetor da base do N (U ), fazendo x2 = 1 e x4 = 0 que so as variveis livres, logo 1 3 3 2 0 x1 0 = 1 0 + x3 3 + 0 1 = 0 0 0 0 0 0 e, da, 1 0 0 3 3 x1 = 0 3 x2 0 0

de onde obtemos: x1 = 3 e x3 = 0. 3 1 primeiro vetor da base: . 0 0

e, da,

Vamos encontrar agora o segundo vetor da base para N (U ). Fazendo x2 = 0 e x4 = 1, temos 1 3 3 2 0 x1 0 + 0 0 + x3 0 + 1 1 = 0 0 0 0 0 0 1 0 0 2 3 x1 = 1 3 x3 0 0

de onde obtemos: x1 = 1 e x3 =

1 3 1 0 segundo vetor da base: 1 . 3 1 Ento,

ou ainda,

3 1 1 0 base N (U ) = , 1 = base N (A). 0 3 1 0 3 1 1 0 N (A) = x2 + x4 1 x2 , x4 R 0 3 1 0 dim N (A) = 2

Proposio 4.17 (Base Para R(A)): Dada A Rmn e a fatorao PA=LU, a base para R(A) formada pelas colunas da matriz A associadas s colunas dos pivs da matriz U . ..............................................................................................

4 Independncia Linear, Base e Dimenso

68

A razo que, Ax = 0 exatamente quando U x = 0. Este dois sistemas so equivalentes e tem as mesmas solues. Assim, se
| | | | | | | | | | | |

A = A1 A2 . . . An , x1 x2 tal que Ax = 0, ento ex= . . . xn

U1 U2 . . . Un

x1 A1 + x2 A2 + + xn An = 0 x1 U1 + x2 U2 + + xn Un = 0.

Portanto, a se uma coluna Ai de A pode ser escrita como um C.L. das outras colunas, temos que Ui tambm pode escrita como uma C.L. das outras colunas de U , ambas as combinaes com os mesmos coecientes. Ento, se um conjunto de colunas de A LI, tambm sero as colunas correspondentes de U , e vice-versa. Logo, a Proposio 4.17 nos garante que as colunas escolhidas so LIs, pois sabemos que as colunas de U associadas aos pivs tambm so LIs. Observao: Se A Rmn e tem posto(A) = r , ento dim R(A) = r (4.8)

Note que R(A) = R(U ), e que se A Rm tem posto(A) = r , ento dim R(A) = dim R(AT ), isto , o nmero de colunas LI igual ao nmero de linhas LI.
E XEMPLO 61: No Exemplo 58 3 1 base R(A) = 2 , 9 . 1 3 dim R(A) = 2.

Proposio 4.18 (Base Para N (AT )): Dada a matriz A Rmn e a fatorao PA=LU, fazendo L1 P A = U , uma base para N (AT ) so as linhas de L1 P associadas s linhas nulas de U . Se A Rmn , ento AT Rnm , isto , o espao nulo de AT um subespao de Rm . Assim, precisamos encontrar os vetores y Rm , tais que y T A = 0, ou seja, as componentes que multiplicando a matriz A produzem linhas nulas: y T A = y1 y2 ym A = 0 0 0

Para encontrarmos esses y s, comeamos pela fatorao PA=LU e, ento, fazemos L1 P A = U . Ento, as linhas de L1 P associadas as linhas nulas de U sero a base do N (AT ), porque estas linhas quando multiplicam a matriz A, produzem as linhas nulas de U . ..............................................................................................

4 Independncia Linear, Base e Dimenso

69

Note que, como estamos fazendo a fatorao PA=LU as linhas nulas de U , quando existirem, sempre sero as ltimas, conseqentemente a base para N (AT ) sero as mesmas ltimas linhas de L1 P . Ento: . . . 1 L P = A=U y T Assim, se a matriz A Rmn tem posto(A) = r , para calcularmos N (AT ) precisamos das m r linhas de L1 P = (P T L)1 . Fazendo x = (P T L)1 , temos que x(P T L) = I (LT P )xT = I LT P xT = I, logo as m r linhas de x que queremos so as mesmas que as m r colunas de xT . Logo, para encontrarmos estes vetores temos que resolver (m r ) sistemas do tipo, (LT P )yi = ei onde, i = r + 1, . . . , m. Observao: Se A Rmn e tem posto(A) = r , ento: dim N (AT ) = m r (4.9)

e, aplicando uma substituio regressiva, encontramos 5 y = 2 . 1 Portanto,

E XEMPLO 62: No exemplo 58, temos que apenas a ltima linha da matriz U nula, logo (LT P )y = e3 e ainda, como P = I , basta fazermos LT y = e3 , 0 1 2 1 y1 0 1 2 y2 = 0 1 0 0 1 y3

5 base N (AT ) = 2 1 dim N (AT ) = 1

Observao: Note que, quando aplicamos a eliminao gaussiana na matriz do exemplo 58, a linha nula da matriz U foi obtida quando zemos o passo l3 l3 + (2)l2 ..............................................................................................

4 Independncia Linear, Base e Dimenso

70

mas as linhas 2 e 3 j haviam sido modicadas pelos passos l3 l3 + l1 e l2 l2 + (2)l1 substituindo esses dois passos no anterior, camos com 0 = l3 + l1 + (2)(l2 + (2)l1 ) ou seja, 0 = l3 2l2 + 5l1 e, da, 5 y = 2 1

E XEMPLO 63: Vamos encontrar uma base para cada um dos 4 subespaos para a matriz a seguir. 3 2 0 4 l2 l2 + 2l1 6 4 1 1 l3 l3 + l1 3 2 1 5 3 2 0 4 0 0 1 9 l3 l3 + (1)l2 0 0 1 9 3 U =0 0 2 0 0 0 1 0 4 9 0

da,

1 0 0 L = 2 1 0 1 1 1 1 P = 0 0 0 0 1 0 0 1

0 3 2 0 T base R(A ) = , 0 1 4 9 0 3 base R(A) = 6 , 1 3 1

dim R(AT ) = 2

dim R(A) = 2

Para encontrar uma base para N (A), temos que x1 , x3 so as variveis bsicas e x2 , x4 as variveis livres, logo

..............................................................................................

4 Independncia Linear, Base e Dimenso para x2 = 1 e x4 = 0

71
3 0 2 x1 = 0 0 1 x2 0 0 0 2
3

2 e obtemos x1 = e x3 = 0, e o primeiro vetor da base : 3 para x2 = 0 e x4 = 1

3 0 4 x1 = 9 0 1 x2 0 0 0

1 . 0 0

0 4 e x3 = 9, e o segundo vetor da base : . Portanto, de onde obtemos x1 = 3 9 1 2 4 3 3 1 0 base N (A) = , 0 9 0 1 e dim N (A) = 2

4
3

Vamos encontrar agora uma base para N (AT ). Note que, como apenas a ltima linha da matriz U nula, precisamos resolver apenas um sistema da forma (LT P )y = e3 logo, 1 0 0 2 1 y1 0 1 1 y2 = 0 0 1 y3 1

e, da, y3 = 1, y2 = 1 e y1 = 1. Portanto, 1 base N (AT ) = 1 1

dim N (AT ) = 1.

Ainda, podemos encontrar a base N (AT ) atravs dos passos da fatorao PA=LU, fazendo 0 = l3 + (1)l2 0 = l3 + (1)(l2 + 2l1 ) 0 = l3 + l1 (1)(l2 + 2l1 )

0 = l3 2 l2 l1 1 que so os valores de y , isto , y = 1. 1

..............................................................................................

4 Independncia Linear, Base e Dimenso

72

Teorema 4.19: Seja a matriz A Rmn e posto(A) = r , ento: dim N (A) = n r dim N (AT ) = m r dim R(A) = r dim R(AT ) = r e, portanto, i) dim N (A) + dim R(AT ) = n ii) dim N (AT ) + dim R(A) = m

..............................................................................................

5
Transformaes Lineares
Denio 5.1: Sejam V e W espaos vetoriais. Uma transformao linear T : V W uma funo que associa a cada vetor u V um vetor T (u) W , tal que para quaisquer u, v V e R, as seguintes propriedades so vlidas: T1) T (u + v ) = T (u) + T (v ) T2) T (u) = T (u) No caso em que V = W , a transformao linear T : V W denominada um operador linear de V .
E XEMPLO 64: Seja T : R R denida por x 2x ou T (x) = 2x uma transformao linear, pois: T1) x, y R, tem-se T2) R, tem-se T (x + y ) = 2x + 2y = T (x) + T (y ) 2x = 2x = T (x).

T (x) =

E XEMPLO 65: T : R R denida por u u2 ou T (u) = u2 no uma transformao linear, pois u, v R T (u + v ) = (u + v )2 = u2 + 2uv + v 2 e T (u) + T (v ) = u2 + v 2 Portanto, T (u + v ) = T (u) + T (v ).

5 Transformaes Lineares

74

E XEMPLO 66: T : R2 R3 denida por (x, y ) (2x, 0, x + y ) ou T (x, y ) = (2x, 0, x + y ) uma transformao linear, pois T1) x1 x2 , R2 , tem-se: y1 y2 x1 x2 + y1 y2 2(x1 + x2 ) x1 + x2 =T = 0 y1 + y2 (x1 + x2 ) + (y1 + y2 ) 2x1 2x2 x1 = 0 + 0 =T +T y1 x1 + y1 x2 + y2 2x 2x = 0 = 0 = T x + y x+y

x2 y2

T2) R, tem-se: T x y =T x y

x y

E XEMPLO 67: Seja T : P1 P2 dada por T (at + b) = t(at + b). Mostre que T uma transformao linear. Sejam at + b e ct + d vetores em P1 e R. Ento T1) T [(at + b) + (ct + d)] = t[(at + b) + (ct + d)] = t(at + b) + t(ct + d) = T (at + b) + T (ct + d) T2) T [(at + b)] = t[(at + b)] = t(at + b) = t(at + b) = T (at + b). Portanto, T uma transformao linear. E XEMPLO 68: T : P1 P2 denida por T [p(t)] = tp(t) + t2 uma transformao linear? Sejam p(t) e q (t) vetores em P1 e R. Ento T [p(t) + q (t)] = t[p(t) + q (t)] + t2 = tp(t) + tq (t) + t2 e T [p(t)] + T [q (t)] = [tp(t) + t2 ] + [tq (t) + t2 ] = tp(t) + tq (t) + 2t2 Como T [p(t) + q (t)] = T [p(t)] + T [q (t)], segue que T no uma transformao linear. E XEMPLO 69: Seja T : Mnn Mnn denida por T (A) = AT para A Mnn . T uma transformao linear? Sejam A, B Mnn T1) T (A + B ) = (A + B )T = AT + B T = T (A) + T (B ) T2) T (A) = (A)T = AT = T (A). Portanto, T uma transformao linear.

..............................................................................................

5 Transformaes Lineares

75

Teorema 5.2: Se T : V W uma transformao linear, ento: T (1 u1 + 2 u2 + + n un ) = 1 T (u1 ) + 2 T (u2 ) + + n T (un ) quaisquer que sejam os vetores u1 , u2 , . . . , un V e 1 , 2 , . . . , n R. Demonstrao. Provaremos pro induo. Para n = 1, temos T (1 u1 ) = 1 T (u1 ) Suponhamos vlido para n = k, isto , T (1 u1 + 2 u2 + + k uk ) = 1 T (u1 ) + 2 T (u2 ) + + k T (uk ). Vamos provar que vale para n = k + 1, isto , que T (1 u1 + 2 u2 + + k uk + k+1 uk+1 ) = 1 T (u1 ) + 2 T (u2 ) + + k T (uk ) + k+1 T (uk+1 ). Fazendo u = 1 u1 + 2 u2 + + k uk , temos que T (u + k+1 uk+1 ) = T (u) + k+1 T (uk+1 ) e, da, pela hiptese de induo T (u) = 1 T (u1 ) + 2 T (u2 ) + + k T (uk ) ou seja, T (1 u1 + 2 u2 + + k uk + k+1 uk+1 ) = 1 T (u1 ) + 2 T (u2 ) + + k T (uk ) + k+1 T (uk+a ) Portanto, pelo Princpio de Induo segue a tese. Teorema 5.3: Se T : V W uma transformao linear, ento: i) T (0) = 0 ii) T (u) = T (u) iii) T (u v ) = T (u) T (v ) Demonstrao. i) Como 0 + 0 = 0 e T uma transformao linear, temos T (0) = T (0 + 0) = T (0) + T (0). Somando T (0) em ambos os lados obtemos 0 = T (0). O mesmo vale para T (0) = T (0) = T (0) R, em especial para = 0. ..............................................................................................

5 Transformaes Lineares

76

ii) Para provar esta propriedade, tomemos = 1 em T (u) = T (u), logo T (1u) = 1T (u) isto , T (u) = T (u). iii) Escrevendo u v = u + (v ) temos por T1), que T (u v ) = T (u + (v )) = T (u) + T (v ) como (v ) = (1)v , segue por ii), que T (v ) = T (v ). Da, T (u v ) = T (u) + T (v ) = T (u) + (T (u)) = T (u) T (v ). Teorema 5.4: Sejam V e W espaos vetoriais de dimenso nita, T : V W uma transformao linear e {v1 , v2 , . . . , vn } uma base de V . Se u um vetor qualquer de V , ento T (u) ca completamente determinado por {T (v1 ), T (v2 ), . . . , T (vn )}. Demonstrao. Como u V , podemos escrever u = 1 v1 + 2 v2 + + n vn , onde 1 , 2 , . . . , n R. Ento, T (u) = T (1 v1 + 2 v2 + + n vn ) = 1 T (v1 ) + 2 T (v2 ) + + n T (vn ), pelo Teorema 5.2. Portanto, T (u) cou completamente determinado pelos elementos T (v1 ), T (v2 ), . . . , T (vn ). Teorema 5.5: Dados dois espaos vetoriais reais V e W e {v1 , v2 , . . . , vn } uma base de V , sejam w1 , . . . , wn elementos arbitrrios de W . Ento existe uma nica transformao linear T : V W tal que T (v1 ) = w1 , T (v2 ) = w2 , . . . , T (vn ) = wn . Demonstrao. Todo vetor v V se exprime de modo nico como uma C.L. v = 1 v1 + + n vn de elementos v1 , . . . , vn da base {v1 , . . . , vn }. Denimos T : V W pondo T (v ) = 1 w1 + + n wn Dados v, w V , temos v = 1 v1 + + n vn

..............................................................................................

5 Transformaes Lineares

77

e w = 1 v1 + + n vn Ento, v + w = (1 v1 + + n vn ) + (1 v1 + + n vn ) = (1 v1 + 1 v1 ) + + (n vn + n vn ) = (1 + 1 )v1 + + (n + n )vn


n

=
i=1

(i + i)vi

logo, T (v + w) = (i + i)wi = i w1 + 1 w1 = T (v ) + T (w).

De maneira anloga, v-se que T (v ) = T (v ), portanto T : V W , assim denida, uma transformao linear tal que T (vi ) = wi , para todo vi {v1 , . . . , vn }, com i = 1, . . . , n. Para mostrarmos a unicidade, seja L : V W uma outra transformao linear tal que L(v1 ) = w1 , . . . , L(vn ) = wn . Ento, para cada v = i vi V tem-se L(v ) = L 1 v1 = i L(v1 ) = i wi = T (v ).

Portanto, L = T e a demonstrao est completa.


E XEMPLO 70: Qual a transformao linear T : R2 R3 tal que T (1, 0) = (2, 1, 0) e T (0, 1) = (0, 0, 1)? Temos neste caso e1 = (1, 0) e e2 = (0, 1) base de R2 e w1 = (2, 1, 0) e w2 = (0, 0, 1). Dado v = (x1 , x2 ) qualquer, v = x1 e1 + x2 e2 e T (v ) = x1 T (e1 ) + x2 T (e2 ) = x1 (2, 1, 0) + x2 (0, 0, 1) = (2x1 , x1 , x2 ) E XEMPLO 71: Qual a transformao linear T : R2 R3 tal que T (1, 1) = (3, 2, 1) e T (0, 2) = (0, 1, 0)? Dado v = (x1 , x2 ) qualquer, temos v = c1 (1, 1) + c2 (0, 2) T (v ) = c1 T (1, 1) + c2 T (0, 2) vamos, determinar os valores de c1 e c2 : (x1 , x2 ) = c1 (1, 1) + c2 (0, 2) x = c 1 1 x2 = c1 2c2

..............................................................................................

5 Transformaes Lineares de onde obtemos c1 = x1 e c2 = x1 x2 . Logo, 2 x1 x2 (0, 2) 2 x1 x2 T (0, 2) T (v ) = x1 T (1, 1) + 2 x1 x2 T (v ) = x1 (3, 2, 1) + (0, 1, 0) 2 5x1 x2 T (v ) = 3x1 , , x1 . 2 v = x1 (1, 1) +

78

Denio 5.6: Seja T : V W uma transformao linear, denimos: i) Ncleo de T = {v V |T (v ) = 0} ii) Imagem de T = {w W |w = T (v ), para algum v V } Denotaremos ncleo de T por N (T ) ou ker(T ) e a imagem de T por I m(T ).
E XEMPLO 72: T : R2 R denido por (x, y ) x + y . Assim, N (T ) = {(x, y ) R2 |x + y = 0}, isto , N (T ) a reta y = x, ou ainda, podemos escrever N (T ) = {(x, x)|x R} = {x(1, 1)|x R} Por outro lado, I m(T ) = R, pois dado w R, w = T (w, 0)
y N (T )

I m(T ) = R x 0

y = x

Teorema 5.7: Se T : V W uma transformao linear, ento N (T ) um subespao de V . Demonstrao. Note que N (T ) nunca vazio, pois 0 est no ncleo. Ento, sejam u, v N (T ), como T uma transformao linear T (u + v ) = T (u) + T (v ) = 0 + 0 = 0 logo, u + v N (T ). Alm disso, seja R, ento T (u) = T (u) = 0 = 0 e u N (T ). Portanto N (T ) um subespao de V . ..............................................................................................

5 Transformaes Lineares

79

Observao: Se T : Rn Rm uma transformao linear denida por T (x) = Ax, onde A Rmn , ento o ncleo de T o espao soluo do sistema homogneo Ax = 0. Denio 5.8: Dizemos que uma transformao linear T : V W : i) injetora, se u, v V e T (u) = T (v ) implicar em u = v . Ou equivalentemente, se u = v implicar T (u) = T (v ). ii) sobrejetora, se w W existir v V tal que T (v ) = w.
V W V W

u v

T (u) T (v)

T (u) = w

T injetora

T sobrejetora

E XEMPLO 73: Seja T : R2 R2 denida por T (x, y ) = (x + y, x y ). Vamos determinar se T injetora ou no, para isso, sejam u1 = (a1 , a2 ) e u2 = (b1 , b2 ) ento se, T (u1 ) = T (u2 ) temos a + a = b + b 1 2 1 2 a1 a2 = b1 b2

somando as duas equaes, obtemos 2a1 = 2b1 , isto , a1 = b1 o que implica que a2 = b2 . Portanto, u1 = u2 e assim, T injetora.

Teorema 5.9: Seja T : V W uma transformao linear. Ento T injetora se, e somente se, N (T ) = {0}. Demonstrao. Seja x N (T ), ento T (x) = 0, mas pelo Teorema 5.3i), T (0) = 0, da, T (x) = T (0), logo x = 0 pois T injetora. Portanto N (T ) = {0}. Reciprocamente, sejam x, y V tais que T (x) = T (y ). Ento T (x) T (y ) = 0 da, pelo Teorema 5.3iii), T (x) T (y ) = T (x y ) = 0, isto , x y N (T ). Mas por hiptese, o N (T ) = {0}, ento x y = 0, isto , x = y . Logo T injetora. ..............................................................................................

5 Transformaes Lineares

80

Teorema 5.10: Uma transformao linear injetora se, e somente se, leva vetores LI em vetores LI. Demonstrao. Seja T : V W uma transformao linear injetora. Se u1 , u2 , . . . , un V so LI, vamos provar que suas imagens T (u1 ), T (u2 ), . . . , T (un ) W so LI. Com efeito, se 1 T (u1 ) + 2 T (u2 )+ + n T (un ) = 0 ento T (1 u1 + 2 u2 + + n un ) = 0, logo 1 u1 + 2 u2 + + n un = 0 pois T injetora. Como u1 , u2 , . . . , un so LI, segue que 1 = 2 = = n = 0, portanto T (u1 ), T (u2 ), . . . , T (un ) so LI. Reciprocamente, se a transformao linear T : V W leva vetores LI em vetores LI, ento u = 0 em V implica que {u} LI {T (u)} LI T (u) = 0. Portanto N (T ) = {0} e T injetora. Teorema 5.11 (Teorema do Ncleo e Imagem): Seja T : V W uma transformao linear. Ento dim N (T ) + dim I m(T ) = dim V Demonstrao. Seja {v1 , . . . , vn } uma base do N (T ). Como N (T ) V subespao de V , podemos completar este conjunto de modo a obter uma base de V . Seja ento {v1 , . . . , vn , w1 , . . . , wm } a base de V . Queremos mostrar que T (w1 ), . . . , T (wm ) uma base da I m(T ). Dado w I m(T ), temos que existe u V tal que T (u) = w. Como u V , existem a1 , . . . , an , b1 . . . , bm R tais que u = a1 v1 + + an vn + b1 w1 + + bm wm . Ento w = T (u) = T (a1 v1 + + an vn + b1 w1 + + bm wm ) = a1 T (v1 ) + + an T (vn ) + b1 T (w1 ) + + bm T (wm ) como v1 , . . . , vn N (T ), T (vi ) = 0 para i = 1, . . . , n. Assim, w = b1 T (w1 ) + + bm T (wm ) Portanto, w ger{T (w1 ), . . . , T (wm )} e conseqentemente I m(T ) = ger{T (w1 ), . . . , T (wm )}. Vamos mostrar agora que {T (w1 ), . . . , T (wm )} LI. Sejam c1 , . . . , cm R tais que c1 T (w1 ) + + cm T (wm ) = 0 da, T (c1 w1 ) + + T (cm wm ) = 0 isto , T (c1 w1 + + cm wm ) = 0 ..............................................................................................

5 Transformaes Lineares

81

ento, c1 w1 + + cm wm N (T ). Logo, c1 w1 + + cm wm pode ser escrito como uma C.L. da base {v1 , . . . , vn } do N (T ), isto , existem d1 , . . . , dn R tais que c1 w1 + + cm wm = d1 v1 + + dn vn ou, c1 w1 + + cm wm d1 v1 + dn vn = 0 Mas, {v1 , . . . , vn , w1 , . . . , wm } uma base de V e, da c1 = = cm = d1 = = dn = 0. Portanto {T (w1 ), . . . , T (wm )} LI.

Corolrio 5.12: Seja T : V W uma transformao linear e suponha que dim V = dim W . Ento, T injetora se, e somente se, T sobrejetora. Demonstrao. Suponha que T injetora, ento N (T ) = {0} e da, dim N (T ) = 0. Ento, pelo Teorema 5.11, dim I m(T ) + 0 = dim V = dim W ou seja, dim I m(T ) = dim W logo I m(T ) = V Portanto, T sobrejetora. Suponha agora que T sobrejetora. Ento, I m(T ) = W . Portanto, dim I m(T ) = dim W = dim V. Assim, pelo Teorema 5.11, dim N (T ) = dim V dim I m(T ) = 0 logo, N (T ) = {0} e portanto, T injetora.

Denio 5.13: Seja T : V W uma transformao linear. Se T injetora e sobrejetora, dizemos que T um isomorsmo de V em W . Neste caso, dizemos que V isomorfo W .

..............................................................................................

5 Transformaes Lineares E XEMPLO 74: Pn isomorfo ao Rn+1 De fato, dado p Pn denido por p(t) = a0 + a1 t + + an tn dena T : Pn Rn+1 por a0 a1 T (p) = . . . an

82

Note que dim Pn = dim Rn+1 = n + 1.

Pelo Corolrio 5.12 basta mostrar a sobrejetividade de T , pois a injetividade segue como conseqncia. b1 b2 Rn+1 , tome Dado y = . . . bn q (t) = b0 + b1 t + + bn tn Pn (R) b0 b1 =y T (q ) = . . . bn ento,

Portanto T sobrejetora e injetora, logo Pn (R) isomorfo ao Rn+1 . E XEMPLO 75: Dado V espao vetorial de dimenso nita, com dim V = n. Ento, V isomorfo ao Rn . Fixe = {v1 , . . . , vn } base de V . Dado v V , existem x1 , . . . , xn R tais que v = x1 v1 + + xn vn Dena T : V Rn por x1 x2 Rn . T (v ) = . . . xn

Assim, prova-se facilmente que T sobrejetora, conseqentemente injetora, ou seja V isomorfo ao Rn . E XEMPLO 76: Seja T : R3 R3 dada por T (x, y, z ) = (x 2y, z, x + y ). Vamos mostrar que T um isomorsmo.

..............................................................................................

5 Transformaes Lineares Pelo Corolrio 5.12, basta mostrarmos que T injetora. Assim, basta mostrarmos que N (T ) = {(0, 0, 0)}. Mas, N (T ) = {(x, y, z )|T (x, y, z ) = (0, 0, 0)}

83

Resolvendo, encontramos que x = y = z = 0 a nica soluo. Portanto T um isomorsmo. E XEMPLO 77: Seja T : P2 P2 a transformao linear denida por

e T (x, y, z ) = {(0, 0, 0)} se, e somente se (x 2y, z, x + y ) = (0, 0, 0). Logo, temos o seguinte sistema x 2y = 0 z=0 x+y =0

T (at2 + bt + c) = (a + 2b)t + (b c). a) 4t2 + 2t 2 pertence ao N (T )? b) t2 + 2t + 1 pertence a I m(T )? c) Encontre uma base para N (T ) d) T injetora? e) Encontre uma base para I m(T ) f) T sobrejetora? g) Verique o Teorema do Ncleo e Imagem. Soluo: a) Como T (4t2 + 2t 2) = (4 + 2 2)t + (2 + 2) = 0, conclumos que 4t2 + 2t 2 pertence ao N (T ). b) O vetor t2 + 2t1 pertence a I m(T ) se existe um vetor at2 + bt + c em P2 tal que T (at2 + bt + c) = t2 + 2t + 1. Como T (at2 + bt + c) = (a + 2b)t + (b + c), temos (a + 2b)t + (b + c) = t2 + 2t + 1. O lado esquerdo dessa equao pode ser escrito, tambm, na forma 0t2 + (a + 2b)t + (b + c), logo 0t2 + (a + 2b)t + (b + c) = t2 + 2t + 1. Temos ento, 0=1 a + 2b = 2 b+c=1 Como esse sistema linear no tem soluo, o vetor dado no pertence imagem de T .

..............................................................................................

5 Transformaes Lineares c) O vetor at2 + bt + c pertence ao ncleo de T se T (at2 + bt + c) = 0, isto , se (a + 2b)t + (b + c) = 0. Ento, a + 2b = 0 b+c =0

84

Aplicando a fatorao LU no sistema linear encontrado, obtemos 2 base N (A) = 1 , 1 logo {2t2 t + 1} uma base para N (T ). d) Como N (T ) no contm apenas o vetor nulo, T no injetora. e) Todo vetor pertencente imagem de T da forma (a + 2b)t + (b + c) de modo que os vetores t e 1 geram I m(T ). Como esses vetores so tambm LI, eles formam uma base para I m(T ). f) dim P2 = 3, enquanto que I m(T ) um subespao de P2 de dimenso 2, logo, I m(T ) = P2 . Portanto, L no sobrejetora. g) De (d), dim N (T ) = 1 e, de (e), dim I m(T ) = 2, de modo que 3 = dim P2 = dim N (T ) + dim I m(T ).

Transformaes e Matrizes
Na continuao deste captulo, vamos estudar a relao entre transformaes lineares e matri-

zes.
Seja V um espao vetorial com dimenso nita e B = {v1 , . . . , vn } uma base de V . Dado v V , existem x1 , . . . , xn R tais que v = x1 v1 + + xn vn , e denotamos por x1 . n . v B= . R xn

onde o smbolo v

a representao de V na base B.

..............................................................................................

5 Transformaes Lineares E XEMPLO 78: Seja B = {(1, 0), (0, 1)} e v = (1, 1) R2 , ento v pois, v = 1(1, 0) + 1(0, 1) = (1, 1) Se B1 = {(1, 1), (1, 0)} ache v
B1 B

85

1 1

. Neste caso v

B1

1 pois 0

v = 1(1, 1) + 0(1, 0) = (1, 1).

Inicialmente veremos como, dados dois espaos vetoriais V e W com bases B1 e B2 respectivamente, e uma matriz A, podemos obter uma transformao linear TA : V W.
E XEMPLO 79: Sejam V, W = R2 e B1 = {(1, 0), (0, 1)} e B2 = {(1, 1), (1, 1)} base de R2 , e a matriz A = 2 0 . 0 1

Queremos associar a esta matriz A uma transformao linear que depende de A e das bases dadas B1 , B2 , isto , TA :R2 R2 v TA (v ) Seja v = (x, y ). Ento, v
B1

x , logo y = 2 0 0 1 x 2x = = TA (v ) y y

A v

B1

B2

Ento TA (v ) = 2x(1, 1) + y (1, 1) = (2x y, 2x + y ). Por exemplo, se v = (2, 1), ento TA (2, 1) = (3, 5).

De maneira geral, dados os espaos vetoriais V e W , com dim V = n e dim W = m, as bases B1 = {v1 , . . . , vn } de V e B2 = {w1 , . . . , wm } de W , e ainda a matriz A Rmn , podemos denir TA :V W v TA (v ) uma transformao linear, dada por: v V onde v = x1 v1 + + xn vn , com x1 , . . . , xn R e, da, x1 . n . v B1 = . R xn

..............................................................................................

5 Transformaes Lineares

86

Dena, y e TA : V W por onde yi = Ai v Observao: TA (v )


B2 B2

=A v

B1

T (v ) = y1 w1 + + ym wm W e Ai a i-sima linha de A.

B1

= y

B2

=A v

B1

Em geral, dada uma matriz Amn , ela vista como uma transformao linear TA : Rn Rm em relao s bases cannicas de Rn e Rm .
E XEMPLO 80: Sejam A= 1 3 2 4 5 , 1 B1 = {(1, 0), (0, 1)}, B2 = {(1, 0, 0), (0, 1, 0), (0, 0, 1)}

e TA : R3 R2 . Encontre esta transformao linear. x Seja v = y , ento z A v


B2

1 3 2 4

Ento

x 5 x 3y + 5z y = 1 2x + 4y z z

TA (v ) = (x 3y + 5z )(1, 0) + (2x + 4y z )(0, 1) = (x 3y + 5z, 2x + 4y z ) E XEMPLO 81: P3 com base B1 = {1, x, x2 , x3 } 1 2 0 1 0 0 2 0

R2 com base B2 = {(1, 0), (0, 1)} A= Encontre TA : P3 R2 . Tome p P3 , tal que p(x) = a0 + a1 x + a2 x2 + a3 x3 . Assim, a0 a 1 = R4 a2 a3

B1

..............................................................................................

5 Transformaes Lineares a0 0 0 a0 + 2 a1 a1 = 2 0 a2 a1 + 2 a2 a3

87

y=A p

B1

1 2 0 1

TA (p) = (a0 + 2a1 )(1, 0)+)(a1 + 2a2 )(0, 1) = (a0 + 2a1 , a1 + 2a2 ) Portanto, TA : P3 R2 .

Observao: No que segue vamos denotar TA apenas por T . Vamos pensar agora no procedimento inverso, ou seja, dados uma base B1 de V , uma base B2 de W e T : V W uma transformao linear, como podemos fazer para encontrar a matriz A que representa esta transformao. Para isso, seja T : V W uma transformao linear, B1 = {v1 , . . . , vn } base de V e B2 = {w1 , . . . , wm } base de W . Ento T (v1 ), . . . , T (vn ) so vetores de W e, portanto T (v1 ) = a11 w1 + + am1 wm . . . T (vn ) = ain w1 + + amn wm Ento, a matriz transposta dos coecientes deste sistema chamada matriz de T em relao s bases B B1 e B2 , denotada por T B1 , ou ainda, 2 a11 . . = . am1 a1n . . . amn

A= T

B1 B2

E XEMPLO 82: Seja T : R3 R2 tal que T (x, y, z ) = (2x + y z, 3x 2y + 4z ), B1 = {(1, 1, 1, ), (1, 1, 0), (1, 0, 0)} e B1 B2 = {(1, 3), (1, 4)}. Encontre T B2 . Vamos calcular T nos elementos da base B2 T (1, 1, 1) = (2, 5) da, T (1, 1, 0) = (3, 1) da, (3, 1) = c3 (1, 3) + c4 (1, 4) c3 = 11 e c4 = 8 T (1, 0, 0) = (2, 3) da, (2, 3) = c5 (1, 3) + c6 (1, 4) c5 = 5 e c6 = 3

(2, 5) = c1 (1, 3) + c2 (1, 4) c1 = 3 e c2 = 1

..............................................................................................

5 Transformaes Lineares Ento T


B1 B2

88

c1 c2

c3 c4

c5 3 11 5 = c6 1 8 3

Observao: Se xarmos outras bases Bi e Bj , teremos uma outra matriz para a transformao T .
E XEMPLO 83: Seja T a mesma transformao linear do exemplo anterior, e sejam B1 = {(1, 0, 0), (0, 1, 0), (0, 0, 1)} e B1 B2 = {(1, 0), (0, 1)}. Calculando T B2 , temos T (1, 0, 0) = (2, 3) = 2(1, 0) + 3(0, 1) T (0, 1, 0) = (1, 2) = 1(1, 0) 2(0, 1) T (0, 0, 1) = (1, 4) = 1(1, 0) + 4(0, 1) Ento T
B1 B2

2 1 1 3 2 4

Observao: Em alguns livros, quando temos uma transformao linear T : Rm Rn em relao s bases cannicas, a matriz de transformao linear simplesmente denotada por T , ou seja, B T B1 = T . 2 Vamos considerar agora a transformao linear T : V V tal que T (v ) = v , isto , T a identidade. Sejam B1 = {v1 , . . . , vn } e B2 = {w1 , . . . , wn } bases de V , ento T (v1 ) = v1 = a11 w1 + + an1 wn . . . T (vn ) = vn = a1n w1 + + ann wn e T
B1 B2

Essa matriz que leva a representao de um elemento v V , da base B1 para a base B2 , chamada matriz de mudana de base e denotada por T Observao: Temos que v = a1 v1 + + an vn V v = b1 v1 + + bn vn V Ento v
B2 B1 B2

a11 . . = . an1

a1n . . . ann

= I

B1 B2

B1

..............................................................................................

5 Transformaes Lineares

89 b1 . . .= I bn a1 B1 . . . B2 an

isto ,

Note que, I

B1 B2

R nn .

E XEMPLO 84: Dadas as bases B1 = {(1, 1), (0, 1)} de R2 e B2 = {(0, 3, 0), (1, 0, 0), (0, 1, 1)} de R3 , encontraremos a transformao linear T : R2 R3 cuja matriz 0 2 B1 T B2 = 1 0 1 3 Temos ento, T (1, 1) = 0(0, 3, 0) 1(1, 0, 0) 1(0, 1, 1) = (1, 1, 1) T (0, 1) = 2(0, 3, 0) + 0(1, 0, 0) + 3(0, 1, 1) = (0, 9, 3) Dado (x, y ) R2 , devemos encontrar T (x, y ), ento (x, y ) = c1 (1, 1) + c2 (0, 1) T (x, y ) = c1 T (1, 1) + c2 (0, 1) vamos achar c1 e c2 : x = c1 y = c1 + c2 logo, c1 = x e c2 = y x Da, T (x, y ) = xT (1, 1) + (y x)T (0, 1) = (x, 9y 10x, 3y 4x) = x(1, 1, 1) + (y x)(0, 9, 3)

O teorema a seguir nos d o signicado da matriz de uma transformao linear. Teorema 5.14: Seja V e W espaos vetoriais com bases B1 = {v1 , . . . , vn } e B2 = {w1 , . . . , wm }, respectivamente. Dada T : V W uma transformao linear, ento para todo v V . T (v )
B2

= T

B1 B2

B1

Demonstrao. Como v V , ento existem d1 , . . . , dn R tais que v = d1 v1 + + dn vn Da, T (v ) = T (d1 v1 + + dn vn ) = d1 T (v1 ) + + dn T (vn ). ..............................................................................................

5 Transformaes Lineares

90

que

Como T (v1 ), . . . , T (vn ) W , ento existem aij R, com i = 1, . . . , m e j = 1, . . . , n tais T (v1 ) = a11 w1 + + am1 wm . . . T (vn ) = a1n w1 + + amn wm

Portanto, T (v ) = d1 (a11 w1 + + am1 wm )+ + d2 (a12 w1 + + am2 wm ) + + + dn (a1n w1 + + amn wm ) ou ainda, T (v ) = (a11 d1 + a12 d2 + + a1n dn )w1 + + (a21 d1 + a12 d2 + + a2n dn )w2 + + + (am1 d1 + am2 d2 + + amn dn )wn Note que v e T (v )
B2

B1

Assim, T (v )

a11 d1 + + a1n dn . Rm . = . am1 d1 + + amn dn a1n d1 . . . . . . =A v amn dn


B1 B2

d1 . n . = . R dn

B2

como A = T

B1 , B2

temos

a11 . = . . am1 T (v )
B2

B1

= T

B1

Observao:

B1 B2

a11 . . . = am1
T (v1 )
B2

T (vn )

a1n . . . amn

B2

Assim, a matriz T que representa uma transformao linear T : V W em relao as bases B1 e B2 tem a i-sima coluna dada por T (vi ) B2 . ..............................................................................................

B1 B2

5 Transformaes Lineares E XEMPLO 85: Seja a transformao linear T : R2 R3 dada por T


B1 B2

91

onde B1 = {(1, 0), (0, 1)} base de R2 e B2 = {(1, 0, 1), (2, 0, 1), (0, 1, 0)} base de R3 . Encontre T (v ), onde v = (2, 3). v = (2, 3) = 2(1, 0) 3(0, 1) = v Pelo Teorema 5.14 T (v ) ento
B2

1 1 = 0 1 2 3

B1

= (2, 3)

B1

2 3

1 1 5 2 = 0 = 3 1 3 2 0 13

T (v ) = 5(1, 0, 1) + (3)(2, 0, 1) + (13)(0, 1, 0) = (11, 13, 2)

Teorema 5.15: Seja T : V W uma transformao linear e B1 uma base de V e B2 uma base de W . Ento: i) dim I m(T ) = posto ii) dim N (T ) = dim N Demonstrao. T T
B1 B2 B1 B2

Teorema 5.16: Sejam T1 : V W e T2 : W U transformaes lineares e B1 , B2 e B3 bases de V , W e U respectivamente. Ento a composta de T1 com T2 , T2 T1 : V U , uma transformao linear e B B B T2 T1 B1 = T2 B2 T1 B1 3 3 2 Demonstrao. Vamos provar que T2 T1 uma transformao linear. Para isso, sejam u, v V e R. Ento, (T2 T1 )(u + v ) = T2 (T1 (u + v )) = T2 (T1 (u) + T2 (v )) = T2 (T1 (u)) + T2 (T1 (v )) e = (T2 T1 )(u) + (T2 T1 )(v ) (T2 T1 )(u) = T2 (T1 (u)) = T2 (T1 (u)) = T2 (T1 (u)) = (T2 T1 )(u) Portanto T2 T1 uma transformao linear.

..............................................................................................

5 Transformaes Lineares E XEMPLO 86: Sejam as transformaes lineares T1 : R2 R3 e T2 : R3 R2 com, 1 0 0 1 1 B1 B2 T1 B2 = 1 1 e T 2 B3 = 0 0 0 0 1

92

sendo B1 = {(1, 0), (0, 2)}, B2 = {( 1 3 , 0, 3), (1, 1, 15), (2, 0, 5)} e B3 = {(2, 0), (1, 1)}. Encontre T 2 T 1 : R2 R2 . Seja R = T2 T1 , assim, pelo Teorema 5.16, R ou seja, R
B1 B3 B1 B3 B2 B3 B1 B2

= T2

T1

0 0

Vamos achar R : R2 R2 . Dado v = (x, y ) R2 , temos que (x, y ) = c1 (1, 0) + c2 (0, 2). Da, R(x, y ) = c1 R(1, 0) + c2 R(0, 2) Mas, R(1, 0) = 1(2, 0) + 0(1, 1) = (2, 0) R(0, 2) = 2(2, 0) + 0(1, 1) = (4, 0) com isso, R(x, y ) = c1 (2, 0) + c2 (4, 0). Precisamos encontrar c1 e c2 : c = x 1 (x, y ) = c1 (1, 0) + c2 (0, 2) 2c2 = y c1 = x c2 =
y 2

1 0 1 1 1 2 1 1 = 0 0 0 0 0 1

Portanto,

y R(x, y ) = x(2, 0) + (4, 0) = (2x 2y, 0) 2

Teorema 5.17: Seja T : V W uma transformao linear invertvel (T um isomorfo) e B1 e B2 bases de V e W . Ento T 1 : W V uma transformao linear e T 1 Demonstrao. A matriz identidade I
B1 B1 B2 B1

B1 1 B2 B1 B1

= T 1 T

= T 1

B2 B1

B1 . B2

..............................................................................................

6
Ortogonalidade
Denio 6.1: Dado x Rn , x = {x1 , . . . , xn } denimos por x = a norma de x (norma euclidiana). Observao: Note que x
2 2 x2 1 + + x2

= xT x = x =

xT x.

Denio 6.2 (A Norma Em Um Espao Vetorial): Dada V um espao vetorial real, dizemos que a funo : V R uma norma em V , se v, w V e R vale que N1) v 0

N2) v = 0 v = 0 N3) v = |v | v N4) v + w


E XEMPLO 87:

v + w

(Desigualdade Triangular).

v1 . . V = Rn . Ento dado v = . , a funo vn v = dene uma norma em Rn .


2 + + v2 v1 n

6 Ortogonalidade E XEMPLO 88: V = Rmn . Dada A Rmn a funo A = T r(AT A)

94

dene uma norma em V , chamada de Norma de Frobenius. Note que A = a2 ij . Notao da norma de Frobenius: A
F.

E XEMPLO 89: Se S simtrica denida positiva (xT Sx > 0, x = 0), ento x = xT Sx dene uma norma em Rn .

Denio 6.3: Se v = 1 dizemos que v est normalizado. Denio 6.4: Dado v V e uma norma em V , ento w= um vetor normalizado. Denio 6.5: Dado v, w Rn , dizemos que v ortogonal a w quando vT w = 0 Neste caso, denotamos v w. Observao: A notao v T w denominada produto interno de v por w, o qual zero se, e somente se, v e w so ortogonais. Denio 6.6 (Produto Interno Em Um Espao Vetorial): Dado V um espao vetorial, dizemos que < , >: V V R um produto interno em V , se v, w, z V e R vale as seguintes propriedades: P1) < v, v > 0 1 v v

P2) < v, v >= 0 v = 0 P3) < v, w >= < v, w > P4) < v + w, z >=< v, z > + < w, z > P5) < v, w >=< w, v > ..............................................................................................

6 Ortogonalidade

95

Observao: Se < , > um produto interno em V ento v V v = dene uma norma em V . Daqui em diante vamos considerar o produto interno de x, y Rn por xT y Rn , e xy se, e somente se, xT y = 0. Teorema 6.7: Sejam v1 , . . . , vn Rn vetores no nulos e ortogonais entre si. Ento {v1 , . . . , vn } LI. Demonstrao. Sejam 1 , . . . , n R tais que c1 v1 + + cn vn = 0 Temos que mostrar que 1 = = n = 0. Como os vetores v1 , . . . , vn so ortogonais entre si, segue T v = 0, para i = j , v T v = 0 para i = j . Ento, multiplicando a equao v + + v = 0 que vi j 1 1 n n i j T por vi , temos T vi (1 v1 + + i vi + + n vn ) = 0 isto ,
T T T 1 v1 v1 + + i vi vi + + n vn vn = 0

< v, v >

da,
T vi = 0 i vi

ou seja, i vi como vi = 0, segue que vi


2 2

=0

= 0 vi = 0, logo i = 0.

Como o ndice i foi escolhido arbitrariamente, segue que 1 = = n = 0 Logo, {v1 , . . . , vn } LI.
E XEMPLO 90: Em Rn , e1 , e2 , . . . , en so todos ortogonais entre si, isto , eT i ej = 0 se i = j .

Denio 6.8: Dados, V, W Rn subespaos vetoriais, dizemos que V e W so ortogonais, se para todo v V e w W temos que v T w = 0. ..............................................................................................

6 Ortogonalidade

96

Notao: V W .
E XEMPLO 91: 1 1 1 0 V = a1 + a2 a1 , a2 R 0 0 0 0 0 0 0 0 W = b1 + b2 b1 , b2 R 1 0 1 1 a1 a2 a 1 v V, ento v = 0 0

Neste caso V W . Tome

Da, v T w = 0. Assim v w, e portanto V e W so subespaos ortogonais.

0 0 w w, ento w = b1 b1 b2

A seguir, veremos a relao de ortogonalidade entre os quatro subespaos fundamentais. Relembrando: A Rmn , ento dim R(A) + dim N (AT ) = m dim R(AT ) + dim N (A) = n Teorema 6.9: Dada a matriz A Rmn , ento i) N (AT )R(A) ii) N (A)R(AT ) Demonstrao. i) Dado y AT e b R(A), ento AT y = 0 e b = Ax para algum x, logo bT y = (Ax)T y = xT AT y = xT 0 = 0. Portanto, N (AT )R(A). ..............................................................................................

6 Ortogonalidade

97

ii) Tome, x N (A), ento Ax = 0, e ainda z R(AT ), ento z = AT y . Logo, z T x = (AT y )x = y T (AT )T x = y T (Ax) = y T 0 = 0. Portanto, N (A)R(AT ).
E XEMPLO 92: 3 6 9 1 base R(A) = 2 3 base R(AT ) = 1 3 1 A = 2 3

base N (A) =

3 1

R(AT )N (A) Note que: R(A)N (AT )

3 2 base N (AT ) = 1 , 0 0 1

Denio 6.10 (Complemento Ortogonal): Seja W um subespao vetorial do espao vetorial V , isto , W V . O complemento ortogonal de W em V , denotado por W (l-se W perp), o conjunto W = {y V |y T x = 0 x W }.
E XEMPLO 93: 1 W = {(1, 2, 1)} e V = {(1, 0, 1), (1, 2 , 0)} Note que V W e dim W + dim V = 3, logo V = W .

Teorema 6.11: Seja W um subespao de Rn . Ento, W um subespao de Rn .


T x = 0 e y T x = 0 para todo x W . Demonstrao. Suponha que y1 e y2 pertencem a W . Ento y1 2 Logo, (y1 + y2 )T x = 0

..............................................................................................

6 Ortogonalidade

98

ou seja, y1 + y2 W . Suponha que y W e seja R. Ento, para qualquer vetor x W , temos: (y T )x = (y T x) = 0 = 0. Teorema 6.12: Se W um subespao de Rn , ento: W W = {0} Demonstrao. Note que W W contm pelo menos o vetor nulo, pois W e W so subespaos de Rn . Mas esse o nico vetor em W W , pois se y W W , ento y W e y W , logo y T y = 0 e, da, y 2 = 0, isto , y = 0. A seguir, temos um teorema que relaciona os quatro subespaos fundamentais com seus complementos ortogonais. Teorema 6.13: Seja A Rmn . Ento i) N (A) = R(AT ) ii) N (AT ) = R(A) Demonstrao. i) Sabemos que N (AT )R(A), ento N (AT ) R(A) . Logo, considere x N (AT ) ento, xR(A), isto , x N (AT ) AT x = 0. Se y R(A), ento y = Az para algum z . Mas y T x = (Az )T x = z T AT x = 0. Por outro lado, considere x R(A) . Ento, devemos que mostrar que AT x = 0. Logo, y R(A), y = Az , temos que xT y = 0 xT Az = 0, z Rn . Tome z = AT x Rn , da, (xT A)(AT x) = 0, isto , (AT x)T (AT x) = 0 Ax 2 = 0, portanto AT x = 0. ii) Para provar que N (A) = R(A) , basta trocar A por AT na parte i). Corolrio 6.14: Seja A Rmn . Ento: i) Rm = R(A) ii) Rn = R(AT ) N (AT ) N (A)

..............................................................................................

6 Ortogonalidade

99

Demonstrao. Para ambos os casos, devemos mostrar que so satisfeitas as condies de soma direta. i) Primeiro mostraremos que Rm = R(A) + N (AT ). Temos que dim Rm = m e, sabemos que dim R(A) = r e dim N (AT ) = m r , logo dim Rm = dim R(A) + dim N (AT ) isto , m = dim Rm = r + m r = m Portanto Rm = R(A) + N (AT ). Falta mostrar que R(A) N (AT ) = {0}. Sejam y R(A) N (AT ), isto , y R(A) e y N (AT ). Mas R(A)N (AT ), ento, yT y = 0 y Portanto, Rm = R(A) N (AT ).
2

= 0 y = 0.

ii) A demonstrao se faz inteiramente semelhante a do item i).

Vamos estudar agora, o caso em que dado um vetor b no espao n-dimensional e um vetor a, tal que a e b no so ortogonais, como podemos encontrar um ponto p em a tal que p o ponto mais prximo de b. Neste caso, a soluo deste problema cai automaticamente em ortogonalidade. Esta idia est representada na gura abaixo.
xn b = (b1 , . . . , b n )

a = (a 1 , . . . , a n ) p x2

x1

Projees
O nosso interesse agora obter o ponto p que chamamos de projeo do vetor b na direo do

vetor a.
Esta idia est representada na gura a seguir.

..............................................................................................

6 Ortogonalidade

100

Note que, p est na direo de a, logo p = a, R

como p o ponto mais prximo de b, ento o vetor b p = b a ortogonal a a, logo aT (b a) = 0 aT b aaT = 0 = Isto nos sugere a seguinte denio. Denio 6.15: A projeo de b por uma linha entre O e a p = a = Note que, p = a = aT b aT a a=a aT b aaT = b aT a aT a aT b a. aT a aT b . aT a

aaT Denindo a matriz P = T , temos que P a projeo ortogonal na direo do vetor a, logo podemos a a escrever p = P b. A matriz P Rnn e tem posto(1), ainda mais P T = P e P T P = P 2 = P , ou seja, P idempotente.
E XEMPLO 94: 1 1 b = 2 e a = 1. Ento: 3 1

1 T b a = 1 2 3 1 = 6 1 1 aT a = 1 1 1 1 = 3 1

..............................................................................................

6 Ortogonalidade 2 logo = 2 e p = a = 2 e a matriz de projeo : 2 P =


1 3 1 3 1 3 1 3 1 3 1 3

101

1 3 1 3 1 3

O Problema de Mnimos Quadrados


O nosso interesse agora, encontrar solues para problemas do tipo min Ax b
2

ou seja, precisamos encontrar x Rn tal que o erro seja o menor possvel (x chamado soluo de mnimos quadrados). Geometricamente, temos a seguinte situao:
b / R(A)

p = Ax

R(A)

Resumidamente, temos
b p = b Ax b

p = Ax

Temos que b p N (A), logo b p = b Ax R(A). Ento, se aij a coluna j de A, j = 1, . . . , n, aT ) = 0 AT (b Ax ) = 0 AT b AT Ax = 0 AT Ax = AT b j (b Ax O sistema AT Ax = AT b chamado sistema das equaes normais, e x a soluo das equaes normais, conseqentemente do problema. ..............................................................................................

6 Ortogonalidade E XEMPLO 95:

102

1 2 4 Dada a matriz A = 1 3, e o vetor b = 5 encontre a soluo de mnimos quadrados e o ponto p 0 0 6 que representa a projeo de b em A. Calculando os seguintes produtos, temos 1 2 1 2 1 0 2 1 3 = 3 0 5 0 0 1 1 2 3 5 13

AT A =

AT b = Ento o sistema AT Ax = AT b ca 2 5

4 0 9 5 = 0 23 6

x 1 9 = x 2 23 4 2 e a soluo x 1 = 2 e x 2 = 1, isto x = e p = Ax = 5 1 0

5 13

Se a matriz A tem posto(A) = n, ento N (A) = {0}. Mas, N (AT A) = N (A) = {0}, ento AT A invertvel. Portanto, no caso que A tem posto completo, x nico e x = (AT A)1 AT b e, da, p = Ax = A(AT A)1 AT b Ento, a matriz P Rmm onde P = A(AT A)1 AT , a matriz que projeta ortogonalmente em R(A). Assim, p = Pb e ainda, b p = b P b = (I P )b logo (I P ) projeta ortogonalmente em N (AT ). Portanto,
(I P )b N (A T ) b Rm

P b R(A)

..............................................................................................

6 Ortogonalidade

103

e, da, b = P b + (I P )b
R(A) N (AT )

logo Rm = R(A)

N (AT ).

Regresso Linear
Suponha que tenhamos obtido n pontos (ti , yi ), i = 1, . . . , n, em geral no-colineares, ento, estamos interessados em obter uma reta que melhor se ajuste esses pontos. Geomtricamente temos
y

considere os pontos t t1 t2 t3 . . . tn y y1 y2 y3 . . . yn

Como poderiamos ajustar uma reta pelos pontos (t1 , y1 ), (t2 , y2 ), . . . , (tn , yn ) R2 ? Note que, Se (t1 , y1 ) pertence a reta y (t) ento y1 = at1 + b; Se (t2 , y2 ) pertence a reta y (t) ento y2 = at2 + b; . . . Se (tn , yn ) pertence a reta y (t) ento yn = atn + b; ..............................................................................................

6 Ortogonalidade

104 y1 t1 1 y2 t2 1 a . = . . . . . b . . . yn tn 1 x
C A

ou seja,

logo C / R(A). Da, as equaes normais cam n t1 1 t2 i t2 1 tn . . = i=1 n 1 . . . . ti tn 1 i=1


n i=1

t1 t2 A A= 1 1
T

AT C =

t1 t2 1 1

logo,
n

n y1 bi yi y tn .2 = i=1 n 1 . . yi yn i=1
n

ti n

i=1 n ti
i=1

t2 i

i=1

AT Ax = AT C n

ti bi yi a i =1 = n b n y
i i=1

Matrizes Ortogonais e Gram-Schmidt


Denio 6.16: Dizemos que um conjunto {q1 , q2 , . . . , qr } Rn ortonormal se 0 se i = j T qi qj = 1 se i = j
T q = 0. Observao: O conjunto dito ortogonal se qi i

E XEMPLO 96: 1 1 q1 = e q2 = so ortogonais. 1 1 1 2 q1 = 0 e q2 = 0 so ortonormais.


1 2 1 2 1 2

E XEMPLO 97:

..............................................................................................

6 Ortogonalidade

105

Observao: Dado um conjunto {q1 , . . . , qk } ortogonal podemos obter um conjunto ortonormal. {q 1 , . . . , q k } onde q 1 = qi qi

Proposio 6.17: Se {qi , q2 , . . . , qk } ortonormal, ento b = 1 q1 + 2 q2 + + k qk resulta em


T 1 = qi b,

i = i, . . . , k.

Demonstrao.

T T qi b = qi

k j =1

T j qj = i qi qi = i

Denio 6.18: Dizemos que Q Rnn ortogonal se QT Q = QQT = I Note que neste caso Q1 = QT . Observao: i) As colunas de Q Rnn formam uma base ortonormal para Rn . QT Q = I mas QQT = I. iii) Se Q Rnn com colunas ortonormais, ento Q admite inversa esquerda, isto , QT Q = I . Mas Q quadrada, ento Q tambm admite inversa direita, isto QQT = I . Logo, QT 1 QT 2 Q= . . . QT n QT Q = 0 se i = j j i QT Q = 1 se i = j j i

ii) Se Q Rmn , com m > n, e colunas ortonormais. Ento

Portanto as linhas de Q tambm formam um conjunto ortonormal. ..............................................................................................

onde Qi a i-sima linha de Q. Da, QT 1 | | QT 2 Q Q In = . 1 2 | . | . QT n

Qn
|

6 Ortogonalidade

106

Concluso: Q Rnn tem colunas ortonormais se, e somente se, tem linhas ortonormais. Teorema 6.19 (Matriz Ortogonal Preserva Norma): Se Q Rnn uma matriz ortogonal, ento Qx = x , Demonstrao. Temos que Qx Logo, Qx isto , Qx = x . Corolrio 6.20 (Matriz Ortogonal Preserva Produto Interno): Se Q Rnn uma matriz ortogonal, ento < Qx, Qy >=< x, y >, x, y Rn . isto , (Qx)T (Qy ) = xT y . Demonstrao. Temos que < Qx, Qy >= (Qx)T (Qy ) logo, < Qx, Qy >= (Qx)T (Qy ) = xT QT Qy = xT Iy = xT y =< x, y > . Observao: Tambm, se Q Rnn uma matriz ortogonal, ento Q preserva o ngulo entre vetores, isto , dados x, y Rn e o ngulo entre eles, temos que cos = xT y x y e < x, y >= xT y
2 2

x Rn .

= (Qx)T (Qx) = xT QT Qx = xT Ix = xT x = x

= x

o ngulo entre Qx e Qy , ento e ainda, de Qx = x e Qy = y e sendo = cos (Qx)T (Qy ) xT y = = cos . Qx Qy x y

..............................................................................................

7
Determinantes
Inicialmente, vamos denir o determinante de matrizes apenas para os casos 1 1 e 2 2 e o denotaremos o determinante de uma matriz A por det(A). Caso 1 1 : Para cada matriz A R11 , isto , A = [a11 ], o determinante de A o prprio elemento a11 , ou seja, det(A) = a11 . Caso 2 2 : Para matrizes de ordem n = 2, isto , A R22 , denimos o det(A) como sendo o produto dos elementos da diagonal principal menos o produto dos elementos da diagonal secundria, isto 11 a12 , dada a matriz A = [ a a21 a22 ], temos que: det(A) = a11 a12 = a11 a22 a12 a21 a21 a22 +

Quando escrevermos o determinante em forma de matriz, escreveremos a matriz apenas por barras verticais, isto , |A|, como no caso acima. Observao: Vamos considerar conhecido tambm a denio de determinante para matrizes 3 3, isto , A R33 , mas o caso que nos interessa para determinantes de ordem n = 2. Caso 3 3 : Dada uma matriz a11 a12 a13 A = a21 a22 a23 a31 a32 a33

denimos o determinante de A por:

7 Determinantes

108

a11 a12 det(a) = a21 a22 a31 a32

a13 a23 a33 + + +

= a11 a22 a33 + a12 a23 a31 + a12 a21 a32 a13 a22 a31 a11 a23 a32 a12 a21 a33 No que segue, vamos apresentar uma longa lista com as propriedades sobre determinantes, estas propriedades so de fcil compreenso, e algumas delas ns iremos mostrar apenas para o caso 2 2, mas mesmo estas so aplicveis para qualquer matriz n n.

Propriedades:
D1) O determinante depende linearmente da primeira linha. i) a + a b + b a b a b = + c d c d c d ta tb a b =t c d c d

ii)

Demonstrao. a + a b + b a b a b + = (a + a )d (b + b )c = (ad bc) + (a d b c) = c d c d c d ta tb a b = tad tbc = t(ad bc) = t c d c d D2) Se trocarmos duas linhas da matriz A, o sinal do determinante trocado. Demonstrao. a b = ad bc c d e a b c d = cb ad = (ad bc) = c d a b ..............................................................................................

i)

ii)

7 Determinantes

109

D3) O determinante da matriz identidade 1, isto , det(I ) = 1. D4) Se A tem duas linhas iguais, ento det(A) = 0. Demonstrao. Vamos mostrar esta propriedade de dois modos: i) Pelo clculo usual de determinantes, temos A= ii) Usando a propriedade (D2): = a b = det(A ) = det(A), A a b ) = det(A) det(A) = 0. mas det(A) = det(A a b a b = = (ab ab) = 0 a b a b

D5) Operaes elementares de subtrair de uma linha i um mltiplo R de uma outra linha j no altera o determinante (lj lj li ). Demonstrao. A= a b c d l1 l1 l2

= a c b d A c d logo, ) = det(A a c b d c d
(D1)

= =

a b c d c d c d a b 0 c d

(D4)

= det(A). Vamos demonstrar esta propriedade para o caso geral tambm. Seja a11 a12 . . . . . . ai1 ai2 A= a j 1 aj 2 . . . . . . an1 an2 a1n . . . ain i ajn j . . . ann

..............................................................................................

7 Determinantes

110

Considere a operao elementar li li lj e chamando esta nova matriz de B , obtemos a11 a12 a1n . . . . . . . . . ai1 aj 1 ai2 aj 2 ain ajn i B= j aj 1 aj 2 ajn . . . . . . . . . an1 an2 ann trocando a linha i com a linha 1, camos com ai1 aj 1 ai2 aj 2 . . . . . . a11 a12 = A aj 1 aj 2 . . . . . . an1 an2 , camos com ento, aplicando (D1) em A ai1 . . . ) = det(A a11 aj 1 . . . ai2 . . . ain . . . ain ajn . . . i a1n j ajn . . . ann aj 1 . . . aj 2 . . . ajn . . . a1n ajn . . . ann

a12 aj 2 . . .

an1 an2 e da, pelas propriedades (D4) e (D2), temos

a1n a11 a12 ajn aj 1 aj 2 . . . . . . . . . an1 an2 ann

) = ( det(A)) = det(A). det(B ) = det(A D6) Se uma matriz A Rnn tem uma linha nula, ento det(A) = 0. Demonstrao. 0 0 00 00 = c d c d Para o caso geral, temos:
(D1)

= 0

0 0 = 0. c d

..............................................................................................

a11 . . . A = 0 . . . an1

a1n . . . 0 . . . ann

7 Determinantes

111

e a11 . . . det(A) = 0 . . . a1n . . . 0 . . . ann 0 . . . 0 . . . a1n . . . ann D7) Se A uma matriz triangular superior diagonal, isto , a11 0 a22 A= . .. . . . . . . 0 0 Demonstrao. Suponhamos que aii = 0, para i = 1, . . . , n, onde aii so os pivs de A. Assim, pelo processo de Gauss-Jordan, podemos transformar a matriz A em uma matriz diagonal D usando as operaes elementares, de modo que a11 0 . . .. . . D= . . . 0 ann e, pela propriedade (D5) o det(A) = det(B ). Da, note que a11 0 0 a22 det(D) = . . .. . . . . . 0 0 0 0 . . . ann 0 0 0 ann 1 0 0 a22 . . .. . . . . . 0 0
(D1)

(D2)

= a11 . . . an1

(D1)

= 0 = 0.

an1

(ou inferior), ento det(A) o produto dos elementos da . . . ann

= det(A) = a11 a22 ann

(D1)

= a11

0 0 0 ann 0 1 1 0 . . .. . . . . . 0 0 0 0 0 ann

0 a22 1 0 (D2) = a11 (1) . . .. . . . . . 0 0

= a11 (1)a22

1 0 0 1 (D2) = a11 (1)a22 (1) . . . . .. . . . 0 0

0 0 0 ann

..............................................................................................

7 Determinantes

112

e continuando dessa forma aplicando (D1) e (D2), temos: det(D) = a11 a22 ann det(I ) e, da, por (D3) det(D) = a11 a22 ann . No caso onde algum aii = 0, i = 1, . . . , n, no processo de Gauss-Jordan aparecer uma linha nula, logo, por (D6), temos que det(A) = a11 a22 aii ann = 0
0

D8) i) Se A singular, ento det(A) = 0.

ii) Se A invertvel, ento det(A) = 0. Demonstrao. i) Como A singular, ento no processo da eliminao gaussiana (operaes elementares do tipo li li lj e li lj ) ir aparecer um linha nula. Ento, como no se altera o valor absoluto do determinante, temos que | det(A) = 0|, ou seja, det(A) = 0. ii) Pela eliminao gaussiana, podemos transformar A em U , onde U triangular superior. Assim por (D5), tem-se que det(A) = det(U ), devido as trocas de linhas se existirem, mas por (D7), det(U ) = u11 u22 unn . Portanto, det(A) = u11 u22 unn = 0. D9) Sejam A, B Rnn . Ento det(AB ) = det(A) det(B ). Demonstrao. d11 . .. . Seja D = . . 0 0 . . . dnn b11 b diagonal e B = 21 . . . b12 b22 . . . .. . b1n b2n . . . . bnn

bn1 bn2

Vamos mostrar que det(DB ) = det(D) det(B ).

..............................................................................................

7 Determinantes

113 d11 b1n d22 b2n . . .

Note que

Ento, por (D1) e (D2), temos que

d11 b11 d22 b21 DB = . . .

d11 b12 d22 b22 . . .

.. .

dnn bn1 dnn bn2 b12 b22 . .. . . .

dnn bnn

b11 b21 det(DB ) = d11 d22 dnn . . .

b1n b2n = det(D) det(B ). . . . bnn

bn1 bn2

Seja A Rnn qualquer e suponha que A tenha um conjunto completo de pivs. Ento, atravs de operaes elementares podemos transformar A em D, onde D uma matriz diagonal. Assim, det(A) = det(D) = d11 d22 dnn Aplicando as mesmas operaes elementares e trocas de linhas em AB transformamos AB em DB e, neste caso, det(AB ) = det(DB ) mas det(DB ) = det(D) det(B ) assim, det(AB ) = det D det B ou seja, det(AB ) = det(A) det(B ). No caso onde A no tem um conjunto completo de pivs temos que, A singular, e ento det(A) = 0. Mas neste caso, AB tambm singular, o que resulta em det(AB ) = 0. Portanto, ainda vale que det(AB ) = det(A) det(B ). D10) A transposta da matriz A tem o mesmo determinante de A, isto , det(A) = det(AT ). Demonstrao. Se A singular, ento AT tambm singular. Portanto, para este caso det(A) = 0 = det(AT ).

..............................................................................................

7 Determinantes

114

Vamos supor A no-singular (invertvel). Ento, existem as matrizes: P matriz de permutao, U triangular superior com 1s na diagonal, D matriz diagonal, L triangular inferior com 1s na diagonal, tal que P A = LDU . Ento det(P A) = det(LDU ) det(P ) det(A) = det(L) det(D) det(U ). Tambm, como AT P T = U T DT LT , temos que det(AT ) det(P T ) = det(U T ) det(DT ) det(LT ). Mas, por (D7), det(L) = det(U ) = det(LT ) = det(U T ) = 1, e como D diagonal, det(D) = det(DT ). Conseqentemente, det(P ) det(A) = det(D) det(AT ) det(P T ) = det(DT ) (1) (2)

Como P e P T so matrizes de permutao que fazem o mesmo nmero de trocas de linhas na matriz identidade, temos que det(P ) = 1 = det(P T ) Assim, segue diretamente de (1) e (2) que det(A) = det(AT ).

Expanso em Cofatores
Agora vamos desenvolver um procedimento para calcular determinantes de ordem maior que trs, que baseado na idia de expressar o determinante em termos de determinantes de ordem menor. Considere a matriz A = [aij ]nn , onde a11 a12 a21 a22 A= . . . . . . a1n a2n R nn . . .

.. .

(7.1)

an1 an2

ann

Denio 7.1: Para i, j {1, . . . , n} denimos a matriz Mij R(n1)(n1) como sendo a matriz formada pelos elementos de A, excluindo-se a linha i e a coluna j de A.
E XEMPLO 98: Considerando a matriz A em (7.1), temos: i) M11 retira a primeira linha e a primeira coluna de A, isto , a22 a23 a2n a32 a33 a3n M11 = . . . .. . . . . . . . an2 an3 ann

..............................................................................................

7 Determinantes ii) M21 retira a segunda linha e a primeira coluna de A, isto , a12 a13 a1n a32 a33 a3n M21 = . . . .. . . . . . . . an2 an2 ann

115

Denio 7.2: O determinante da matriz Mij denominado de determinante menor de aij , ou simplesmente o menor de aij . Denio 7.3: Denimos o cofator Aij de aij por Aij = (1)i+j det(Mij )
E XEMPLO 99: 3 1 Seja A = 2 5 1 4 4 6 8 3 1 det(M11 ) = 2 5 1 4 4 5 6 = 16 6 = 4 8 8

i) O menor da entrada a11

e o cofator correspondente :

A11 = (1)1+1 det(M11 ) = det(M11 ) = 16 ii) O menor da entrada a32 : 3 1 det(M32 ) = 2 5 1 4 4 3 6 = 2 8 4 = 26 6

e o cofator correspondente :

A32 = (1)3+2 det(M32 ) = det(M32 ) = 26

Denio 7.4: Os cofatores da primeira linha de A so A11 , A12 , . . . , A1n e os cofatores da i-sima linha de A so Ai1 , Ai2 , . . . , Ain .

..............................................................................................

7 Determinantes

116

Observao: Note que um menor e seu cofator associado ou so iguais ou diferem um do outro por um sinal negativo. Este sinal que dado por (1)i+j segue um padro como o de um tabuleiro de xadrez: + + + + + + + + + + + + + + + + + + + + + 33 + + + + + 44 . . . . . .. . . . . . . . . . . .
nn

Denio 7.5: O determinante de uma matriz A = [aij ]nn a combinao linear da primeira linha de A com os cofatores da primeira linha, ou seja det(A) = a11 A11 + a12 A12 + + a1n A1n . (frmula recursiva para calcular o determinante de uma matriz n n).
E XEMPLO 100: Vamos calcular o determinante de 3 6 9

(7.2)

ento,

1 2 A = 4 5 7 8

A11 = (1)1+1 det(M11 ) = 93 A12 = (1)1+2 det(M12 ) = 78 A13 = (1)1+3 det(M13 ) = 3 logo, det(A) = 1 (93) + 2 (78) + 3 (3) = 240.

Observao: Para calcular o determinate de Ann , reduz-se recursivamente pela equao (7.2) at o caso 2 2, mas supondo conhecido o clculo de determinantes para o caso 3 3, podemos reduzir at este caso. Para valer as dez propriedades vistas anteriormente, com relao a Denio 7.5, basta provar as trs primeiras propriedades, isto , D1, D2 e D3. Vamos a provar a propriedade D1 O determinante depende linearmente da primeira linha. a11 + tb1 a12 + tb2 a1n + tbn a11 a12 a1n a22 a2n a21 a21 a22 a2n . Seja A = . . . . . .. .. eA= . . . . . . . . . . . . . . . an1 an2 ann an1 an2 ann

..............................................................................................

7 Determinantes

117

Ento, det(A) = (a11 + tb1 )A11 + (a12 + tb2 )A12 + + (a1n + tbn )A1n mas, A11 = A11 , A12 = A12 , . . . , A1n = A1n . Assim, det(A) = (a11 + tb1 )A11 + (a12 + tb2 )A12 + + (a1n + tbn )A1n = (a11 A11 + tb1 A11 ) + + (a1n A1n + tbn A1n ) = (a11 A11 + + a1n A1n ) + (tb1 A11 + + tbn A1n ) = det(A) + det(B ). onde, b1 a21 B= . . . b2 a22 . .. . . . bn a2n . . . .

an1 an2

ann

Note que,

B11 = A11 , B12 = A12 , . . . , B1n = A1n ento, (a11 + tb1 ) a21 . . . an1 (a1n + tbn ) a11 a2n a21 = . . . . . . ann a1n b1 a2n a + t 21 . . . . . . an1 bn a2n . . .

an1

ann

ann

o que prova a linearidade do determinante com relao a primeira linha.

Vamos provar a propriedade D2 Trocar duas linhas de A troca o sinal do determinante. Provaremos para o caso onde troca-se a linha i com a linha j , onde i, j = 1. a11 a12 a1n a11 a12 a1n . . . . . . . . . . . . . . . . . . ai1 ai2 ain aj 1 aj 2 ajn . Sejam A = e A = a aj 1 aj 2 ajn i1 ai2 ain . . . . . . . . . . . . . . . . . . an1 an2 ann an1 an2 ann

..............................................................................................

7 Determinantes

118

Note que det(A) = a11 A11 + a12 A12 + + a1n A1n , onde A1k = (1)1+k det(M1k ) e a21 a2(k1) a2(k+1) a2n . . . . . . . . . . . . ai1 ai(k1) ai(k+1) ain M1k = aj (k1) aj (k+1) ajn aj 1 . . . . . . . . . . . . an1 an (k 1) an(k+1) ann (n1)(n1) Mas, det(A) = a11 A11 + a12 A12 + + a1n A1n , onde A1k a2(k1) a2(k+1) a21 . . . . . . . . . aj 1 aj (k1) aj (k+1) M1k = a ai(k1) ai(k+1) i1 . . . . . . . . . an1 an (k 1) an(k+1) = (1)1+k det(M1k ) e a2n . . . ajn ain . . . ann (n1)(n1)

Note que M1k a matriz M1k trocando-se a linha i com a linha a j . Assim, demonstrando por induo, facilmente prova-se para o caso n = 2 e supondo que D2 vale para o caso (n 1) tem-se que, pelo fato de M1k ter ordem (n 1) o det(M1k = det(M1k ). Portanto, A1k = (1)1+k det(M1k ) = (1)1+k det(M1k ) conseqentemente, = A1k det(A) = a11 A11 + a12 A12 + + a1n A1n = a11 (A11 ) + a12 (A12 ) + + a1n (A1n ) = (a11 A11 + + a1n A1n ) = det(A). Portanto, provamos a propriedade D2 para o caso onde i, j = 1. Vamos provar agora a propriedade D3 det(I ) = 1. 1 0 0 0 1 0 Seja A = (Identidade). . .. . . . . . . . . . 0 0 1
nn

= (1)1+k ( det(M1k ))

Vamos mostrar por induo:

..............................................................................................

7 Determinantes

119

n=1 A = [1] det(A) = 1. n=2 A = 1 0 det(A) = 1 0 = 1. 0 1

Supondo que I R(n1)(n1) a identidade de ordem (n 1), isto , 1 0 . . . .. . . In1 = . . 0 1 ento det(In1 ) = 1 vamos provar ento que det(I ) = 1, onde I Rnn , det(I ) = 1 I11 + 0 I12 + + 0 I1n = 1 I11 = I11 mas, 1 0 0 1 = (1)1+1 . .. . . . . . . 0 0 = (1) det(In1 )
2

I11

0 0 . . . 1

(n1)(n1)

mas pela hiptese de Induo, det(In1 ) = 1. Assim, det(I ) = 1 1 = 1.

Teorema 7.6: Seja A Rn com n 2. Ento, det(A) pode ser expresso como expanso dos cofatores de A em relao a qualquer linha ou qualquer coluna, isto , xada uma linha i det(A) = ai1 Ai1 + ai2 Ai2 + + ain Ain ou xada uma coluna j det(A) = a1j A1j + a2j A2j + + anj Anj Demonstrao. Vamos provar a primeira parte para o caso onde n = 3. a11 a12 a13 Seja A = a21 a22 a23 ento, pela Denio 7.5 a31 a32 a33 det(A) = a11 (1)1+1 a22 a23 a21 a23 a21 a22 + a12 (1)1+2 + a13 (1)1+3 a32 a33 a31 a33 a31 a32

..............................................................................................

7 Determinantes

120

Vamos mostrar que podemos expandir com relao a linha 3, isto , vamos mostrar que: det(A) = a31 (1)3+1 Considere a12 a13 a11 a13 a11 a12 + a32 (1)3+2 + a33 (1)3+3 a22 a23 a21 a23 a21 a22 a31 a32 a33 A = a21 a22 a23 a11 a12 a13 (7.3)

por D2), det(A) = det(A), mas pela Denio 7.5, det(A) = a31 (1)1+1
D2)

a22 a23 a21 a23 a21 a22 + a32 (1)1+2 + a33 (1)3+3 a12 a13 a11 a13 a11 a12 a12 a23 a11 a13 a11 a12 + a32 (1)1+2 (1) + a33 (1)3+3 (1) a22 a23 a21 a23 a21 a22

= a31 (1)1+1 (1)

como det(A) = (1) det(A), temos que


det(A) = a31 (1)1+1 (1)2 = a31 (1)1+1 a12 a22 a12 a22 a23 a11 + a32 (1)1+2 (1)2 a23 a21 a13 a11 + a33 (1)3+3 (1)2 a23 a21 a12 a22 a12 a22

a23 a11 + a32 (1)1+2 a23 a21

a13 a11 + a33 (1)3+3 a23 a21

que igual a frmula da equao (7.3). Para prova o segundo caso usamos a propriedade D10, ou seja, det(A) = det(AT ).

Fatorao LDU
Este o mtodo mais prtico para calcular o determinante de uma matriz de ordem n Denio 7.7: Se A no-singular, ento A = P 1 LDU e det(A) = det(P 1 ) det(L) det(D) det(U ) = d11 d22 . . . dnn
n

4.

=
i=1

dii

onde dii so os pivs. O sinal 1 aparece do det(P 1 ), pois P uma matriz de permutao, ou seja, faz trocas de linha e como L e U so triangulares inferior e superior respectivamente, com 1s na diagonal, segue que det(L) = det(U ) = 1. ..............................................................................................

7 Determinantes

121

Vamos mostra o caso 2 2. Seja A= pela fatorao LDU, temos a b 1 0 = c c d a 1 a 0 0


adbc a b 1 a 0 1

a b c d

e o det(A) = ad bc, que o produto dos pivs. Agora, trocando as linhas, temos PA = c d 1 0 = a a b c 1 c 0 0
cbda c

1 d c 0 1

e o det(P A) = cb da = (ad bc) = det(A).

Determinantes e a Inversa de Uma Matriz


Pelo Teorema 7.6, temos que det(A) = a11 A11 + a12 A12 + + a1n A1n det(A) = a21 A21 + a22 A22 + + a2n A2n . . . det(A) = an1 An1 + an2 An2 + + ann Ann logo, podemos reescrever o sistema acima como: a11 a12 a1n A11 A21 a21 a22 a2n A12 A22 . . . . . .. .. . . . . . . . . . . . . an1 an2 ann A1n A2n An1 det(A) An2 det(A) = . . . .. . . . . . . . Ann . . . det(A)

()

onde a primeira matriz matriz A Rnn , a segunda matriz a matriz dos cofatores transposta n n e a matriz aps a igualdade a matriz o qual estamos interessados em descobrir seu signicado. Assim, analisando o produto das matrizes da equao () podemos fazer a seguinte pergunta: qual o valor de ai1 Ak1 + ai2 Ak2 + + ain Akn (7.4) onde i = k, pois para i = k sabemos que (7.4) igual ao det(A). Sabendo a resposta desta pergunta encontraremos os valores de na matriz acima e obteremos um mtodo para encontrar a inversa de A. Teorema 7.8: Se A = [aij ] uma matriz n n, ento ai1 Ak1 + ai2 Ak2 + + ain Akn = 0 (7.5)

..............................................................................................

7 Determinantes

122

para i = k e, a1j A1k + a2j A2k + + anj Ank = 0 para j = k. Demonstrao. Provaremos apenas o primeiro caso pois o segundo segue do primeiro com a propriedade D 10). Considere a matriz B , onde B obtida de A colocando-se no lugar da k-sima linha de A a sua i-sima linha, isto , a11 a12 a1n . . . . . . . . . ai1 ai2 ain i B= a i1 ai2 ain k . . . . . . . . . an1 an2 ann 0 = det(B ) = ai1 Bk1 + ai2 Bk2 + + ain Bkn mas, Bk1 = Ak1 , logo, 0 = ai1 Ak1 + ai2 Ak2 + + ain Akn . Voltando ao nosso sistema em (), podemos reescreve-lo ento, como a11 a12 a1n A11 A21 An1 det(A) 0 det(A) a21 a22 a2n A12 A22 An2 0 . . = . . . . . . . . .. . . .. . . .. . . . . . . . . . . . . . an1 an2 ann A1n A2n Ann 0 0 Bk2 = Ak2 , ..., Bkn = Akn (7.6)

Ento, pela propriedade D 4), det(B ) = 0. Expandindo o det(B ) em relao k-sima linha, temos que

0 0 . . . det(A)

Denio 7.9: A matriz n n cujo elemento (i, j ) o cofator Aji de aji , isto , a matriz dos cofatores transposta denominada a matriz adjunta de A e denotada por Adj(A), ou seja, A11 A12 Adj(A) = . . . A21 A22 . . . .. . An1 An2 . . . . Ann

A1n A2n

..............................................................................................

7 Determinantes

123

Ento, o nosso sistema acima resume-se a: A Adj(A) = det(A) In ou Adj(A) A = det(A) In de onde obtemos A logo, obtemos o seguinte teorema Teorema 7.10: Se A Rnn e det(A) = 0, ento A1 = Demonstrao. A 1 1 1 Adj(A) = [A Adj(A)] = det(A) In = In det(A) det(A) det(A) 1 Adj(A). det(A) 1 Adj(A) = In det(A)

Regra de Cramer
Na segunda aplicao podemos usar o resultado do Teorema 7.10 para encontrar um outro mtodo para resolver um sistema linear onde A Rn n invertvel e b Rn . 1 Assim, a soluo de Ax = b x = A1 b de onde obtemos que x = Adj(A)b, isto , det(A) x1 A11 A21 An1 b1 A A22 An2 b2 x2 1 . = 12 . . . .. . det(A) . . . . . . . . . . . xn A1n A2n Ann bn Note que, x1 = mas, b1 A11 + b2 A21 + + bn An1 = det(B1 ), onde, b1 b2 B1 = . . . a12 a22 . . . a13 a23 . . . a1n a2n . . . ann 1 [b1 A11 + b2 A21 + + bn An1 ] det(A)

bn an2 an3

..............................................................................................

7 Determinantes

124

Portanto, x1 = De maneira geral, temos Teorema 7.11 (Regra de Cramer): Se Ax = b um sistema linear de n equaes a n incgnitas, e det(A) = 0, ento o sistema tem uma nica soluo dada por xj = det(Bj ) det(A) det(B1 ) . det(A)

onde Bj a matriz obtida de A trocando-se a sua j -sima coluna por b. Demonstrao. Como det(A) deve ser diferente de zero, podemos escrever a soluo Ax = b como A11 A21 An1 b1 A A22 An2 b2 1 1 . .12 Adj(A)b = x = A1 b = . . .. . . det(A) det(A) . . . . . . . A1n A2n Ann bn Assim, a entrada na j -sima coluna de x xj = b1 A1j + b2 A2j + + bn Anj det(A)

onde b1 , b2 , . . . , bn so as entradas de b. Os cofatores nessa expresso vm da j -sima coluna de A e, portanto, permanecem inalterados se trocamos a j -sima coluna de A por b. Logo, Como essa substituio fornece a matriz Bj , o numerador b1 A1j + b2 A2j + + bn Anj pode ser interpretado como a expanso em cofatores com relao a j -sima coluna de Bj . Assim xj = det(Bj ) . det(A)

Frmula Para o Piv


Se A Rnn tal que a pode ser fatorada dividir a matriz em blocos de de modo que k a11 k . . . A = ak1 . . . an1 em A = LDU sem troca de linhas, ento podemos a1n . . . akn . . . ann

a1k . . . akk . . .

ank

..............................................................................................

7 Determinantes

125

e da, A = LDU ca
k k k k L A 0 D 0 k k k k k k k Uk . . . . . . . . = . . . . . . . . . . . . . . . . . . . . . . . 0 0 Lk Dk 0 Uk = . . . . . . . . . . . . . . . . 0 Lk Dk Uk = . . . . . . . . . . . .

Ento,

Ak = (Lk Dk Uk ) Rkk de onde obtemos det(Ak ) = det(Lk ) det(Dk ) det(Uk ) = det(Dk ) mas, d1 d2

logo,

Dk = 0

0 .. . dk

det(Ak ) = d1 d2 . . . dk . Note que se a1n a2n . . .

ento A1 = a11 , A2 =

a11 a12 , .... a21 a22

a11 a21 A= . . .

a12 a22 . . .

.. .

an1 an2

ann

Portanto, det(A1 ) = d1 = a11 gera o primeiro piv. Para det(A2 ) = d1 d2 se det(A1 ) = 0, temos det(A2 ) d11 d2 = = d2 det(A1 ) d1

que nos fornece o segundo piv. De maneira geral, se det(Ak1 ) = 0 temos que det(Ak ) d1 d2 . . . dk = = dk det(Ak1 ) d1 d2 . . . dk1 ..............................................................................................

7 Determinantes

126

ou seja, dk = det(Ak ) det(Ak1 ) Frmula para o k-simo piv de A

..............................................................................................

A
Notao Sigma ou Notao Somatrio
Denio A.1: Seja fk , uma funo de k com m, n Z e m
n

n, ento

fk = fm + fm+1 + . . . + fn
k =m

(A.1)

a soma das parcelas resultantes da substituio de sucessivos nmeros inteiros no lugar de k, com k comeando em m e terminando em n. A letra k o ndice do somatrio e, as letras m e n representam os limites inferiores e superiores do somatrio, respectivamente. Observao: Podemos substituir o ndice do somatrio por qualquer outra letra, no necessrio utilizar a letra k, por exemplo:
n n n

fi =
i=1 j =1

fj =
k =1

fk .

Normalmente, as letras mais utilizadas so: i, j e k.


E XEMPLO 101:
n

A notao
k=1

rk ak representa o mesmo que: r1 a1 + r2 a2 + . . . + rn an .

Teorema A.2 (Propriedades da Notao Somatrio):


n n n

i)
k =1

(fk + gk ) =
k =1

fk +
k =1

gk

A Notao Sigma ou Notao Somatrio


n n

128

ii)
k =1 n

fk a = a

fk
k =1 n n

iii)
k =1 m

(fk + gk )rk =
k =1 n n m

fk rk +
k =1

gk rk

iv)
j =1 k =1

fjk =
k =1 j =1

fjk

Demonstrao: i)
n

(fk + gk ) = (f1 + g1 ) + (f2 + g2 ) + . . . + (fn + gn ) =


k =1

= (f1 + f2 + . . . + fn ) + (g1 + g2 + . . . + gn ) =
n n

=
k =1

fk +
k =1

gk

ii)

fk a = f1 a + f2 a + . . . + fn a = a(f1 + f2 + . . . + fn ) = a
k =1 k =1

fk

iii)
n

(fk + gk )rk = (f1 + g1 )r1 + (f2 + g2 )r2 + . . . + (fn + gn )rn =


k =1

= (f1 r1 + g1 r1 ) + (f2 r2 + g2 r2 ) + . . . + (fn rn + gn rn ) = = (f1 r1 + f2 r2 + . . . + fn rn ) + (g1 r1 + g2 r2 + . . . + gn rn ) =


n n

=
k =1

fk rk +
k =1

gk rk

iv)
m n m

fjk =
j =1 k =1 j =1

(fj 1 + fj 2 + . . . + fjn ) =

= (f11 + f12 + . . . + f1n ) + (f21 + f22 + . . . + f2n ) + . . . . . . + (fm1 + fm2 + . . . + fmn ) = = (f11 + f21 + . . . + fm1 ) + (f12 + f22 + . . . + fm2 ) + . . . . . . + (f1m + f2m + . . . + fmn ) =
n n m

=
k =1

(f1k + f2k + . . . + fnk ) =


k =1 j =1

fjk

..............................................................................................

A Notao Sigma ou Notao Somatrio

129
m n

Observao: Quando aparece a notao


j =1 k =1

fjk , signica que estamos somando primeiro em k e

depois somamos o resultado em j . Podemos formar tambm somas triplas:


m n o

fijk ,
i=1 j =1 k =1

ou innitas somas.

Exerccios
i) Mostre que:
n n

i)
k =1 n

(fk + 1) =
k =1 n

fk fk
n

+n
n

ii)
k =1 n

(fk gk ) =

gk
k =1 n

k =1

iii)
k =1

(fk + gk )a = a
k =1

fk + a
k =1

gk

ii) Sendo u e v vetores, mostre que: i) O mdulo do vetor u pode ser expresso como:
n

||u|| =

uk 2
k =1

ii) O produto escalar de u por v pode ser expresso como:


n

uv =

k =1

uk vk

..............................................................................................

B
Matrizes Elementares
Denio B.1: Denimos uma matriz elementar como a matriz que resulta da aplicao de uma, e somente uma, operao elementar sobre as linhas de In . Denotaremos por E uma matriz elementar. Em alguns casos, vamos representar uma matriz elementar por Eij , onde i, j , signica que a linha i recebeu uma operao elementar da linha j .
E XEMPLO 102: As matrizes a seguir so matrizes elementares. i) E22 = ii) E42 iii) E13 1 0 0 3 Multiplicando a segunda linha de I2 por 3. 1 0 = 0 0 0 0 0 1 0 0 1 0 3 0 1 0 1 0 0

l4 l2 .

1 0 = 0 1 0 0

l1 l1 + 3 l3 .

Observao: Observe que matrizes elementares so sempre quadradas. Teorema B.2: Sejam Emm uma matriz elementar e Amn uma matriz qualquer. Ento, E A ser igual a matriz obtida da aplicao da mesma operao elementar que originou E na matriz A. Demonstrao: Temos trs casos a considerar:

B Matrizes Elementares

131

1o Caso: Corresponde a primeira operao elementar, ou seja, efetuar a troca de duas linhas. Para isso, considere as matrizes Amn e Im , denidas como: 1 0 ... 0 0 ... 0 0 a11 a12 . . . a1n 0 1 . . . 0 0 . . . 0 0 a21 a22 . . . a2n . . . . . .. . . . . . . . . . . . . . . . . . . . . . . . 0 0 . . . 1 0 . . . 0 0 i A= ai1 ai2 . . . ain e I = 0 0 . . . 0 1 . . . 0 0 j aj 1 aj 2 . . . ajn . . . . . .. . . . . . . . . . . . . . . . . . . . . . . . 0 0 . . . 0 0 . . . 1 0 amn am1 am2 0 0 ... 0 0 ... 0 1 trocando a linhai com a linhaj na matriz A, temos: a11 a12 a21 a22 . . . . . . A1 = aj 1 aj 2 ai1 ai2 . . . . . . am1 am2 a1n a2n . . . . . . ajn . . . ain . . . amn ... ... 0 0 0 0 . . . . . . 0 0 j 0 0 i . . . . . . 1 0 0 1 a1n a2n . . . . . . ain = . . . ajn . . . amn ... ...

fazendo a mesma troca de linhas na matriz Im , temos 1 0 ... 0 0 ... 0 1 . . . 0 0 . . . . . . .. . . . . . . . . . . 0 0 . . . 0 1 . . . Eji = 0 0 . . . 1 0 . . . . . . . .. . . . . . . . . . 0 0 . . . 0 0 . . . 0 0 ... 0 0 ... multiplicando Eji e A, temos: 1 0 0 1 . . . . . . 0 0 Eji A = 0 0 . . . . . . 0 0 0 0 ... 0 0 ... 0 0 . .. . . . . . . ... 0 1 ... 1 0 . . . . . .

..............................................................................................

... 0 0 a11 a12 . . . 0 0 a a22 . . 21 . . . . . . . . . . . . . 0 0 ai1 ai2 . . . 0 0 aj 1 aj 2 . .. . . . . . . . . . . . . . . . 0 0 . . . 1 0 am1 am2 ... 0 0 ... 0 1

B Matrizes Elementares

132 a11 a21 . . . = aj 1 ai1 . . . a12 a22 . . . aj 2 ai2 . . . a1n a2n . . . . . . ajn = A1 . . . ain . . . amn ... ... 1 ... 0 ... 0 . . . .. . . . . . . . I = 0 . . . 1 . . . 0 i . . . . . . . .. . . . 0 ... 0 ... 1

am1 am2

2o Caso: Segunda operao elementar. Sejam as matrizes Amn e Im , a11 a12 . . . a1n . . . . . . . . . A = ai1 ai2 . . . ain . . . . . . . . . am1 am2 . . . amn

multiplicando a linhai da matriz A por uma constante k no nula, temos a11 a12 . . . a1n . . . . . . . . . A2 = kai1 kai2 . . . kain . . . . . . . . . am1 am2 . . . amn multiplicando a linhai da matriz Im pela mesma constante k, temos 1 ... 0 ... 0 . . . .. . . . . . . . Eii = 0 . . . k . . . 0 i . . .. . . . . . . . . 0 ... 0 ... 1

fazendo a multiplicao da matriz elementar Eii pela matriz A, obtemos

a11 a12 . . . a1n 1 ... 0 ... 0 a11 a12 . . . a1n . . . . . . . . . .. . . . . . . . . . . . . . . . . . . . Eii A = 0 . . . k . . . 0 ai1 ai2 . . . ain = kai1 kai2 . . . kain = A2 . . . . . . .. . . . . . . . . . . . . . . . . . . . . . . 0 ... 0 ... 1 am1 am2 . . . amn am1 am2 . . . amn

Portanto, tambm vlido para a segunda operao elementar.

..............................................................................................

B Matrizes Elementares

133

3o Caso: Vamos provar agora que vale tambm para a terceira operao elementar. Sejam as mesmas matrizes Amn e Im do item i). Fazendo a seguinte operao elementar lj lj + kli na matriz A, temos a11 a12 ... a1n a22 ... a2n a21 . . . . . . . . . A3 = ai1 ai2 ... ain aj 1 + kai1 aj 2 + kai2 . . . ajn + kain . . . . . . . . . am1 am2 amn aplicando a mesma operao elementar na matriz Im , temos 1 0 ... 0 0 ... 0 1 . . . 0 0 . . . . . . . ... . . . . . . . . 0 0 . . . 1 0 . . . Eji = 0 0 . . . 1 1 . . . . . . . .. . . . . . . . . . 0 0 . . . 0 0 . . . 0 0 ... 0 0 ... 0 0 0 0 . . . . . . 0 0 0 0 . . . . . . 1 0 0 1

fazendo a multiplicao da matriz elementar Eji pela matriz A, obtemos 1 0 ... 0 0 ... 0 0 a11 a12 . . . a1n 0 1 . . . 0 0 . . . 0 0 a21 a22 . . . a2n . . . . . . . . . . . . . . . . . . . . . . . . . . . . . . 0 0 . . . 1 0 . . . 0 0 Eji A = ai1 ai2 . . . ain = 0 0 . . . 1 1 . . . 0 0 aj 1 aj 2 . . . ajn . . . . . .. . . . . . . . . . . . . . . . . . . . . . . . 0 0 . . . 0 0 . . . 1 0 am1 am2 amn 0 0 ... 0 0 ... 0 1 a11 a12 ... a1n a22 ... a2n a21 . . . . . . . . . = A3 = ai1 ai2 ... ain aj 1 + kai1 aj 2 + kai2 . . . ajn + kain . . . . . . . . . am1 am2 amn

Assim, conclumos a demonstrao do teorema, mostrando que o mesmo vale para os trs tipos de operaes elementares.

..............................................................................................

B Matrizes Elementares

134

Corolrio B.3: Uma matriz elementar invertvel e a inversa tambm uma matriz elementar. Demonstrao: Seja E uma matriz elementar, ento, est matriz foi obtida atravs de uma operao elementar sobre as linhas de In . Seja E0 uma matriz elementar que corresponde a operao efetuada sobre as linhas de E tal que, essa operao transforma E de volta na matriz In . Logo, pelo teorema anterior, temos E0 E = EE0 = In Portanto, E0 a inversa de E .

Vamos mostrar agora, um procedimento para obter a inversa de uma matriz, usando matrizes elementares. Suponhamos que a matriz A foi reduzida a In por uma seqncia de produtos de matrizes elementares, isto , Ek . . . E2 E1 A = In como as matrizes elementares so invertveis, podemos expressar A por A = E1 1 E2 1 . . . Ek 1 In tomando a inversa de ambos os lados, temos
1 A1 = E1 E2 . . . Ek In

e, como a inversa da In a prpria In , obtemos A1 = E1 E2 . . . Ek com isso, podemos concluir que a mesma seqncia de produtos de matrizes elementares que reduziram a matriz A a In , geram A1 se aplicadas a In . Nota: Lembrando da decomposio LU de uma matriz, quando aplicamos a eliminao gaussiana na matriz A para encontrarmos a matriz U , estamos na verdade, multiplicando a matriz A, por uma seqncia de matrizes elementares, de modo que Ek . . . E2 E1 A = U assim, temos Ek . . . E2 E1 A = U Ek 1 Ek . . . E2 E1 A = Ek 1 U . . . E2 1 E2 E1 A = E2 1 . . . Ek 1 U
In In

E1 1 E1 A = E1 1 E2 1 . . . Ek 1 U A = E1 1 E2 1 . . . Ek 1 U
In

..............................................................................................

B Matrizes Elementares

135

portanto, a seqncia E1 1 E2 1 . . . Ek 1 de matrizes elementares multiplicado pela U gera a matriz A. Logo, conclumos que L = E1 1 E2 1 . . . Ek 1 (B.1) Poderamos pensar em resolver um sistema linear Ax = b, simplesmente encontrando a inversa de A e fazer x = A1 b, supondo que A seja invertvel. No entanto, esse mtodo no vantajoso, pois faramos muito mais operaes do que usando a fatorao LU ou P LU .

Exerccios
1) Encontre a inversa de cada matriz dada, se possvel. a) 1 3 2 6 b) 2 1 2 3 4 6 7 6 2 1 1 2 3 4 1 2 0 2 1 3 1 4 2 1 5

c)

2) Prove que, se Ax = b um sistema linear com n equaes e n incgnitas e a matriz A dos coecientes invertvel, ento o sistema tem uma nica soluo, a saber, x = A1 b.

..............................................................................................

C
O Processo de Gauss-Jordan

D
Inversas Generalizadas
Neste apndice, vamos estudar a existncia de inversa para matrizes do tipo Amn . Denio D.1: Seja a matriz A Rmn , com m < n. Se existe uma matriz B Rnm , tal que AB = I Rmm , dizemos que A tem inversa direita. Para encontrarmos essa tal matriz B , considere
| | | | | |

B = B1 B2 . . . Bm Da,
| | | | | |

AB = AB1 AB2 . . . ABm portanto, se estamos querendo que AB = I , estamos impondo AB1 = e1 AB2 = e2 . . . AB = e m m

Assim, temos m sistemas lineares e, portanto, a matriz A tem inversa direita se, e somente se, for possvel resolver os m sistemas acima. Ento, para garantir que estes sistemas tenham soluo precisamos impor a hiptese de que posto(A) = m.

D Inversas Generalizadas

138

Teorema D.2: Se A Rmn com posto(A) = m, ento existe B Rnm tal que AB = I , ou seja, A admite inversa direita. Note que, se n > m a matriz A no admite inversa direita. Observao: A matriz B no nica. Pois nos sistemas AB1 = e1 AB2 = e2 . . . AB = e m m

temos (m n) variveis livres, e portanto, innitas maneiras de escolher B1 , B2 , . . . , Bm .

Denio D.3: Seja a matriz A Rmn , com m > n. Se existe uma matriz C Rnm tal que CA = I Rnn , dizemos que A tem inversa esquerda. Do mesmo modo, considere T C1 T C2 C= . . .
T Cn

ento, queremos CA = I , ou seja,

AT C T = I Rnn . Assim, temos que resolver os n sistemas T A C1 = e1 AT C2 = e2 . . . AT C = e n n

e, semelhantemente ao outro caso, o sistema acima tem soluo se posto(A) = n. Teorema D.4: Se A Rmn com posto(A) = n, ento existe C Rnm , tal que CA = I , ou seja, A admite inversa esquerda. Note que, se m < n a matriz A no admite inversa esquerda.

..............................................................................................

D Inversas Generalizadas

139

Observao: A matriz C no nica, pois, novamente, nos sistemas AT C1 = e1 AT C2 = e2 . . . AT C = e n n


E XEMPLO 103: Seja

temos (n m) variveis livres, e portanto, innitas maneiras de escolher c1 , c2 , . . . , cn .


4 A = 0 0 C=
1 4

ento a inversa esquerda de A

0 5 0 0
1 5

c1 c2

com c1 , c2 R. E XEMPLO 104: Seja A= ento, a inversa direita de A 4 0 0 5


1 4

0 0 0
1 5

com b1 , b2 R.

B = 0 b1

b2

Teorema D.5: Seja A Rmn : i) Se posto(A) = m, ento B Rnm tal que AB = I , onde B = AT (AAT )1 ii) Se posto(A) = n, ento C Rnm tal que CA = I , onde C = (AT A)1 AT (D.2) (D.1)

No caso i), chame B = AT y , para algum y Rmm . Ento (AAT )y = I . Como A tem posto(A) = m, segue que, AAT invertvel, logo y = (AAT )1 ..............................................................................................

D Inversas Generalizadas

140

e da, B = AT (AAT )1 . No caso ii), chame C = xAT , para algum x Rnn . Ento x(AT A = I ). Como A tem posto(A) = n, segue que, AT A invertvel, logo x = (AT A)1 e da, C = (AT A)1 AT . Observao: As matrizes B e C acima, denominadas de inversa direita de A e inversa esquerda de A, respectivamente, tambm so chamadas de pseudo-inversas de A, e denotadas muitas vezes por A+ e A , respectivamente.

..............................................................................................

Referncias Bibliogrcas

[1] ANTON, Howard; BUSBY, Robert C. lgebra Linear Contempornea. traduao Claus Ivo Doering. Bookman, Porto Alegre, 2006 [2] BOLDRINI, Jos L.; COSTA, Sueli I. R.; FIGUEIREDO, Vera L.; WETZLER, Henry G. lgebra Linear. Ed. Harbra Ltda., So Paulo, 3a edio, 1986. [3] CARVALHO, Joo Pitombeira de. lgebra Linear - Introduo. Livros Tcnicos e Cientifcos Editora S.A., Rio de Janeiro, 2a edio, 1977. [4] GONALVES, Adilson; SOUZA, Rita M. L. de. Introduo lgebra Linear. Edgard Blcher, Rio de Janeiro, 1977. [5] HOFFMAN, Kenneth; KUNZE, Ray. Linear Algebra. Prentice-Hall, Inc., Englewood Cliffs, 1961. [6] IEZZI, Gelson; HAZZAN, Samuel. Fundamentos de Matemtica Elementar, 4: Seqncias, Matrizes, Determinantes, Sistemas. Ed Atual, So Paulo, 6a edio, 1993. [7] KOLMAN, Bernard. Introduo lgebra Linear com Aplicaes. Prentice Hall do Brasil, Rio de Janeiro, 6a edio, 1998. [8] LADEIRA, Luiz Augusto da Costa. lgebra Linear e Equaes Diferenciais. Departamento de Matemtica, USP, So Carlos, 2004. [9] LIMA, Elon L. lgebra Linear. IMPA, Rio de Janeiro, 2a edio, 1996. [10] ROQUE, Waldir L. Introduo ao Clculo Numrico: Um Texto Integrado com DERIVE c . Ed. Atlas, So Paulo, 2000. [11] SANTOS, Nathan M. dos. Vetores e Matrizes: Uma Introduo lgebra Linear. Thomson Learning, So Paulo, 4a edio revista e ampliada, 2007. [12] SANTOS, Reginaldo J. Um Curso de Geometria Analtica e lgebra Linear. Imprensa Universitria da UFMG, UFMG, Belo Horizonte, 2002. [13] STRANG, Gilbert. Linear Algebra and its Applications. Harcourt Brace Jovanovich, Inc., Orlando, 3a edio, 1988.

Vous aimerez peut-être aussi